Test exam 2 practice

¡Supera tus tareas y exámenes ahora con Quizwiz!

Changes in clarity and volume of spoken sounds during auscultation of the lungs can help yo a. crepitus from stridor. b. a foreign body from a purulent exudate. c. pulmonary edema from pleurisy. d. a right from left tracheal deviation. e. consolidation from airway constriction

: E - consolidation from airway constriction

The nurse is performing an ear examination of an 80-year-old patient. Which of these findings would be considered normal? a. High-tone frequency loss b. Increased elasticity of the pinna c. Thin, translucent membrane d. Shiny, pink tympanic membrane

A A high-tone frequency hearing loss is apparent for those affected with presbycusis, the hearing loss that occurs with aging. The pinna loses elasticity, causing earlobes to be pendulous. The eardrum may be whiter in color and more opaque and duller in the older person than in the younger adult.

To assess a rectal temperature accurately in an adult, the nurse would: a. Use a lubricated blunt tip thermometer. b. Insert the thermometer 2 to 3 inches into the rectum. c. Leave the thermometer in place up to 8 minutes if the patient is febrile. d. Wait 2 to 3 minutes if the patient has recently smoked a cigarette.

A A lubricated rectal thermometer (with a short, blunt tip) is inserted only 2 to 3 cm (1 inch) into the adult rectum and left in place for 2 minutes. Cigarette smoking does not alter rectal temperatures.

A 70-year-old patient is being seen in the clinic for severe exacerbation of his heart failure. Which of these findings is the nurse most likely to observe in this patient? a. Shortness of breath, orthopnea, paroxysmal nocturnal dyspnea, and ankle edema b. Rasping cough, thick mucoid sputum, wheezing, and bronchitis c. Productive cough, dyspnea, weight loss, anorexia, and tuberculosis d. Fever, dry nonproductive cough, and diminished breath sounds

A A person with heart failure often exhibits increased respiratory rate, shortness of breath on exertion, orthopnea, paroxysmal nocturnal dyspnea, nocturia, ankle edema, and pallor in light-skinned individuals. A patient with rasping cough, thick mucoid sputum, and wheezing may have bronchitis. Productive cough, dyspnea, weight loss, and dyspnea indicate tuberculosis; fever, dry nonproductive cough, and diminished breath sounds may indicate Pneumocystis jiroveci (P. carinii) pneumonia (see Table 18-8).

A patient tells the nurse that he has noticed that one of his moles has started to burn and bleed. When assessing his skin, the nurse pays special attention to the danger signs for pigmented lesions and is concerned with which additional finding? a. Color variation b. Border regularity c. Symmetry of lesions d. Diameter of less than 6 mm

A Abnormal characteristics of pigmented lesions are summarized in the mnemonic ABCD: asymmetry of pigmented lesion, border irregularity, color variation, and diameter greater than 6 mm.

When performing an otoscopic examination of a 5-year-old child with a history of chronic ear infections, the nurse sees that his right tympanic membrane is amber-yellow in color and that air bubbles are visible behind the tympanic membrane. The child reports occasional hearing loss and a popping sound with swallowing. The preliminary analysis based on this information is that the child: a. Most likely has serous otitis media. b. Has an acute purulent otitis media. c. Has evidence of a resolving cholesteatoma. d. Is experiencing the early stages of perforation.

A An amber-yellow color to the tympanic membrane suggests serum or pus in the middle ear. Air or fluid or bubbles behind the tympanic membrane are often visible. The patient may have feelings of fullness, transient hearing loss, and a popping sound with swallowing. These findings most likely suggest that the child has serous otitis media. The other responses are not correct.

During an examination of a 3-year-old child, the nurse notices a bruit over the left temporal area. The nurse should: a. Continue the examination because a bruit is a normal finding for this age. b. Check for the bruit again in 1 hour. c. Notify the parents that a bruit has been detected in their child. d. Stop the examination, and notify the physician.

A Bruits are common in the skull in children under 4 or 5 years of age and in children with anemia. They are systolic or continuous and are heard over the temporal area.

A 2-week-old infant can fixate on an object but cannot follow a light or bright toy. The nurse would: a. Consider this a normal finding. b. Assess the pupillary light reflex for possible blindness. c. Continue with the examination, and assess visual fields. d. Expect that a 2-week-old infant should be able to fixate and follow an object.

A By 2 to 4 weeks an infant can fixate on an object. By the age of 1 month, the infant should fixate and follow a bright light or toy.

While discussing the history of a 6-month-old infant, the mother tells the nurse that she took a significant amount of aspirin while she was pregnant. What question would the nurse want to include in the history? a. "Does your baby seem to startle with loud noises?" b. "Has your baby had any surgeries on her ears?" c. "Have you noticed any drainage from her ears?" d. "How many ear infections has your baby had since birth?"

A Children at risk for a hearing deficit include those exposed in utero to a variety of conditions, such as maternal rubella or to maternal ototoxic drugs.

The nurse is performing an assessment on a 7-year-old child who has symptoms of chronic watery eyes, sneezing, and clear nasal drainage. The nurse notices the presence of a transverse line across the bridge of the nose, dark blue shadows below the eyes, and a double crease on the lower eyelids. These findings are characteristic of: a. Allergies. b. Sinus infection. c. Nasal congestion. d. Upper respiratory infection.

A Chronic allergies often develop chronic facial characteristics and include blue shadows below the eyes, a double or single crease on the lower eyelids, open-mouth breathing, and a transverse line on the nose.

The nurse just noted from the medical record that the patient has a lesion that is confluent in nature. On examination, the nurse expects to find: a. Lesions that run together. b. Annular lesions that have grown together. c. Lesions arranged in a line along a nerve route. d. Lesions that are grouped or clustered together.

A Confluent lesions (as with urticaria [hives]) run together. Grouped lesions are clustered together. Annular lesions are circular in nature. Zosteriform lesions are arranged along a nerve route.

During auscultation of breath sounds, the nurse should correctly use the stethoscope in which of the following ways? a. Listening to at least one full respiration in each location b. Listening as the patient inhales and then going to the next site during exhalation c. Instructing the patient to breathe in and out rapidly while listening to the breath sounds d. If the patient is modest, listening to sounds over his or her clothing or hospital gown

A During auscultation of breath sounds with a stethoscope, listening to one full respiration in each location is important. During the examination, the nurse should monitor the breathing and offer times for the person to breathe normally to prevent possible dizziness.

The nurse is preparing to do an otoscopic examination on a 2-year-old child. Which one of these reflects the correct procedure? a. Pulling the pinna down b. Pulling the pinna up and back c. Slightly tilting the child's head toward the examiner d. Instructing the child to touch his chin to his chest

A For an otoscopic examination on an infant or on a child under 3 years of age, the pinna is pulled down. The other responses are not part of the correct procedure.

When evaluating the temperature of older adults, the nurse should remember which aspect about an older adult's body temperature? a. The body temperature of the older adult is lower than that of a younger adult. b. An older adult's body temperature is approximately the same as that of a young child. c. Body temperature depends on the type of thermometer used. d. In the older adult, the body temperature varies widely because of less effective heat control mechanisms.

A In older adults, the body temperature is usually lower than in other age groups, with a mean temperature of 36.2° C.

The nurse is performing an oral assessment on a 40-year-old Black patient and notices the presence of a 1 cm, nontender, grayish-white lesion on the left buccal mucosa. Which one of these statements is true? This lesion is: a. Leukoedema and is common in dark-pigmented persons. b. The result of hyperpigmentation and is normal. c. Torus palatinus and would normally be found only in smokers. d. Indicative of cancer and should be immediately tested.

A Leukoedema, a grayish-white benign lesion occurring on the buccal mucosa, is most often observed in Blacks.

A 68-year-old woman is in the eye clinic for a checkup. She tells the nurse that she has been having trouble reading the paper, sewing, and even seeing the faces of her grandchildren. On examination, the nurse notes that she has some loss of central vision but her peripheral vision is normal. These findings suggest that she may have: a. Macular degeneration. b. Vision that is normal for someone her age. c. The beginning stages of cataract formation. d. Increased intraocular pressure or glaucoma.

A Macular degeneration is the most common cause of blindness. It is characterized by the loss of central vision. Cataracts would show lens opacity. Chronic open-angle glaucoma, the most common type of glaucoma, involves a gradual loss of peripheral vision. These findings are not consistent with vision that is considered normal at any age.

A patient's vision is recorded as 20/80 in each eye. The nurse interprets this finding to mean that the patient: a. Has poor vision. b. Has acute vision. c. Has normal vision. d. Is presbyopic.

A Normal visual acuity is 20/20 in each eye; the larger the denominator, the poorer the vision.

The nurse is auscultating the lungs of a patient who had been sleeping and notices short, popping, crackling sounds that stop after a few breaths. The nurse recognizes that these breath sounds are: a. Atelectatic crackles that do not have a pathologic cause. b. Fine crackles and may be a sign of pneumonia. c. Vesicular breath sounds. d. Fine wheezes.

A One type of adventitious sound, atelectatic crackles, does not have a pathologic cause. They are short, popping, crackling sounds that sound similar to fine crackles but do not last beyond a few breaths. When sections of alveoli are not fully aerated (as in people who are asleep or in older adults), they deflate slightly and accumulate secretions. Crackles are heard when these sections are expanded by a few deep breaths. Atelectatic crackles are heard only in the periphery, usually in dependent portions of the lungs, and disappear after the first few breaths or after a cough.

When assessing the tongue of an adult, the nurse knows that an abnormal finding would be: a. Smooth glossy dorsal surface. b. Thin white coating over the tongue. c. Raised papillae on the dorsal surface. d. Visible venous patterns on the ventral surface.

A The dorsal surface of the tongue is normally roughened from papillae. A thin white coating may be present. The ventral surface may show veins. Smooth, glossy areas may indicate atrophic glossitis (see Table 16-5).

When considering a nutritional assessment, the nurse is aware that the most common anthropometric measurements include: a. Height and weight. b. Leg circumference. c. Skinfold thickness of the biceps. d. Hip and waist measurements.

A The most commonly used anthropometric measures are height, weight, triceps skinfold thickness, elbow breadth, and arm and head circumferences.

The salivary gland that is the largest and located in the cheek in front of the ear is the _________ gland. a. Parotid b. Stensen's c. Sublingual d. Submandibular

A The mouth contains three pairs of salivary glands. The largest, the parotid gland, lies within the cheeks in front of the ear extending from the zygomatic arch down to the angle of the jaw. The Stensen's duct (not gland) drains the parotid gland onto the buccal mucosa opposite the second molar. The sublingual gland is located within the floor of the mouth under the tongue. The submandibular gland lies beneath the mandible at the angle of the jaw.

The nurse is testing a patient's visual accommodation, which refers to which action? a. Pupillary constriction when looking at a near object b. Pupillary dilation when looking at a far object c. Changes in peripheral vision in response to light d. Involuntary blinking in the presence of bright light

A The muscle fibers of the iris contract the pupil in bright light and accommodate for near vision, which also results in pupil constriction. The other responses are not correct.

The nurse is evaluating patients for obesity-related diseases by calculating the waist-to-hip ratios. Which one of these patients would be at increased risk? a. 29-year-old woman whose waist measures 33 inches and hips measure 36 inches b. 32-year-old man whose waist measures 34 inches and hips measure 36 inches c. 38-year-old man whose waist measures 35 inches and hips measure 38 inches d. 46-year-old woman whose waist measures 30 inches and hips measure 38 inches

A The waist-to-hip ratio assesses body fat distribution as an indicator of health risk. A waist-to-hip ratio of 1.0 or greater in men or 0.8 or greater in women is indicative of android (upper body obesity) and increasing risk for obesity-related disease and early death. The 29-year-old woman has a waist-to-hip ratio of 0.92, which is greater than 0.8. The 32-year-old man has a waist-to-hip ratio of 0.94; the 38-year-old man has a waist-to-hip ratio of 0.92; the 46-year-old woman has a waist-to-hip ratio of 0.78.

During an examination, the nurse finds that a patient has excessive dryness of the skin. The best term to describe this condition is: a. Xerosis. b. Pruritus. c. Alopecia. d. Seborrhea.

A Xerosis is the term used to describe skin that is excessively dry. Pruritus refers to itching, alopecia refers to hair loss, and seborrhea refers to oily skin.

It is not unusual to find postauricular and occipital nodes in a. children younger than 2 years of age. b. school-age children. c. adolescents. d. adults. e. older adults.

A - children younger than 2 years of age.

Increased oxygen tension in the arterial blood of a newborn infant causes a. closure of the ductus arteriosus. b. hyperinflation of the lungs. c. passive respiratory movements. d. reopening of the foramen ovale. e. the pulmonary arteries to contract.

A - closure of the ductus arteriosus.

The characteristic that best differentiates psoriasis from other skin abnormalities is the a. color of the scales. b. formation of tiny papules. c. general distribution over the body. d. recurrence. e. loss of hair.

A - color of the scales. Unlike other skin conditions, silvery papules and plaques characterize psoriasis.

The nails of older adults grow slowly because of a. decreased circulation. b. dietary deficiencies. c. fungal infections. d. low hormone levels. e. high estrogen levels.

A - decreased circulation.

The skin repairs epidermal wounds by a. exaggerating cell replacement. b. excreting lactic acid. c. producing vitamins. d. providing a mechanical barrier. e. increasing the vascularity of the epidermis

A - exaggerating cell replacement.

Palpation of the scalene triangle for supraclavicular nodes should be done with a. fingers hooked over the clavicle next to the sternocleidomastoid muscle. b. fingers along the depression above the medial humeral condyle. c. the pads of three fingers in a superficial circular motion. d. tissue rolled gently against the chest wall, moving sideways. e. the pads of the fingertips along the anterior surface of the trapezius muscle.

A - fingers hooked over the clavicle next to the sternocleidomastoid muscle

A 4 to 3-cm, rough, elevated area of psoriasis is an example of a a. plaque. b. patch. c. macule. d. papule. e. wheal.

A - plaque.

While measuring a patient's blood pressure, the nurse uses the proper technique to obtain an accurate reading. Which of these situations will result in a falsely high blood pressure reading? Select all that apply. a. The person supports his or her own arm during the blood pressure reading. b. The blood pressure cuff is too narrow for the extremity. c. The arm is held above level of the heart. d. The cuff is loosely wrapped around the arm. e. The person is sitting with his or her legs crossed. f. The nurse does not inflate the cuff high enough.

A, B, D, E Several factors can result in blood pressure readings that are too high or too low. Having the patient's arm held above the level of the heart is one part of the correct technique. (Refer to Table 9-5, Common Errors in Blood Pressure Measurement.)

The nurse is preparing to palpate the thorax and abdomen of a patient. Which of these statements describes the correct technique for this procedure? Select all that apply. a. Warm the hands first before touching the patient. b. For deep palpation, use one long continuous palpation when assessing the liver. c. Start with light palpation to detect surface characteristics. d. Use the fingertips to examine skin texture, swelling, pulsation, and presence of lumps. e. Identify any tender areas, and palpate them last. f. Use the palms of the hands to assess temperature of the skin.

A, C, D, E The hands should always be warmed before beginning palpation. Intermittent pressure rather than one long continuous palpation is used; any tender areas are identified and palpated last. Fingertips are used to examine skin texture, swelling, pulsation, and the presence of lumps. The dorsa (backs) of the hands are used to assess skin temperature because the skin on the dorsa is thinner than on the palms.

The nurse is preparing for a certification course in skin care and needs to be familiar with the various lesions that may be identified on assessment of the skin. Which of the following definitions are correct? Select all that apply. a. Petechiae: Tiny punctate hemorrhages, 1 to 3 mm, round and discrete, dark red, purple, or brown in color b. Bulla: Elevated, circumscribed lesion filled with turbid fluid (pus) c. Papule: Hypertrophic scar d. Vesicle: Known as a friction blister e. Nodule: Solid, elevated, and hard or soft growth that is larger than 1 cm

A, D, E A pustule is an elevated, circumscribed lesion filled with turbid fluid (pus). A hypertrophic scar is a keloid. A bulla is larger than 1 cm and contains clear fluid. A papule is solid and elevated but measures less than 1 cm.

During assessment of a patient's pain, the nurse is aware that certain nonverbal behaviors are associated with chronic pain. Which of these behaviors are associated with chronic pain? Select all that apply. a. Sleeping b. Moaning c. Diaphoresis d. Bracing e. Restlessness f. Rubbing

A, D, F Behaviors that have been associated with chronic pain include bracing, rubbing, diminished activity, sighing, and changes in appetite. In addition, those with chronic pain may sleep in an attempt at distraction. The other behaviors are associated with acute pain.

While measuring a patient's blood pressure, the nurse recalls that certain factors, such as __________, help determine blood pressure. a. Pulse rate b. Pulse pressure c. Vascular output d. Peripheral vascular resistance

ANS: The level of blood pressure is determined by five factors: cardiac output, peripheral vascular resistance, volume of circulating blood, viscosity, and elasticity of the vessel walls.

A semiconscious woman is brought to the emergency department after she was found on the floor in her kitchen. Her face, nail beds, lips, and oral mucosa are a bright cherry-red color. The nurse suspects that this coloring is due to: a. Polycythemia. b. Carbon monoxide poisoning. c. Carotenemia. d. Uremia.

B A bright cherry-red coloring in the face, upper torso, nail beds, lips, and oral mucosa appears in cases of carbon monoxide poisoning.

A patient comes to the clinic complaining of a cough that is worse at night but not as bad during the day. The nurse recognizes that this cough may indicate: a. Pneumonia. b. Postnasal drip or sinusitis. c. Exposure to irritants at work. d. Chronic bronchial irritation from smoking.

B A cough that primarily occurs at night may indicate postnasal drip or sinusitis. Exposure to irritants at work causes an afternoon or evening cough. Smokers experience early morning coughing. Coughing associated with acute illnesses such as pneumonia is continuous throughout the day.

During an examination of a patient's abdomen, the nurse notes that the abdomen is rounded and firm to the touch. During percussion, the nurse notes a drumlike quality of the sounds across the quadrants. This type of sound indicates: a. Constipation. b. Air-filled areas. c. Presence of a tumor. d. Presence of dense organs.

B A musical or drumlike sound (tympany) is heard when percussion occurs over an air-filled viscus, such as the stomach or intestines

The nurse is performing the diagnostic positions test. Normal findings would be which of these results? a. Convergence of the eyes b. Parallel movement of both eyes c. Nystagmus in extreme superior gaze d. Slight amount of lid lag when moving the eyes from a superior to an inferior position

B A normal response for the diagnostic positions test is parallel tracking of the object with both eyes. Eye movement that is not parallel indicates a weakness of an extraocular muscle or dysfunction of the CN that innervates it.

A patient with pleuritis has been admitted to the hospital and complains of pain with breathing. What other key assessment finding would the nurse expect to find upon auscultation? a. Stridor b. Friction rub c. Crackles d. Wheezing

B A patient with pleuritis will exhibit a pleural friction rub upon auscultation. This sound is made when the pleurae become inflamed and rub together during respiration. The sound is superficial, coarse, and low-pitched, as if two pieces of leather are being rubbed together. Stridor is associated with croup, acute epiglottitis in children, and foreign body inhalation. Crackles are associated with pneumonia, heart failure, chronic bronchitis, and other diseases (see Table 18-6). Wheezes are associated with diffuse airway obstruction caused by acute asthma or chronic emphysema.

The nurse suspects that a patient has otitis media. Early signs of otitis media include which of these findings of the tympanic membrane? a. Red and bulging b. Hypomobility c. Retraction with landmarks clearly visible d. Flat, slightly pulled in at the center, and moves with insufflation

B An early sign of otitis media is hypomobility of the tympanic membrane. As pressure increases, the tympanic membrane begins to bulge.

The nurse would use bimanual palpation technique in which situation? a. Palpating the thorax of an infant b. Palpating the kidneys and uterus c. Assessing pulsations and vibrations d. Assessing the presence of tenderness and pain

B Bimanual palpation requires the use of both hands to envelop or capture certain body parts or organs such as the kidneys, uterus, or adnexa. The other situations are not appropriate for bimanual palpation.

A patient reports excruciating headache pain on one side of his head, especially around his eye, forehead, and cheek that has lasted approximately to 2 hours, occurring once or twice each day. The nurse should suspect: a. Hypertension. b. Cluster headaches. c. Tension headaches. d. Migraine headaches.

B Cluster headaches produce pain around the eye, temple, forehead, and cheek and are unilateral and always on the same side of the head. They are excruciating and occur once or twice per day and last to 2 hours each.

During palpation of the anterior chest wall, the nurse notices a coarse, crackling sensation over the skin surface. On the basis of these findings, the nurse suspects: a. Tactile fremitus. b. Crepitus. c. Friction rub. d. Adventitious sounds.

B Crepitus is a coarse, crackling sensation palpable over the skin surface. It occurs in subcutaneous emphysema when air escapes from the lung and enters the subcutaneous tissue, such as after open thoracic injury or surgery.

During an assessment of an infant, the nurse notes that the fontanels are depressed and sunken. The nurse suspects which condition? a. Rickets b. Dehydration c. Mental retardation d. Increased intracranial pressure

B Depressed and sunken fontanels occur with dehydration or malnutrition. Mental retardation and rickets have no effect on the fontanels. Increased intracranial pressure would cause tense or bulging and possibly pulsating fontanels.

A 22-year-old woman comes to the clinic because of severe sunburn and states, "I was out in the sun for just a couple of minutes." The nurse begins a medication review with her, paying special attention to which medication class? a. Nonsteroidal antiinflammatory drugs for pain b. Tetracyclines for acne c. Proton pump inhibitors for heartburn d. Thyroid replacement hormone for hypothyroidism

B Drugs that may increase sunlight sensitivity and give a burn response include sulfonamides, thiazide diuretics, oral hypoglycemic agents, and tetracycline.

A nurse is helping at a health fair at a local mall. When taking blood pressures on a variety of people, the nurse keeps in mind that: a. After menopause, blood pressure readings in women are usually lower than those taken in men. b. The blood pressure of a Black adult is usually higher than that of a White adult of the same age. c. Blood pressure measurements in people who are overweight should be the same as those of people who are at a normal weight. d. A teenager's blood pressure reading will be lower than that of an adult.

B In the United States, a Black adult's blood pressure is usually higher than that of a White adult of the same age. The incidence of hypertension is twice as high in Blacks as it is in Whites. After menopause, blood pressure in women is higher than in men; blood pressure measurements in people who are obese are usually higher than in those who are not overweight. Normally, a gradual rise occurs through childhood and into the adult years.

During an assessment of a patient who has been homeless for several years, the nurse notices that his tongue is magenta in color, which is an indication of a deficiency in what mineral and/or vitamin? a. Iron b. Riboflavin c. Vitamin D and calcium d. Vitamin C

B Magenta tongue is a sign of riboflavin deficiency. In contrast, a pale tongue is probably attributable to iron deficiency. Vitamin D and calcium deficiencies cause osteomalacia in adults, and a vitamin C deficiency causes scorbutic gums.

Which of these specific measurements is the best index of a child's general health? a. Vital signs b. Height and weight c. Head circumference d. Chest circumference

B Physical growth, measured by height and weight, is the best index of a child's general health.

When assessing the pulse of a 6-year-old boy, the nurse notices that his heart rate varies with his respiratory cycle, speeding up at the peak of inspiration and slowing to normal with expiration. The nurse's next action would be to: a. Immediately notify the physician. b. Consider this finding normal in children and young adults. c. Check the child's blood pressure, and note any variation with respiration. d. Document that this child has bradycardia, and continue with the assessment.

B Sinus arrhythmia is commonly found in children and young adults. During the respiratory cycle, the heart rate varies, speeding up at the peak of inspiration and slowing to normal with expiration.

The nurse is examining an infant and prepares to elicit the Moro reflex at which time during the examination? a. When the infant is sleeping b. At the end of the examination c. Before auscultation of the thorax d. Halfway through the examination

B The Moro or startle reflex is elicited at the end of the examination because it may cause the infant to cry.

Which statement about the apices of the lungs is true? The apices of the lungs: a. Are at the level of the second rib anteriorly. b. Extend 3 to 4 cm above the inner third of the clavicles. c. Are located at the sixth rib anteriorly and the eighth rib laterally. d. Rest on the diaphragm at the fifth intercostal space in the midclavicular line (MCL).

B The apex of the lung on the anterior chest is 3 to 4 cm above the inner third of the clavicles. On the posterior chest, the apices are at the level of C7.

While auscultating heart sounds, the nurse hears a murmur. Which of these instruments should be used to assess this murmur? a. Electrocardiogram b. Bell of the stethoscope c. Diaphragm of the stethoscope d. Palpation with the nurse's palm of the hand

B The bell of the stethoscope is best for soft, low-pitched sounds such as extra heart sounds or murmurs. The diaphragm of the stethoscope is best used for high-pitched sounds such as breath, bowel, and normal heart sounds.

The nurse is examining a patient who is complaining of "feeling cold." Which is a mechanism of heat loss in the body? a. Exercise b. Radiation c. Metabolism d. Food digestion

B The body maintains a steady temperature through a thermostat or feedback mechanism, which is regulated in the hypothalamus of the brain. The hypothalamus regulates heat production from metabolism, exercise, food digestion, and external factors with heat loss through radiation, evaporation of sweat, convection, and conduction.

The nurse is using an otoscope to assess the nasal cavity. Which of these techniques is correct? a. Inserting the speculum at least 3 cm into the vestibule b. Avoiding touching the nasal septum with the speculum c. Gently displacing the nose to the side that is being examined d. Keeping the speculum tip medial to avoid touching the floor of the nares

B The correct technique for using an otoscope is to insert the apparatus into the nasal vestibule, avoiding pressure on the sensitive nasal septum. The tip of the nose should be lifted up before inserting the speculum.

The nurse is conducting a visual examination. Which of these statements regarding visual pathways and visual fields is true? a. The right side of the brain interprets the vision for the right eye. b. The image formed on the retina is upside down and reversed from its actual appearance in the outside world. c. Light rays are refracted through the transparent media of the eye before striking the pupil. d. Light impulses are conducted through the optic nerve to the temporal lobes of the brain.

B The image formed on the retina is upside down and reversed from its actual appearance in the outside world. The light rays are refracted through the transparent media of the eye before striking the retina, and the nerve impulses are conducted through the optic nerve tract to the visual cortex of the occipital lobe of the brain. The left side of the brain interprets vision for the right eye.

A mother brings her 3-month-old infant to the clinic for evaluation of a cold. She tells the nurse that he has had "a runny nose for a week." When performing the physical assessment, the nurse notes that the child has nasal flaring and sternal and intercostal retractions. The nurse's next action should be to: a. Assure the mother that these signs are normal symptoms of a cold. b. Recognize that these are serious signs, and contact the physician. c. Ask the mother if the infant has had trouble with feedings. d. Perform a complete cardiac assessment because these signs are probably indicative of early heart failure.

B The infant is an obligatory nose breather until the age of 3 months. Normally, no flaring of the nostrils and no sternal or intercostal retraction occurs. Significant retractions of the sternum and intercostal muscles and nasal flaring indicate increased inspiratory effort, as in pneumonia, acute airway obstruction, asthma, and atelectasis; therefore, immediate referral to the physician is warranted. These signs do not indicate heart failure, and an assessment of the infant's feeding is not a priority at this time.

The nurse is reviewing in age-related changes in the eye for a class. Which of these physiologic changes is responsible for presbyopia? a. Degeneration of the cornea b. Loss of lens elasticity c. Decreased adaptation to darkness d. Decreased distance vision abilities

B The lens loses elasticity and decreases its ability to change shape to accommodate for near vision. This condition is called presbyopia.

The primary purpose of the ciliated mucous membrane in the nose is to: a. Warm the inhaled air. b. Filter out dust and bacteria. c. Filter coarse particles from inhaled air. d. Facilitate the movement of air through the nares.

B The nasal hairs filter the coarsest matter from inhaled air, whereas the mucous blanket filters out dust and bacteria. The rich blood supply of the nasal mucosa warms the inhaled air.

A male patient with a history of acquired immunodeficiency syndrome (AIDS) has come in for an examination and he states, "I think that I have the mumps." The nurse would begin by examining the: a. Thyroid gland. b. Parotid gland. c. Cervical lymph nodes. d. Mouth and skin for lesions.

B The parotid gland may become swollen with the onset of mumps, and parotid enlargement has been found with human immunodeficiency virus (HIV).

A patient's vision is recorded as 20/30 when the Snellen eye chart is used. The nurse interprets these results to indicate that: a. At 30 feet the patient can read the entire chart. b. The patient can read at 20 feet what a person with normal vision can read at 30 feet. c. The patient can read the chart from 20 feet in the left eye and 30 feet in the right eye. d. The patient can read from 30 feet what a person with normal vision can read from 20 feet.

B The top number indicates the distance the person is standing from the chart; the denominator gives the distance at which a normal eye can see.

When assessing an older adult, which vital sign changes occur with aging? a. Increase in pulse rate b. Widened pulse pressure c. Increase in body temperature d. Decrease in diastolic blood pressure

B With aging, the nurse keeps in mind that the systolic blood pressure increases, leading to widened pulse pressure. With many older people, both the systolic and diastolic pressures increase. The pulse rate and temperature do not increase.

While performing an assessment of the mouth, the nurse notices that the patient has a 1-cm ulceration that is crusted with an elevated border and located on the outer third of the lower lip. What other information would be most important for the nurse to assess? a. Nutritional status b. When the patient first noticed the lesion c. Whether the patient has had a recent cold d. Whether the patient has had any recent exposure to sick animals

B With carcinoma, the initial lesion is round and indurated, but then it becomes crusted and ulcerated with an elevated border. Most cancers occur between the outer and middle thirds of the lip. Any lesion that is still unhealed after 2 weeks should be referred.

A patient tells the nurse that his food simply does not have any taste anymore. The nurse's best response would be: a. "That must be really frustrating." b. "When did you first notice this change?" c. "My food doesn't always have a lot of taste either." d. "Sometimes that happens, but your taste will come back."

B With changes in appetite, taste, smell, or chewing or swallowing, the examiner should ask about the type of change and when the change occurred. These problems interfere with adequate nutrient intake. The other responses are not correct.

In barrel chest, the ratio of the anteroposterior diameter to the transverse (lateral) diameter is a. 0.7 to 0.75. b. 1.0. c. 1.3 to 1.5. d. 1.5 to 2.

B - 1.0.

When auscultating the apex of the lung, you should listen a. even with the second rib. b. 4 cm above the first rib. c. higher on the right side. d. on the convex diaphragm surface. e. directly over the clavicles.

B - 4 cm above the first rib.

To begin counting the ribs and the intercostal spaces, you begin by palpating the reference p a. distal point of the xiphoid. b. manubriosternal junction. c. suprasternal notch. d. acromion process. e. clavicle.

B - manubriosternal junction.

Age spots are also called a. seborrheic keratoses. b. senile lentigines. c. cutaneous horns. d. acrochordon. e. cutaneous tags.

B - senile lentigines Senile lentigines are irregular, round, gray-brown lesions with a rough surface that occur in su age spots.

The predominant formed elements of normal lymph fluid are a. red blood cells. b. white blood cells. c. platelets. d. antigens. e. antibodies.

B - white blood cells Lymph fluid is mostly composed of a variety of lymphocytes, minimal red blood cells, no pla antibodies according to its immune function.

The nurse is assessing a patient who is obese for signs of metabolic syndrome. This condition is diagnosed when three or more certain risk factors are present. Which of these assessment findings are risk factors for metabolic syndrome? Select all that apply. a. Fasting plasma glucose level less than 100 mg/dL b. Fasting plasma glucose level greater than or equal to 110 mg/dL c. Blood pressure reading of 140/90 mm Hg d. Blood pressure reading of 110/80 mm Hg e. Triglyceride level of 120 mg/dL

B, C Metabolic syndrome is diagnosed when three or more of the following risk factors are present: (1) fasting plasma glucose level greater than or equal to 100 mg/dL; (2) blood pressure greater than or equal to 130/85 mm Hg; (3) waist circumference greater than or equal to 40 inches for men and 35 inches for women; (4) high-density lipoprotein cholesterol less than 40 in men and less than 50 in women; and (5) triglyceride levels greater than or equal to 150 mg/dL (ATP III, 2001).

The nurse is teaching a health class to high-school boys. When discussing the topic of using smokeless tobacco (SLT), which of these statements are accurate? Select all that apply. a. One pinch of SLT in the mouth for 30 minutes delivers the equivalent of one cigarette. b. Using SLT has been associated with a greater risk of oral cancer than smoking. c. Pain is an early sign of oral cancer. d. Pain is rarely an early sign of oral cancer. e. Tooth decay is another risk of SLT because of the use of sugar as a sweetener. f. SLT is considered a healthy alternative to smoking.

B, D, E One pinch of SLT in the mouth for 30 minutes delivers the equivalent of three cigarettes. Pain is rarely an early sign of oral cancer. Many brands of SLT are sweetened with sugars, which promotes tooth decay. SLT is not considered a healthy alternative to smoking, and the use of SLT has been associated with a greater risk of oral cancer than smoking.

The nurse is preparing to examine a 6-year-old child. Which action is most appropriate? a. The thorax, abdomen, and genitalia are examined before the head. b. Talking about the equipment being used is avoided because doing so may increase the child's anxiety. c. The nurse should keep in mind that a child at this age will have a sense of modesty. d. The child is asked to undress from the waist up.

C A 6-year-old child has a sense of modesty. The child should undress him or herself, leaving underpants on and using a gown or drape. A school-age child is curious to know how equipment works, and the sequence should progress from the child's head to the toes.

The nurse notices that an infant has a large, soft lump on the side of his head and that his mother is very concerned. She tells the nurse that she noticed the lump approximately 8 hours after her baby's birth and that it seems to be getting bigger. One possible explanation for this is: a. Hydrocephalus. b. Craniosynostosis. c. Cephalhematoma. d. Caput succedaneum.

C A cephalhematoma is a subperiosteal hemorrhage that is the result of birth trauma. It is soft, fluctuant, and well defined over one cranial bone. It appears several hours after birth and gradually increases in size.

The nurse has discovered decreased skin turgor in a patient and knows that this finding is expected in which condition? a. Severe obesity b. Childhood growth spurts c. Severe dehydration d. Connective tissue disorders such as scleroderma

C Decreased skin turgor is associated with severe dehydration or extreme weight loss.

A patient has been admitted to the emergency department with a possible medical diagnosis of pulmonary embolism. The nurse expects to see which assessment findings related to this condition? a. Absent or decreased breath sounds b. Productive cough with thin, frothy sputum c. Chest pain that is worse on deep inspiration and dyspnea d. Diffuse infiltrates with areas of dullness upon percussion

C Findings for pulmonary embolism include chest pain that is worse on deep inspiration, dyspnea, apprehension, anxiety, restlessness, partial arterial pressure of oxygen (PaO2) less than 80 mm Hg, diaphoresis, hypotension, crackles, and wheezes.

The nurse knows that auscultation of fine crackles would most likely be noticed in: a. A healthy 5-year-old child. b. A pregnant woman. c. The immediate newborn period. d. Association with a pneumothorax.

C Fine crackles are commonly heard in the immediate newborn period as a result of the opening of the airways and a clearing of fluid. Persistent fine crackles would be noticed with pneumonia, bronchiolitis, or atelectasis.

During auscultation of the lungs of an adult patient, the nurse notices the presence of bronchophony. The nurse should assess for signs of which condition? a. Airway obstruction b. Emphysema c. Pulmonary consolidation d. Asthma

C Pathologic conditions that increase lung density, such as pulmonary consolidation, will enhance the transmission of voice sounds, such as bronchophony (see Table 18-7).

A patient states that the pain medication is "not working" and rates his postoperative pain at a 10 on a 1-to-10 scale. Which of these assessment findings indicates an acute pain response to poorly controlled pain? a. Confusion b. Hyperventilation c. Increased blood pressure and pulse d. Depression

C Responses to poorly controlled acute pain include tachycardia, elevated blood pressure, and hypoventilation. Confusion and depression are associated with poorly controlled chronic pain (see Table 10-1).

The nurse is examining a patient's lower leg and notices a draining ulceration. Which of these actions is most appropriate in this situation? a. Washing hands, and contacting the physician b. Continuing to examine the ulceration, and then washing hands c. Washing hands, putting on gloves, and continuing with the examination of the ulceration d. Washing hands, proceeding with rest of the physical examination, and then continuing with the examination of the leg ulceration

C The examiner should wear gloves when the potential contact with any body fluids is present. In this situation, the nurse should wash his or her hands, put on gloves, and continue examining the ulceration.

A patient, an 85-year-old woman, is complaining about the fact that the bones in her face have become more noticeable. What explanation should the nurse give her? a. Diets low in protein and high in carbohydrates may cause enhanced facial bones. b. Bones can become more noticeable if the person does not use a dermatologically approved moisturizer. c. More noticeable facial bones are probably due to a combination of factors related to aging, such as decreased elasticity, subcutaneous fat, and moisture in her skin. d. Facial skin becomes more elastic with age. This increased elasticity causes the skin to be more taught, drawing attention to the facial bones.

C The facial bones and orbits appear more prominent in the aging adult, and the facial skin sags, which is attributable to decreased elasticity, decreased subcutaneous fat, and decreased moisture in the skin.

The nurse is reviewing the function of the cranial nerves (CNs). Which CN is responsible for conducting nerve impulses to the brain from the organ of Corti? a. I b. III c. VIII d. XI

C The nerve impulses are conducted by the auditory portion of CN VIII to the brain.

In using the ophthalmoscope to assess a patient's eyes, the nurse notices a red glow in the patient's pupils. On the basis of this finding, the nurse would: a. Suspect that an opacity is present in the lens or cornea. b. Check the light source of the ophthalmoscope to verify that it is functioning. c. Consider the red glow a normal reflection of the ophthalmoscope light off the inner retina. d. Continue with the ophthalmoscopic examination, and refer the patient for further evaluation.

C The red glow filling the person's pupil is the red reflex and is a normal finding caused by the reflection of the ophthalmoscope light off the inner retina. The other responses are not correct.

A patient is especially worried about an area of skin on her feet that has turned white. The health care provider has told her that her condition is vitiligo. The nurse explains to her that vitiligo is: a. Caused by an excess of melanin pigment b. Caused by an excess of apocrine glands in her feet c. Caused by the complete absence of melanin pigment d. Related to impetigo and can be treated with an ointment

C Vitiligo is the complete absence of melanin pigment in patchy areas of white or light skin on the face, neck, hands, feet, body folds, and around orifices—otherwise, the depigmented skin is normal.

A 10 year old is at the clinic for "a sore throat that has lasted 6 days." Which of these findings would be consistent with an acute infection? a. Tonsils 1+/1-4+ and pink; the same color as the oral mucosa b. Tonsils 2+/1-4+ with small plugs of white debris c. Tonsils 3+/1-4+ with large white spots d. Tonsils 3+/1-4+ with pale coloring

C With an acute infection, tonsils are bright red and swollen and may have exudate or large white spots. Tonsils are enlarged to 2+, 3+, or 4+ with an acute infection.

The middle lobe of the right lung is best auscultated over the a. anterior chest. b. posterior chest. c. axilla. d. midclavicular line. e. scapula.

C - axilla.

The nurse assesses the nail base angle using the Schamroth technique. The normal expecte are _____ at the bases. a. flat b. convex c. concave d. bowed e. elevated

C - concave

Nodes located nearest the elbow are the a. supraclavicular. b. axillary. c. epitrochlear. d. popliteal. e. femoral.

C - epitrochlear. Nodes nearest the elbow are the epitrochlear, nearest the clavicle are the supraclavicular, ne the inguinal region are the femoral, and nearest the armpit are the axillary.

Inspection to determine color variations of the skin is best conducted: a. using an episcope. b. under fluorescent lighting. c. with illumination provided by daylight. d. using a Wood's lamp. e. using a pen light.

C - with illumination provided by daylight.

The foramen ovale should close by a. 24 weeks of gestation. b. the initiation of labor. c. within minutes of birth. d. 4 weeks of age. e. 12 months of age.

C - within minutes of birth.

During an otoscopic examination, the nurse notices an area of black and white dots on the tympanic membrane and the ear canal wall. What does this finding suggest? a. Malignancy b. Viral infection c. Blood in the middle ear d. Yeast or fungal infection

D A colony of black or white dots on the drum or canal wall suggests a yeast or fungal infection (otomycosis).

A patient comes into the emergency department after an accident at work. A machine blew dust into his eyes, and he was not wearing safety glasses. The nurse examines his corneas by shining a light from the side across the cornea. What findings would suggest that he has suffered a corneal abrasion? a. Smooth and clear corneas b. Opacity of the lens behind the cornea c. Bleeding from the areas across the cornea d. Shattered look to the light rays reflecting off the cornea

D A corneal abrasion causes irregular ridges in reflected light, which produce a shattered appearance to light rays. No opacities should be observed in the cornea. The other responses are not correct.

During percussion, the nurse knows that a dull percussion note elicited over a lung lobe most likely results from: a. Shallow breathing. b. Normal lung tissue. c. Decreased adipose tissue. d. Increased density of lung tissue.

D A dull percussion note indicates an abnormal density in the lungs, as with pneumonia, pleural effusion, atelectasis, or a tumor. Resonance is the expected finding in normal lung tissue.

The nurse is bathing an 80-year-old man and notices that his skin is wrinkled, thin, lax, and dry. This finding would be related to which factor in the older adult? a. Increased vascularity of the skin b. Increased numbers of sweat and sebaceous glands c. An increase in elastin and a decrease in subcutaneous fat d. An increased loss of elastin and a decrease in subcutaneous fat

D An accumulation of factors place the aging person at risk for skin disease and breakdown: the thinning of the skin, a decrease in vascularity and nutrients, the loss of protective cushioning of the subcutaneous layer, a lifetime of environmental trauma to skin, the social changes of aging, a increasingly sedentary lifestyle, and the chance of immobility.

A patient has a long history of chronic obstructive pulmonary disease (COPD). During the assessment, the nurse will most likely observe which of these? a. Unequal chest expansion b. Increased tactile fremitus c. Atrophied neck and trapezius muscles d. Anteroposterior-to-transverse diameter ratio of 1:1

D An anteroposterior-to-transverse diameter ratio of 1:1 or barrel chest is observed in individuals with COPD because of hyperinflation of the lungs. The ribs are more horizontal, and the chest appears as if held in continuous inspiration. Neck muscles are hypertrophied from aiding in forced respiration. Chest expansion may be decreased but symmetric. Decreased tactile fremitus occurs from decreased transmission of vibrations.

A pregnant woman is interested in breastfeeding her baby and asks several questions about the topic. Which information is appropriate for the nurse to share with her? a. Breastfeeding is best when also supplemented with bottle feedings. b. Babies who are breastfed often require supplemental vitamins. c. Breastfeeding is recommended for infants for the first 2 years of life. d. Breast milk provides the nutrients necessary for growth, as well as natural immunity.

D Breastfeeding is recommended for full-term infants for the first year of life because breast milk is ideally formulated to promote normal infant growth and development, as well as natural immunity. The other statements are not correct.

The nurse is examining a 2-year-old child and asks, "May I listen to your heart now?" Which critique of the nurse's technique is most accurate? a. Asking questions enhances the child's autonomy b. Asking the child for permission helps develop a sense of trust c. This question is an appropriate statement because children at this age like to have choices d. Children at this age like to say, "No." The examiner should not offer a choice when no choice is available

D Children at this age like to say, "No." Choices should not be offered when no choice is really available. If the child says, "No" and the nurse does it anyway, then the nurse loses trust. Autonomy is enhanced by offering a limited option, "Shall I listen to your heart next or your tummy?"

A woman is leaving on a trip to Hawaii and has come in for a checkup. During the examination the nurse learns that she has diabetes and takes oral hypoglycemic agents. The patient needs to be concerned about which possible effect of her medications? a. Increased possibility of bruising b. Skin sensitivity as a result of exposure to salt water c. Lack of availability of glucose-monitoring supplies d. Importance of sunscreen and avoiding direct sunlight

D Drugs that may increase sunlight sensitivity and give a burn response include sulfonamides, thiazide diuretics, oral hypoglycemic agents, and tetracycline

When examining a 16-year-old male teenager, the nurse should: a. Discuss health teaching with the parent because the teen is unlikely to be interested in promoting wellness. b. Ask his parent to stay in the room during the history and physical examination to answer any questions and to alleviate his anxiety. c. Talk to him the same manner as one would talk to a younger child because a teen's level of understanding may not match his or her speech. d. Provide feedback that his body is developing normally, and discuss the wide variation among teenagers on the rate of growth and development.

D During the examination, the adolescent needs feedback that his or her body is healthy and developing normally. The adolescent has a keen awareness of body image and often compares him or herself with peers. Apprise the adolescent of the wide variation among teenagers on the rate of growth and development.

A patient comes to the clinic and states that he has noticed that his skin is redder than normal. The nurse understands that this condition is due to hyperemia and knows that it can be caused by: a. Decreased amounts of bilirubin in the blood b. Excess blood in the underlying blood vessels c. Decreased perfusion to the surrounding tissues d. Excess blood in the dilated superficial capillaries

D Erythema is an intense redness of the skin caused by excess blood (hyperemia) in the dilated superficial capillaries.

The nurse is conducting a child safety class for new mothers. Which factor places young children at risk for ear infections? a. Family history b. Air conditioning c. Excessive cerumen d. Passive cigarette smoke

D Exposure to passive and gestational smoke is a risk factor for ear infections in infants and children.

A pregnant woman states that she is concerned about her gums because she has noticed they are swollen and have started bleeding. What would be an appropriate response by the nurse? a. "Your condition is probably due to a vitamin C deficiency." b. "I'm not sure what causes swollen and bleeding gums, but let me know if it's not better in a few weeks." c. "You need to make an appointment with your dentist as soon as possible to have this checked." d. "Swollen and bleeding gums can be caused by the change in hormonal balance in your system during pregnancy."

D Gum margins are red and swollen and easily bleed with gingivitis. A changing hormonal balance may cause this condition to occur in pregnancy and puberty.

During an interview, the patient states he has the sensation that "everything around him is spinning." The nurse recognizes that the portion of the ear responsible for this sensation is the: a. Cochlea. b. CN VIII. c. Organ of Corti. d. Labyrinth.

D If the labyrinth ever becomes inflamed, then it feeds the wrong information to the brain, creating a staggering gait and a strong, spinning, whirling sensation called vertigo.

In performing an examination of a 3-year-old child with a suspected ear infection, the nurse would: a. Omit the otoscopic examination if the child has a fever. b. Pull the ear up and back before inserting the speculum. c. Ask the mother to leave the room while examining the child. d. Perform the otoscopic examination at the end of the assessment.

D In addition to its place in the complete examination, eardrum assessment is mandatory for any infant or child requiring care for an illness or fever. For the infant or young child, the timing of the otoscopic examination is best toward the end of the complete examination.

During the aging process, the hair can look gray or white and begin to feel thin and fine. The nurse knows that this occurs because of a decrease in the number of functioning: a. Metrocytes. b. Fungacytes. c. Phagocytes. d. Melanocytes.

D In the aging hair matrix, the number of functioning melanocytes decreases; as a result, the hair looks gray or white and feels thin and fine. The other options are not correct.

The nurse is obtaining a health history on a 3-month-old infant. During the interview, the mother states, "I think she is getting her first tooth because she has started drooling a lot." The nurse's best response would be: a. "You're right, drooling is usually a sign of the first tooth." b. "It would be unusual for a 3 month old to be getting her first tooth." c. "This could be the sign of a problem with the salivary glands." d. "She is just starting to salivate and hasn't learned to swallow the saliva."

D In the infant, salivation starts at 3 months. The baby will drool for a few months before learning to swallow the saliva. This drooling does not herald the eruption of the first tooth, although many parents think it does.

The nurse is reviewing causes of increased intraocular pressure. Which of these factors determines intraocular pressure? a. Thickness or bulging of the lens b. Posterior chamber as it accommodates increased fluid c. Contraction of the ciliary body in response to the aqueous within the eye d. Amount of aqueous produced and resistance to its outflow at the angle of the anterior chamber

D Intraocular pressure is determined by a balance between the amount of aqueous produced and the resistance to its outflow at the angle of the anterior chamber. The other responses are incorrect.

A patient has had a "terrible itch" for several months that he has been continuously scratching. On examination, the nurse might expect to find: a. A keloid. b. A fissure. c. Keratosis. d. Lichenification.

D Lichenification results from prolonged, intense scratching that eventually thickens the skin and produces tightly packed sets of papules. A keloid is a hypertrophic scar. A fissure is a linear crack with abrupt edges, which extends into the dermis; it can be dry or moist. Keratoses are lesions that are raised, thickened areas of pigmentation that appear crusted, scaly, and warty.

A mother brings her 2-month-old daughter in for an examination and says, "My daughter rolled over against the wall, and now I have noticed that she has this spot that is soft on the top of her head. Is something terribly wrong?" The nurse's best response would be: a. "Perhaps that could be a result of your dietary intake during pregnancy." b. "Your baby may have craniosynostosis, a disease of the sutures of the brain." c. "That 'soft spot' may be an indication of cretinism or congenital hypothyroidism." d. "That 'soft spot' is normal, and actually allows for growth of the brain during the first year of your baby's life."

D Membrane-covered "soft spots" allow for growth of the brain during the first year of life. They gradually ossify; the triangular-shaped posterior fontanel is closed by 1 to 2 months, and the diamond-shaped anterior fontanel closes between 9 months and 2 years.

A patient complains that while studying for an examination he began to notice a severe headache in the frontotemporal area of his head that is throbbing and is somewhat relieved when he lies down. He tells the nurse that his mother also had these headaches. The nurse suspects that he may be suffering from: a. Hypertension. b. Cluster headaches. c. Tension headaches. d. Migraine headaches.

D Migraine headaches tend to be supraorbital, retroorbital, or frontotemporal with a throbbing quality. They are severe in quality and are relieved by lying down. Migraines are associated with a family history of migraine headaches.

The nurse hears bilateral loud, long, and low tones when percussing over the lungs of a 4-year-old child. The nurse should: a. Palpate over the area for increased pain and tenderness. b. Ask the child to take shallow breaths, and percuss over the area again. c. Immediately refer the child because of an increased amount of air in the lungs. d. Consider this finding as normal for a child this age, and proceed with the examination.

D Percussion notes that are loud in amplitude, low in pitch, of a booming quality, and long in duration are normal over a child's lung.

A musical squeaking noise heard on auscultation of the lungs is called a. stridor. b. rales. c. rhonchi. d. wheezing. e. friction rub.

D - wheezing.

Ms. R, age 74 years, has no known health problems or diseases. You are doing a preventive Which symptom is associated with an intrathoracic infection? a. Barrel chest b. Cor pulmonale c. Pectus excavatum d. Pectus carinatum e. Malodorous breath

E - Malodorous breath

When enlarged, which lymph nodes are most likely to be a sign of malignancy? a. Occipital b. Anterior cervical c. Posterior cervical d. Femoral e. Supraclavicular

E - Supraclavicular Supraclavicular nodal enlargement is of special concern because it suggests a malignancy e node of Hodgkin lymphoma.

As adults age, their ability to resist infection decreases because of lymphatic nodes becomin a. hematopoietic. b. mucoid. c. porous. d. profuse. e. fibrotic. ANS: E - fibrotic.

Older adults' lymph nodes diminish in both number and size and are replaced with more fibro

What causes barrel chest

a. birth trauma. b. high fever. c. excess adipose tissue. d. kidney disease. e. chronic asthma, emphysema, or cystic fib

A Dennie-Morgan fold is probably caused by a. birth trauma. b. high fever. c. excess adipose tissue. d. kidney disease. e. chronic rubbing.

: E - chronic rubbing

Initial signs and symptoms of Epstein-Barr virus mononucleosis most often include a. pharyngitis, fever, and malaise. b. bleeding gums and spontaneous nosebleeds. c. headache, visual disturbance, and rash. d. inguinal adenopathy and painful urination. e. discrete labial and gingival ulcers and high-grade fever.

A - pharyngitis, fever, and malaise.

What is the pulse pressure for a patient whose blood pressure is 158/96 mm Hg and whose pulse rate is 72 beats per minute?

62 The pulse pressure is the difference between the systolic and diastolic and reflects the stroke volume. The pulse rate is not necessary for pulse pressure calculations.

Breath odors may alert the examiner to certain underlying metabolic conditions. The odor of a. uremia. b. tuberculosis. c. hepatic dysfunction. d. diabetic ketoacidosis. e. intestinal obstruction.

: A - uremia.

Breath sounds normally auscultated over most of the lung fields are called a. vesicular. b. hyperresonance. c. bronchial. d. tubular. e. bronchovesicular.

: A - vesicular

Your older clinic patient is being seen today as a follow-up for a 2-day history of pneumonia. productive cough, shortness of breath, and lethargy and has been spending most of the day examination by a. percussing all lung fields. b. auscultating the lung bases. c. determining tactile fremitus. d. estimating diaphragmatic excursion. e. auscultating the apices.

: B - auscultating the lung bases.

REF: p. 282 The pregnant woman is expected to develop a. tachypnea and decreased tidal volume. b. deep breathing but not more frequent breathing. c. dyspnea and increased functional residual capacity. d. bradypnea and increased tidal volume. e. tachypnea and increased functional residual capacity.

: B - deep breathing but not more frequent breathing.

Tactile fremitus is best felt a. along the costal margin and xiphoid process. b. in the suprasternal notch along the clavicle. c. at the level of bifurcation of the bronchi. d. posterolaterally over the scapulas. e. in the midaxillary lines.

: C - at the level of bifurcation of the bronchi

The nurse assesses the patient's nails and finds transverse white bands that cover the nail e Additional physical examination of this patient should include a. palpation of the liver. b. auscultation for carotid bruits. c. inspection for jaundice. d. fist percussion of the kidney. e. chest percussion.

: C - inspection for jaundice.

Lymphoid tissue normally regresses to adult size by a. 2 years of age. b. 5 years of age. c. 10 years of age. d. puberty. e. the early 20s.

: D - puberty. The extent of lymphoid tissue is abundant in infants, increases in childhood, and regresses

Breath sounds normally heard over the trachea are called a. bronchovesicular. b. amphoric. c. crepitus. d. vesicular. e. bronchial.

: E - bronchial.

A student is late for his appointment and has rushed across campus to the health clinic. The nurse should: a. Allow 5 minutes for him to relax and rest before checking his vital signs. b. Check the blood pressure in both arms, expecting a difference in the readings because of his recent exercise. c. Immediately monitor his vital signs on his arrival at the clinic and then 5 minutes later, recording any differences. d. Check his blood pressure in the supine position, which will provide a more accurate reading and will allow him to relax at the same time.

A A comfortable, relaxed person yields a valid blood pressure. Many people are anxious at the beginning of an examination; the nurse should allow at least a 5-minute rest period before measuring blood pressure.

The nurse is percussing over the lungs of a patient with pneumonia. The nurse knows that percussion over an area of atelectasis in the lungs will reveal: a. Dullness. b. Tympany. c. Resonance. d. Hyperresonance.

A A dull percussion note signals an abnormal density in the lungs, as with pneumonia, pleural effusion, atelectasis, or a tumor.

A 75-year-old woman who has a history of diabetes and peripheral vascular disease has been trying to remove a corn on the bottom of her foot with a pair of scissors. The nurse will encourage her to stop trying to remove the corn with scissors because: a. The woman could be at increased risk for infection and lesions because of her chronic disease. b. With her diabetes, she has increased circulation to her foot, and it could cause severe bleeding. c. She is 75 years old and is unable to see; consequently, she places herself at greater risk for self-injury with the scissors. d. With her peripheral vascular disease, her range of motion is limited and she may not be able to reach the corn safely.

A A personal history of diabetes and peripheral vascular disease increases a person's risk for skin lesions in the feet or ankles. The patient needs to seek a professional for assistance with corn removal.

The nurse is performing a nutritional assessment on a 15-year-old girl who tells the nurse that she is "so fat." Assessment reveals that she is 5 feet 4 inches and weighs 110 pounds. The nurse's appropriate response would be: a. "How much do you think you should weigh?" b. "Don't worry about it; you're not that overweight." c. "The best thing for you would be to go on a diet." d. "I used to always think I was fat when I was your age."

A Adolescents' increased body awareness and self-consciousness may cause eating disorders such as anorexia nervosa or bulimia, conditions in which the real or perceived body image does not favorably compare with an ideal image. The nurse should not belittle the adolescent's feelings, provide unsolicited advice, or agree with her

During a nutritional assessment, why is it important for the nurse to ask a patient what medications he or she is taking? a. Certain drugs can affect the metabolism of nutrients. b. The nurse needs to assess the patient for allergic reactions. c. Medications need to be documented in the record for the physician's review. d. Medications can affect one's memory and ability to identify food eaten in the last 24 hours.

A Analgesics, antacids, anticonvulsants, antibiotics, diuretics, laxatives, antineoplastic drugs, steroids, and oral contraceptives are drugs that can interact with nutrients, impairing their digestion, absorption, metabolism, or use. The other responses are not correct.

A man is at the clinic for a physical examination. He states that he is "very anxious" about the physical examination. What steps can the nurse take to make him more comfortable? a. Appear unhurried and confident when examining him. b. Stay in the room when he undresses in case he needs assistance. c. Ask him to change into an examining gown and to take off his undergarments. d. Defer measuring vital signs until the end of the examination, which allows him time to become comfortable.

A Anxiety can be reduced by an examiner who is confident, self-assured, considerate, and unhurried. Familiar and relatively nonthreatening actions, such as measuring the person's vital signs, will gradually accustom the person to the examination.

An adult patient with a history of allergies comes to the clinic complaining of wheezing and difficulty in breathing when working in his yard. The assessment findings include tachypnea, the use of accessory neck muscles, prolonged expiration, intercostal retractions, decreased breath sounds, and expiratory wheezes. The nurse interprets that these assessment findings are consistent with: a. Asthma. b. Atelectasis. c. Lobar pneumonia. d. Heart failure.

A Asthma is allergic hypersensitivity to certain inhaled particles that produces inflammation and a reaction of bronchospasm, which increases airway resistance, especially during expiration. An increased respiratory rate, the use of accessory muscles, a retraction of the intercostal muscles, prolonged expiration, decreased breath sounds, and expiratory wheezing are all characteristics of asthma. (See Table 18-8 for descriptions of the other conditions.)

The nurse is providing nutrition information to the mother of a 1-year-old child. Which of these statements represents accurate information for this age group? a. Maintaining adequate fat and caloric intake is important for a child in this age group. b. The recommended dietary allowances for an infant are the same as for an adolescent. c. The baby's growth is minimal at this age; therefore, caloric requirements are decreased. d. The baby should be placed on skim milk to decrease the risk of coronary artery disease when he or she grows older.

A Because of rapid growth, especially of the brain, both infants and children younger than 2 years of age should not drink skim or low-fat milk or be placed on low-fat diets. Fats (calories and essential fatty acids) are required for proper growth and central nervous system development.

If a 29-year-old woman weighs 156 pounds, and the nurse determines her ideal body weight to be 120 pounds, then how would the nurse classify the woman's weight? a. Obese b. Mildly overweight c. Suffering from malnutrition d. Within appropriate range of ideal weight

A Obesity, as a result of caloric excess, refers to weight more than 20% above ideal body weight. For this patient, 20% of her ideal body weight would be 24 pounds, and greater than 20% of her body weight would be over 144 pounds. Therefore, having a weight of 156 pounds would be considered obese.

While performing an assessment of a 65-year-old man with a history of hypertension and coronary artery disease, the nurse notices the presence of bilateral pitting edema in the lower legs. The skin is puffy and tight but normal in color. No increased redness or tenderness is observed over his lower legs, and the peripheral pulses are equal and strong. In this situation, the nurse suspects that the likely cause of the edema is which condition? a. Heart failure b. Venous thrombosis c. Local inflammation d. Blockage of lymphatic drainage

A Bilateral edema or edema that is generalized over the entire body is caused by a central problem such as heart failure or kidney failure. Unilateral edema usually has a local or peripheral cause.

In teaching a patient how to determine total body fat at home, the nurse includes instructions to obtain measurements of: a. Height and weight. b. Frame size and weight. c. Waist and hip circumferences. d. Mid-upper arm circumference and arm span.

A Body mass index, calculated by using height and weight measurements, is a practical marker of optimal weight for height and an indicator of obesity. The other options are not correct.

During auscultation of the lungs, the nurse expects decreased breath sounds to be heard in which situation? a. When the bronchial tree is obstructed b. When adventitious sounds are present c. In conjunction with whispered pectoriloquy d. In conditions of consolidation, such as pneumonia

A Decreased or absent breath sounds occur when the bronchial tree is obstructed, as in emphysema, and when sound transmission is obstructed, as in pleurisy, pneumothorax, or pleural effusion.

During an assessment of a 20-year-old patient with a 3-day history of nausea and vomiting, the nurse notices dry mucosa and deep vertical fissures in the tongue. These findings are reflective of: a. Dehydration. b. Irritation by gastric juices. c. A normal oral assessment. d. Side effects from nausea medication.

A Dry mouth occurs with dehydration or fever. The tongue has deep vertical fissures.

The nurse is reviewing percussion techniques with a newly graduated nurse. Which technique, if used by the new nurse, indicates that more review is needed? a. Percussing once over each area b. Quickly lifting the striking finger after each stroke c. Striking with the fingertip, not the finger pad d. Using the wrist to make the strikes, not the arm

A For percussion, the nurse should percuss two times over each location. The striking finger should be quickly lifted because a resting finger damps off vibrations. The tip of the striking finger should make contact, not the pad of the finger. The wrist must be relaxed and is used to make the strikes, not the arm.

The nurse is providing care for a 68-year-old woman who is complaining of constipation. What concern exists regarding her nutritional status? a. Absorption of nutrients may be impaired. b. Constipation may represent a food allergy. c. The patient may need emergency surgery to correct the problem. d. Gastrointestinal problems will increase her caloric demand.

A Gastrointestinal symptoms such as vomiting, diarrhea, or constipation may interfere with nutrient intake or absorption. The other responses are not correct.

During a well-baby checkup, a mother is concerned because her 2-month-old infant cannot hold her head up when she is pulled to a sitting position. Which response by the nurse is appropriate? a. "Head control is usually achieved by 4 months of age." b. "You shouldn't be trying to pull your baby up like that until she is older." c. "Head control should be achieved by this time." d. "This inability indicates possible nerve damage to the neck muscles."

A Head control is achieved by 4 months when the baby can hold the head erect and steady when pulled to a vertical position. The other responses are not appropriate.

The nurse is assessing an 8-year-old child whose growth rate measures below the third percentile for a child his age. He appears significantly younger than his stated age and is chubby with infantile facial features. Which condition does this child have? a. Hypopituitary dwarfism b. Achondroplastic dwarfism c. Marfan syndrome d. Acromegaly

A Hypopituitary dwarfism is caused by a deficiency in growth hormone in childhood and results in a retardation of growth below the third percentile, delayed puberty, and other problems. The child's appearance fits this description. Achondroplastic dwarfism is a genetic disorder resulting in characteristic deformities; Marfan syndrome is an inherited connective tissue disorder characterized by a tall, thin stature and other features. Acromegaly is the result of excessive secretion of growth hormone in adulthood. (For more information, see Table 9-5, Abnormalities in Body Height and Proportion

When auscultating the blood pressure of a 25-year-old patient, the nurse notices the phase I Korotkoff sounds begin at 200 mm Hg. At 100 mm Hg, the Korotkoff sounds muffle. At 92 mm Hg, the Korotkoff sounds disappear. How should the nurse record this patient's blood pressure? a. 200/92 b. 200/100 c. 100/200/92 d. 200/100/92

A In adults, the last audible sound best indicates the diastolic pressure. When a variance is greater than 10 to 12 mm Hg between phases IV and V, both phases should be recorded along with the systolic reading (e.g., 142/98/80).

23. The nurse is performing a nutritional assessment on an 80-year-old patient. The nurse knows that physiologic changes can directly affect the nutritional status of the older adult and include: a. Slowed gastrointestinal motility. b. Hyperstimulation of the salivary glands. c. Increased sensitivity to spicy and aromatic foods. d. Decreased gastrointestinal absorption causing esophageal reflux.

A Normal physiologic changes in aging adults that affect nutritional status include slowed gastrointestinal motility, decreased gastrointestinal absorption, diminished olfactory and taste sensitivity, decreased saliva production, decreased visual acuity, and poor dentition.

When assessing tactile fremitus, the nurse recalls that it is normal to feel tactile fremitus most intensely over which location? a. Between the scapulae b. Third intercostal space, MCL c. Fifth intercostal space, midaxillary line (MAL) d. Over the lower lobes, posterior side

A Normally, fremitus is most prominent between the scapulae and around the sternum. These sites are where the major bronchi are closest to the chest wall. Fremitus normally decreases as one progresses down the chest because more tissue impedes sound transmission.

The nurse is assessing a patient with a history of intravenous drug abuse. In assessing his mouth, the nurse notices a dark red confluent macule on the hard palate. This could be an early sign of: a. Acquired immunodeficiency syndrome (AIDS). b. Measles. c. Leukemia. d. Carcinoma.

A Oral Kaposi's sarcoma is a bruiselike, dark red or violet, confluent macule that usually occurs on the hard palate. It may appear on the soft palate or gingival margin. Oral lesions may be among the earliest lesions to develop with AIDS.

A 75-year-old man with a history of hypertension was recently changed to a new antihypertensive drug. He reports feeling dizzy at times. How should the nurse evaluate his blood pressure? a. Blood pressure and pulse should be recorded in the supine, sitting, and standing positions. b. The patient should be directed to walk around the room and his blood pressure assessed after this activity. c. Blood pressure and pulse are assessed at the beginning and at the end of the examination. d. Blood pressure is taken on the right arm and then 5 minutes later on the left arm.

A Orthostatic vital signs should be taken when the person is hypertensive or is taking antihypertensive medications, when the person reports fainting or syncope, or when volume depletion is suspected. The blood pressure and pulse readings are recorded in the supine, sitting, and standing positions

A 31-year-old patient tells the nurse that he has noticed a progressive loss in his hearing. He says that it does seem to help when people speak louder or if he turns up the volume of a television or radio. The most likely cause of his hearing loss is: a. Otosclerosis. b. Presbycusis. c. Trauma to the bones. d. Frequent ear infections.

A Otosclerosis is a common cause of conductive hearing loss in young adults between the ages of 20 and 40 years. Presbycusis is a type of hearing loss that occurs with aging. Trauma and frequent ear infections are not a likely cause of his hearing loss.

Which of these techniques uses the sense of touch to assess texture, temperature, moisture, and swelling when the nurse is assessing a patient? a. Palpation b. Inspection c. Percussion d. Auscultation

A Palpation

The nurse will use which technique of assessment to determine the presence of crepitus, swelling, and pulsations? a. Palpation b. Inspection c. Percussion d. Auscultation

A Palpation applies the sense of touch to assess texture, temperature, moisture, organ location and size, as well as any swelling, vibration or pulsation, rigidity or spasticity, crepitation, presence of lumps or masses, and the presence of tenderness or pain.

The nurse is reviewing the principles of nociception. During which phase of nociception does the conscious awareness of a painful sensation occur? a. Perception b. Modulation c. Transduction d. Transmission

A Perception is the third phase of nociception and indicates the conscious awareness of a painful sensation. During this phase, the sensation is recognized by higher cortical structures and identified as pain.

When percussing over the liver of a patient, the nurse notices a dull sound. The nurse should: a. Consider this a normal finding. b. Palpate this area for an underlying mass. c. Reposition the hands, and attempt to percuss in this area again. d. Consider this finding as abnormal, and refer the patient for additional treatment.

A Percussion over relatively dense organs, such as the liver or spleen, will produce a dull sound. The other responses are not correct.

During an examination of a child, the nurse considers that physical growth is the best index of a child's: a. General health. b. Genetic makeup. c. Nutritional status. d. Activity and exercise patterns.

A Physical growth is the best index of a child's general health; recording the child's height and weight helps determine normal growth patterns.

19. Which technique is correct when the nurse is assessing the radial pulse of a patient? The pulse is counted for: a. 1 minute, if the rhythm is irregular. b. 15 seconds and then multiplied by 4, if the rhythm is regular. c. 2 full minutes to detect any variation in amplitude. d. 10 seconds and then multiplied by 6, if the patient has no history of cardiac abnormalities.

A Recent research suggests that the 30-second interval multiplied by 2 is the most accurate and efficient technique when heart rates are normal or rapid and when rhythms are regular. If the rhythm is irregular, then the pulse is counted for 1 full minute.

When performing the corneal light reflex assessment, the nurse notes that the light is reflected at 2 o'clock in each eye. The nurse should: a. Consider this a normal finding. b. Refer the individual for further evaluation. c. Document this finding as an asymmetric light reflex. d. Perform the confrontation test to validate the findings.

A Reflection of the light on the corneas should be in exactly the same spot on each eye, or symmetric. If asymmetry is noted, then the nurse should administer the cover test.

The nurse is assessing the vital signs of a 3-year-old patient who appears to have an irregular respiratory pattern. How should the nurse assess this child's respirations? a. Respirations should be counted for 1 full minute, noticing rate and rhythm. b. Child's pulse and respirations should be simultaneously checked for 30 seconds. c. Child's respirations should be checked for a minimum of 5 minutes to identify any variations in his or her respiratory pattern. d. Patient's respirations should be counted for 15 seconds and then multiplied by 4 to obtain the number of respirations per minute.

A Respirations are counted for 1 full minute if an abnormality is suspected. The other responses are not correct actions.

The nurse is observing the auscultation technique of another nurse. The correct method to use when progressing from one auscultatory site on the thorax to another is _______ comparison. a. Side-to-side b. Top-to-bottom c. Posterior-to-anterior d. Interspace-by-interspace

A Side-to-side comparison is most important when auscultating the chest. The nurse should listen to at least one full respiration in each location. The other techniques are not correct.

The nurse suspects that a patient has hyperthyroidism, and the laboratory data indicate that the patient's T4 and T3 hormone levels are elevated. Which of these findings would the nurse most likely find on examination? a. Tachycardia b. Constipation c. Rapid dyspnea d. Atrophied nodular thyroid gland

A T4 and T3 are thyroid hormones that stimulate the rate of cellular metabolism, resulting in tachycardia. With an enlarged thyroid gland as in hyperthyroidism, the nurse might expect to find diffuse enlargement (goiter) or a nodular lump but not an atrophied gland. Dyspnea and constipation are not findings associated with hyperthyroidism.

The nurse knows that one advantage of the tympanic membrane thermometer (TMT) is that: a. Rapid measurement is useful for uncooperative younger children. b. Using the TMT is the most accurate method for measuring body temperature in newborn infants. c. Measuring temperature using the TMT is inexpensive. d. Studies strongly support the use of the TMT in children under the age 6 years.

A The TMT is useful for young children who may not cooperate for oral temperatures and fear rectal temperatures. However, the use a TMT with newborn infants and young children is conflicting.

The nurse is performing an external eye examination. Which statement regarding the outer layer of the eye is true? a. The outer layer of the eye is very sensitive to touch. b. The outer layer of the eye is darkly pigmented to prevent light from reflecting internally. c. The trigeminal nerve (CN V) and the trochlear nerve (CN IV) are stimulated when the outer surface of the eye is stimulated. d. The visual receptive layer of the eye in which light waves are changed into nerve impulses is located in the outer layer of the eye.

A The cornea and the sclera make up the outer layer of the eye. The cornea is very sensitive to touch. The middle layer, the choroid, has dark pigmentation to prevent light from reflecting internally. The trigeminal nerve (CN V) and the facial nerve (CN VII) are stimulated when the outer surface of the eye is stimulated. The retina, in the inner layer of the eye, is where light waves are changed into nerve impulses.

The nurse is preparing to use a stethoscope for auscultation. Which statement is true regarding the diaphragm of the stethoscope? The diaphragm: a. Is used to listen for high-pitched sounds. b. Is used to listen for low-pitched sounds. c. Should be lightly held against the person's skin to block out low-pitched sounds. d. Should be lightly held against the person's skin to listen for extra heart sounds and murmurs.

A The diaphragm of the stethoscope is best for listening to high-pitched sounds such as breath, bowel, and normal heart sounds. It should be firmly held against the person's skin, firmly enough to leave a ring. The bell of the stethoscope is best for soft, low-pitched sounds such as extra heart sounds or murmurs.

The nurse is examining a patient who tells the nurse, "I sure sweat a lot, especially on my face and feet but it doesn't have an odor." The nurse knows that this condition could be related to: a. Eccrine glands. b. Apocrine glands. c. Disorder of the stratum corneum. d. Disorder of the stratum germinativum.

A The eccrine glands are coiled tubules that directly open onto the skin surface and produce a dilute saline solution called sweat. Apocrine glands are primarily located in the axillae, anogenital area, nipples, and naval area and mix with bacterial flora to produce the characteristic musky body odor. The patient's statement is not related to disorders of the stratum corneum or the stratum germinativum.

The nurse needs to pull the portion of the ear that consists of movable cartilage and skin down and back when administering eardrops. This portion of the ear is called the: a. Auricle. b. Concha. c. Outer meatus. d. Mastoid process.

A The external ear is called the auricle or pinna and consists of movable cartilage and skin.

The nurse is performing a general survey. Which action is a component of the general survey? a. Observing the patient's body stature and nutritional status b. Interpreting the subjective information the patient has reported c. Measuring the patient's temperature, pulse, respirations, and blood pressure d. Observing specific body systems while performing the physical assessment

A The general survey is a study of the whole person that includes observing the patient's physical appearance, body structure, mobility, and behavior.

When assessing a patient's lungs, the nurse recalls that the left lung: a. Consists of two lobes. b. Is divided by the horizontal fissure. c. Primarily consists of an upper lobe on the posterior chest. d. Is shorter than the right lung because of the underlying stomach.

A The left lung has two lobes, and the right lung has three lobes. The right lung is shorter than the left lung because of the underlying liver. The left lung is narrower than the right lung because the heart bulges to the left. The posterior chest is almost all lower lobes.

The primary muscles of respiration include the: a. Diaphragm and intercostals. b. Sternomastoids and scaleni. c. Trapezii and rectus abdominis. d. External obliques and pectoralis major.

A The major muscle of respiration is the diaphragm. The intercostal muscles lift the sternum and elevate the ribs during inspiration, increasing the anteroposterior diameter. Expiration is primarily passive. Forced inspiration involves the use of other muscles, such as the accessory neck muscles—sternomastoid, scaleni, and trapezii muscles. Forced expiration involves the abdominal muscles.

When examining a patient's CN function, the nurse remembers that the muscles in the neck that are innervated by CN XI are the: a. Sternomastoid and trapezius. b. Spinal accessory and omohyoid. c. Trapezius and sternomandibular. d. Sternomandibular and spinal accessory.

A The major neck muscles are the sternomastoid and the trapezius. They are innervated by CN XI, the spinal accessory.

The nurse is assessing a patient who may have hearing loss. Which of these statements is true concerning air conduction? a. Air conduction is the normal pathway for hearing. b. Vibrations of the bones in the skull cause air conduction. c. Amplitude of sound determines the pitch that is heard. d. Loss of air conduction is called a conductive hearing loss.

A The normal pathway of hearing is air conduction, which starts when sound waves produce vibrations on the tympanic membrane. Conductive hearing loss results from a mechanical dysfunction of the external or middle ear. The other statements are not true concerning air conduction.

The nurse is examining a patient's retina with an ophthalmoscope. Which finding is considered normal? a. Optic disc that is a yellow-orange color b. Optic disc margins that are blurred around the edges c. Presence of pigmented crescents in the macular area d. Presence of the macula located on the nasal side of the retina

A The optic disc is located on the nasal side of the retina. Its color is a creamy yellow-orange to a pink, and the edges are distinct and sharply demarcated, not blurred. A pigmented crescent is black and is due to the accumulation of pigment in the choroid.

When examining the eye, the nurse notices that the patient's eyelid margins approximate completely. The nurse recognizes that this assessment finding: a. Is expected. b. May indicate a problem with extraocular muscles. c. May result in problems with tearing. d. Indicates increased intraocular pressure.

A The palpebral fissure is the elliptical open space between the eyelids, and, when closed, the lid margins approximate completely, which is a normal finding.

During chest assessment, you note the patient's voice quality while auscultating the lung fie is a nasal quality to the voice, and the e's sound like a's. This is indicative of a. lung consolidation. b. emphysema. c. bronchial obstruction. d. pneumothorax. e. asthma.

A - lung consolidation.

Lymph flows faster in response to a. massage. b. decreased blood volume. c. decreased metabolic rate. d. decreased permeability of the capillary walls. e. decreased capillary pressure.

A - massage. Lymph flow increases with mounting capillary pressure, greater permeability of the capillary w massage.

When preparing to perform a physical examination on an infant, the nurse should: a. Have the parent remove all clothing except the diaper on a boy. b. Instruct the parent to feed the infant immediately before the examination. c. Encourage the infant to suck on a pacifier during the abdominal examination. d. Ask the parent to leave the room briefly when assessing the infant's vital signs.

A The parent should always be present to increase the child's feeling of security and to understand normal growth and development. The timing of the examination should be 1 to 2 hours after feeding when the baby is neither too drowsy nor too hungry. Infants do not object to being nude; clothing should be removed, but a diaper should be left on a boy.

When assessing a patient's pulse, the nurse should also notice which of these characteristics? a. Force b. Pallor c. Capillary refill time d. Timing in the cardiac cycle

A The pulse is assessed for rate, rhythm, and force.

Which of these statements is true regarding the vertebra prominens? The vertebra prominens is: a. The spinous process of C7. b. Usually nonpalpable in most individuals. c. Opposite the interior border of the scapula. d. Located next to the manubrium of the sternum.

A The spinous process of C7 is the vertebra prominens and is the most prominent bony spur protruding at the base of the neck. Counting ribs and intercostal spaces on the posterior thorax is difficult because of the muscles and soft tissue. The vertebra prominens is easier to identify and is used as a starting point in counting thoracic processes and identifying landmarks on the posterior chest.

A patient's laboratory data reveal an elevated thyroxine (T4) level. The nurse would proceed with an examination of the _____ gland. a. Thyroid b. Parotid c. Adrenal d. Parathyroid

A The thyroid gland is a highly vascular endocrine gland that secretes T4 and triiodothyronine (T3). The other glands do not secrete T4.

A 45-year-old farmer comes in for a skin evaluation and complains of hair loss on his head. His hair seems to be breaking off in patches, and he notices some scaling on his head. The nurse begins the examination suspecting: a. Tinea capitis. b. Folliculitis. c. Toxic alopecia. d. Seborrheic dermatitis.

A Tinea capitis is rounded patchy hair loss on the scalp, leaving broken-off hairs, pustules, and scales on the skin, and is caused by a fungal infection. Lesions are fluorescent under a Wood light and are usually observed in children and farmers; tinea capitis is highly contagious. (See Table 12-12, Abnormal Conditions of Hair, for descriptions of the other terms.)

During an examination, the nurse knows that the best way to palpate the lymph nodes in the neck is described by which statement? a. Using gentle pressure, palpate with both hands to compare the two sides. b. Using strong pressure, palpate with both hands to compare the two sides. c. Gently pinch each node between one's thumb and forefinger, and then move down the neck muscle. d. Using the index and middle fingers, gently palpate by applying pressure in a rotating pattern.

A Using gentle pressure is recommended because strong pressure can push the nodes into the neck muscles. Palpating with both hands to compare the two sides symmetrically is usually most efficient.

The nurse recognizes which of these persons is at greatest risk for undernutrition? a. 5-month-old infant b. 50-year-old woman c. 20-year-old college student d. 30-year-old hospital administrator

A Vulnerable groups for undernutrition are infants, children, pregnant women, recent immigrants, persons with low incomes, hospitalized people, and aging adults.

The nurse is listening to the breath sounds of a patient with severe asthma. Air passing through narrowed bronchioles would produce which of these adventitious sounds? a. Wheezes b. Bronchial sounds c. Bronchophony d. Whispered pectoriloquy

A Wheezes are caused by air squeezed or compressed through passageways narrowed almost to closure by collapsing, swelling, secretions, or tumors, such as with acute asthma or chronic emphysema

A 6-month-old infant has been brought to the well-child clinic for a check-up. She is currently sleeping. What should the nurse do first when beginning the examination? a. Auscultate the lungs and heart while the infant is still sleeping. b. Examine the infant's hips, because this procedure is uncomfortable. c. Begin with the assessment of the eye, and continue with the remainder of the examination in a head-to-toe approach. d. Wake the infant before beginning any portion of the examination to obtain the most accurate assessment of body systems.

A When the infant is quiet or sleeping is an ideal time to assess the cardiac, respiratory, and abdominal systems. Assessment of the eye, ear, nose, and throat are invasive procedures that should be performed at the end of the examination.

The nurse assesses the hearing of a 7-month-old by clapping hands. What is the expected response? The infant: a. Turns his or her head to localize the sound. b. Shows no obvious response to the noise. c. Shows a startle and acoustic blink reflex. d. Stops any movement, and appears to listen for the sound.

A With a loud sudden noise, the nurse should notice the infant turning his or her head to localize the sound and to respond to his or her own name. A startle reflex and acoustic blink reflex is expected in newborns; at age 3 to 4 months, the infant stops any movement and appears to listen.

The nurse is preparing to measure the vital signs of a 6-month-old infant. Which action by the nurse is correct? a. Respirations are measured; then pulse and temperature. b. Vital signs should be measured more frequently than in an adult. c. Procedures are explained to the parent, and the infant is encouraged to handle the equipment. d. The nurse should first perform the physical examination to allow the infant to become more familiar with her and then measure the infant's vital signs.

A With an infant, the order of vital sign measurements is reversed to respiration, pulse, and temperature. Taking the temperature first, especially if it is rectal, may cause the infant to cry, which will increase the respiratory and pulse rate, thus masking the normal resting values. The vital signs are measured with the same purpose and frequency as would be measured in an adult.

The nurse is performing an assessment on a 21-year-old patient and notices that his nasal mucosa appears pale, gray, and swollen. What would be the most appropriate question to ask the patient? a. "Are you aware of having any allergies?" b. "Do you have an elevated temperature?" c. "Have you had any symptoms of a cold?" d. "Have you been having frequent nosebleeds?"

A With chronic allergies, the mucosa looks swollen, boggy, pale, and gray. Elevated body temperature, colds, and nosebleeds do not cause these mucosal changes.

During an examination, the nurse notices that the patient stumbles a little while walking, and, when she sits down, she holds on to the sides of the chair. The patient states, "It feels like the room is spinning!" The nurse notices that the patient is experiencing: a. Objective vertigo. b. Subjective vertigo. c. Tinnitus. d. Dizziness.

A With objective vertigo, the patient feels like the room spins; with subjective vertigo, the person feels like he or she is spinning. Tinnitus is a sound that comes from within a person; it can be a ringing, crackling, or buzzing sound. It accompanies some hearing or ear disorders. Dizziness is not the same as true vertigo; the person who is dizzy may feel unsteady and lightheaded.

When examining children affected with Down syndrome (trisomy 21), the nurse looks for the possible presence of: a. Ear dysplasia. b. Long, thin neck. c. Protruding thin tongue. d. Narrow and raised nasal bridge.

A With the chromosomal aberration trisomy 21, also known as Down syndrome, head and face characteristics may include upslanting eyes with inner epicanthal folds, a flat nasal bridge, a small broad flat nose, a protruding thick tongue, ear dysplasia, a short broad neck with webbing, and small hands with a single palmar crease.

Which of the following findings indicates that the examiner is assessing a blood vessel rathe a. A bruit b. Size less than 1 cm c. Tenderness d. Redness e. Warmth

A - A bruit

Which of the following is an ABCD characteristic of malignant melanoma? a. Asymmetric borders b. Borders well demarcated c. Color of lesion is uniform d. Diameter less than 6 mm e. Severe blistering or sunburns as a child

A - Asymmetric borders The ABCD melanoma mnemonic includes asymmetry, borders that are irregular, color that is greater than 6 mm and growing. Severe blistering or sunburns as a child is a risk factor

Which of the following organs does not have lymphatic vessels? a. Brain b. Kidneys c. Liver d. Lungs e. Appendix

A - Brain Lymphatic tissues are found abundantly throughout the body except in two places, the place system).

Cherry angiomas are a common finding in a. adults older than 30 years. b. newborns. c. pregnant women. d. sunbathers. e. adolescents.

A - Cherry angiomas occur in virtually everyone older than 30 years and increase numerically

Respiratory effort usually exhibited by the patient with cerebral brain damage is called a. Cheyne-Stokes respiration. b. paroxysmal nocturnal dyspnea. c. Kussmaul breathing. d. Biot respiration. e. ataxic respiration.

A - Cheyne-Stokes respiration

A single transverse crease seen on the palm of a small child may imply a. Down syndrome. b. Turner syndrome. c. systemic sclerosis. d. profound dehydration. e. neurofibromatosis.

A - Down syndrome

When examining the epitrochlear nodes, which joint should be supported? a. Elbow b. Knee c. Shoulder d. Wrist e. First metacarpophalangeal

A - Elbow

Which of the following, if reported, belongs in the family history of a patient with a skin rash? a. Father has chronic asthma. b. Sister repeatedly uses a tanning booth. c. Mother trims patient's nails too short. d. Sister had measles as a child. e. Grandparent had chemical burn.

A - Father has chronic asthma

The anteroposterior diameter of the chest is normally approximately the same as the transve a. Infants b. School-age children c. Adolescents d. Young adults e. Older adults

A - Infants

Soft, painless, bluish papules in persons who are HIV-positive are most likely a. Kaposi sarcoma (KS). b. malignant melanoma. c. molluscum contagiosum. d. pityriasis rosea. e. herpes zoster.

A - Kaposi sarcoma (KS). KS is the more common malignant skin lesion of HIV-infected persons. The lesions are soft, papules.

Which condition requires immediate emergency intervention? a. Patient with pleuritic pain without dyspnea b. Patient with fever and a productive cough c. Patient with tachypnea but no chest retractions d. Patient with pleuritic pain and rib tenderness e. Patient with absent breath sounds and dull percussion tones

A - Patient with pleuritic pain without dyspnea

A patient describes shortness of breath that gets worse when he sits up. Which term docum a. Platypnea b. Orthopnea c. Tachypnea d. Bradypnea e. Hypopnea

A - Platypnea

The examination of the upper left quadrant of the abdominal cavity is essential to the evalua the location of which organ? a. Spleen b. Liver c. Thymus d. Pancreas e. Stomach

A - Spleen The spleen is the largest of the lymphatic organs. It is located in the upper left portion of the

The most common cutaneous neoplasm is a. basal cell carcinoma. b. compound nevus. c. seborrheic keratosis. d. senile actinic keratosis. e. malignant melanoma.

A - basal cell carcinoma. Basal cell carcinoma is the most common form of skin cancer.

To distinguish crackles from rhonchi, you should auscultate the lungs a. before and after the patient coughs. b. first at the lung base and then at the apex. c. with the patient inhaling and then exhaling. d. with the patient prone and then supine. e. with the patient recumbent and then sitting.

A - before and after the patient coughs.

The most appropriate lighting source to highlight chest movement is (are) a. bright tangential lighting. b. daylight from a window. c. flashlight in a dark room. d. fluorescent ceiling lights. e. a Wood's lamp.

A - bright tangential lighting.

In the most effective percussion technique of the posterior lung fields, the patient cooperates a. folding the arms in front. b. bending the head back. c. standing and bending forward. d. lying on the side and extending the top arm. e. lying prone.

A - folding the arms in front.

To distinguish between a respiratory friction rub and a cardiac friction rub, ask the patient to a. hold his or her breath. b. lean forward. c. say "99" while you palpate the anterior chest. d. identify the location of his or her pain. e. arch backward.

A - hold his or her breath.

Newborns have a poorly developed subcutaneous fat layer and therefore have a reduced abil insulated from the environment. The eccrine sweat glands do not begin to function until after Normal hormone-related changes of adolescence include a. increased oil production. b. the development of fine silky lanugo hair. c. depletion of apocrine glands. d. decreased sebaceous gland activity. e. slowed hair growth.

A - increased oil production.

A 34-year-old man is being seen for complaints of dull pain between the shoulder blades that and coughing. Upon auscultation of the chest, you suspect that you will hear a. rhonchi. b. expiratory wheeze. c. crackles. d. pleural friction rub. e. crepitus.

A - rhonchi.

You are documenting a rash between the eighth and ninth ribs on the lateral border. This inte terms of the a. rib immediately above it. b. rib immediately below it. c. number of centimeters it is positioned below the clavicle. d. number of inches it is positioned below the clavicle. e. relationship to the sternum.

A - rib immediately above it.

A 29-year-old patient presents with a new complaint of productive cough with purulent sputum quadrant abdominal pain. You suspect pneumonia in the _____ lobe. a. right lower b. right middle c. right upper d. left upper e. left lower

A - right lower

The characteristic barrel chest of an older adult is caused by a combination of factors, includ a. skeletal changes of aging. b. increased muscular expansion of the chest wall. c. less fibrous alveoli. d. increased vital capacity. e. increased lung resiliency

A - skeletal changes of aging

The nurse is assessing voice sounds during a respiratory assessment. Which of these findings indicates a normal assessment? Select all that apply. a. Voice sounds are faint, muffled, and almost inaudible when the patient whispers "one, two, three" in a very soft voice. b. As the patient repeatedly says "ninety-nine," the examiner clearly hears the words "ninety-nine." c. When the patient speaks in a normal voice, the examiner can hear a sound but cannot exactly distinguish what is being said. d. As the patient says a long "ee-ee-ee" sound, the examiner also hears a long "ee-ee-ee" sound. e. As the patient says a long "ee-ee-ee" sound, the examiner hears a long "aaaaaa" sound.

A, C, D As a patient repeatedly says "ninety-nine," normally the examiner hears voice sounds but cannot distinguish what is being said. If a clear "ninety-nine" is auscultated, then it could indicate increased lung density, which enhances the transmission of voice sounds, which is a measure of bronchophony. When a patient says a long "ee-ee-ee" sound, normally the examiner also hears a long "ee-ee-ee" sound through auscultation, which is a measure of egophony. If the examiner hears a long "aaaaaa" sound instead, this sound could indicate areas of consolidation or compression. With whispered pectoriloquy, as when a patient whispers a phrase such as "one-two-three," the normal response when auscultating voice sounds is to hear sounds that are faint, muffled, and almost inaudible. If the examiner clearly hears the whispered voice, as if the patient is speaking through the stethoscope, then consolidation of the lung fields may exist.

The nurse is preparing to perform a physical assessment. Which statement is true about the physical assessment? The inspection phase: a. Usually yields little information. b. Takes time and reveals a surprising amount of information. c. May be somewhat uncomfortable for the expert practitioner. d. Requires a quick glance at the patient's body systems before proceeding with palpation.

B A focused inspection takes time and yields a surprising amount of information. Initially, the examiner may feel uncomfortable, staring at the person without also doing something. A focused assessment is significantly more than a "quick glance."

A patient comes into the clinic complaining of pain in her right eye. On examination, the nurse sees a pustule at the lid margin that is painful to touch, red, and swollen. The nurse recognizes that this is a: a. Chalazion. b. Hordeolum (stye). c. Dacryocystitis. d. Blepharitis.

B A hordeolum, or stye, is a painful, red, and swollen pustule at the lid margin. A chalazion is a nodule protruding on the lid, toward the inside, and is nontender, firm, with discrete swelling. Dacryocystitis is an inflammation of the lacrimal sac. Blepharitis is inflammation of the eyelids (see Table 14-3

While performing the otoscopic examination of a 3-year-old boy who has been pulling on his left ear, the nurse finds that his left tympanic membrane is bright red and that the light reflex is not visible. The nurse interprets these findings to indicate a(n): a. Fungal infection. b. Acute otitis media. c. Perforation of the eardrum. d. Cholesteatoma.

B Absent or distorted light reflex and a bright red color of the eardrum are indicative of acute otitis media. (See Table 15-5 for descriptions of the other conditions.)

A patient's weekly blood pressure readings for 2 months have ranged between 124/84 mm Hg and 136/88 mm Hg, with an average reading of 126/86 mm Hg. The nurse knows that this blood pressure falls within which blood pressure category? a. Normal blood pressure b. Prehypertension c. Stage 1 hypertension d. Stage 2 hypertension

B According to the Seventh Report of the Joint National Committee (JNC 7) guidelines, prehypertension blood pressure readings are systolic readings of 120 to 139 mm Hg or diastolic readings of 50 to 89 mm Hg.

A mother and her 13-year-old daughter express their concern related to the daughter's recent weight gain and her increase in appetite. Which of these statements represents information the nurse should discuss with them? a. Dieting and exercising are necessary at this age. b. Snacks should be high in protein, iron, and calcium. c. Teenagers who have a weight problem should not be allowed to snack. d. A low-calorie diet is important to prevent the accumulation of fat.

B After a period of slow growth in late childhood, adolescence is characterized by rapid physical growth and endocrine and hormonal changes. Caloric and protein requirements increase to meet this demand. Because of bone growth and increasing muscle mass (and, in girls, the onset of menarche), calcium and iron requirements also increase.

The nurse is conducting a health fair for older adults. Which statement is true regarding vital sign measurements in aging adults? a. The pulse is more difficult to palpate because of the stiffness of the blood vessels. b. An increased respiratory rate and a shallower inspiratory phase are expected findings. c. A decreased pulse pressure occurs from changes in the systolic and diastolic blood pressures. d. Changes in the body's temperature regulatory mechanism leave the older person more likely to develop a fever.

B Aging causes a decrease in vital capacity and decreased inspiratory reserve volume. The examiner may notice a shallower inspiratory phase and an increased respiratory rate. An increase in the rigidity of the arterial walls makes the pulse actually easier to palpate. Pulse pressure is widened in older adults, and changes in the body temperature regulatory mechanism leave the older person less likely to have fever but at a greater risk for hypothermia.

Which of these assessment findings would the nurse expect to see when examining the eyes of a black patient? a. Increased night vision b. Dark retinal background c. Increased photosensitivity d. Narrowed palpebral fissures

B An ethnically based variability in the color of the iris and in retinal pigmentation exists, with darker irides having darker retinas behind them.

When assessing the intensity of a patient's pain, which question by the nurse is appropriate? a. "What makes your pain better or worse?" b. "How much pain do you have now?" c. "How does pain limit your activities?" d. "What does your pain feel like?"

B Asking the patient "how much pain do you have?" is an assessment of the intensity of a patient's pain; various intensity scales can be used. Asking what makes one's pain better or worse assesses alleviating or aggravating factors. Asking whether pain limits one's activities assesses the degree of impairment and quality of life. Asking "what does your pain feel like" assesses the quality of pain.

A 52-year-old woman has a papule on her nose that has rounded, pearly borders and a central red ulcer. She said she first noticed it several months ago and that it has slowly grown larger. The nurse suspects which condition? a. Acne b. Basal cell carcinoma c. Melanoma d. Squamous cell carcinoma

B Basal cell carcinoma usually starts as a skin-colored papule that develops rounded, pearly borders with a central red ulcer. It is the most common form of skin cancer and grows slowly. This description does not fit acne lesions. (See Table 12-11 for descriptions of melanoma and squamous cell carcinoma.)

During an oral examination of a 4-year-old Native-American child, the nurse notices that her uvula is partially split. Which of these statements is accurate? a. This condition is a cleft palate and is common in Native Americans. b. A bifid uvula may occur in some Native-American groups. c. This condition is due to an injury and should be reported to the authorities. d. A bifid uvula is palatinus, which frequently occurs in Native Americans.

B Bifid uvula, a condition in which the uvula is split either completely or partially, occurs in some Native-American groups

The nurse is assessing the body weight as a percentage of ideal body weight on an adolescent patient who was admitted for suspected anorexia nervosa. The patient's usual weight was 125 pounds, but today she weighs 98 pounds. The nurse calculates the patient's ideal body weight and concludes that the patient is: a. Experiencing mild malnutrition. b. Experiencing moderate malnutrition. c. Experiencing severe malnutrition. d. Still within expected parameters with her current weight.

B By dividing her current weight by her usual weight and then multiplying by 100, a percentage of 78.4% is obtained, which means that her current weight is 78.4% of her ideal body weight. A current weight of 80% to 90% of ideal weight suggests mild malnutrition; a current weight of 70% to 80% of ideal weight suggests moderate malnutrition; a current weight of less than 70% of ideal weight suggests severe malnutrition.

During a skin assessment, the nurse notices that a Mexican-American patient has skin that is yellowish-brown; however, the skin on the hard and soft palate is pink and the patient's scleras are not yellow. From this finding, the nurse could probably rule out: a. Pallor b. Jaundice c. Cyanosis d. Iron deficiency

B Jaundice is exhibited by a yellow color, which indicates rising levels of bilirubin in the blood. Jaundice is first noticed in the junction of the hard and soft palate in the mouth and in the scleras.

A 40-year-old patient who has just finished chemotherapy for breast cancer tells the nurse that she is concerned about her mouth. During the assessment the nurse finds areas of buccal mucosa that are raw and red with some bleeding, as well as other areas that have a white, cheesy coating. The nurse recognizes that this abnormality is: a. Aphthous ulcers. b. Candidiasis. c. Leukoplakia. d. Koplik spots.

B Candidiasis is a white, cheesy, curdlike patch on the buccal mucosa and tongue. It scrapes off, leaving a raw, red surface that easily bleeds. It also occurs after the use of antibiotics or corticosteroids and in persons who are immunosuppressed. (See Table 16-4 for descriptions of the other lesions.)

The nurse is performing a middle ear assessment on a 15-year-old patient who has had a history of chronic ear infections. When examining the right tympanic membrane, the nurse sees the presence of dense white patches. The tympanic membrane is otherwise unremarkable. It is pearly, with the light reflex at 5 o'clock and landmarks visible. The nurse should: a. Refer the patient for the possibility of a fungal infection. b. Know that these are scars caused from frequent ear infections. c. Consider that these findings may represent the presence of blood in the middle ear. d. Be concerned about the ability to hear because of this abnormality on the tympanic membrane.

B Dense white patches on the tympanic membrane are sequelae of repeated ear infections. They do not necessarily affect hearing.

In performing an assessment on a 49-year-old woman who has imbalanced nutrition as a result of dysphagia, which data would the nurse expect to find? a. Increase in hair growth b. Inadequate nutrient food intake c. Weight 10% to 20% over ideal d. Sore, inflamed buccal cavity

B Dysphagia, or impaired swallowing, interferes with adequate nutrient intake.

A patient is unable to differentiate between sharp and dull stimulation to both sides of her face. The nurse suspects: a. Bell palsy. b. Damage to the trigeminal nerve. c. Frostbite with resultant paresthesia to the cheeks. d. Scleroderma.

B Facial sensations of pain or touch are mediated by CN V, which is the trigeminal nerve. Bell palsy is associated with CN VII damage. Frostbite and scleroderma are not associated with this problem.

The nurse is assessing a 16-year-old patient who has suffered head injuries from a recent motor vehicle accident. Which of these statements indicates the most important reason for assessing for any drainage from the ear canal? a. If the drum has ruptured, then purulent drainage will result. b. Bloody or clear watery drainage can indicate a basal skull fracture. c. The auditory canal many be occluded from increased cerumen. d. Foreign bodies from the accident may cause occlusion of the canal.

B Frank blood or clear watery drainage (cerebrospinal leak) after a trauma suggests a basal skull fracture and warrants immediate referral. Purulent drainage indicates otitis externa or otitis media.

During a physical education class, a student is hit in the eye with the end of a baseball bat. When examined in the emergency department, the nurse notices the presence of blood in the anterior chamber of the eye. This finding indicates the presence of: a. Hypopyon. b. Hyphema. c. Corneal abrasion. d. Pterygium.

B Hyphema is the term for blood in the anterior chamber and is a serious result of blunt trauma (a fist or a baseball) or spontaneous hemorrhage and may indicate scleral rupture or major intraocular trauma. (See Table 14-7 for descriptions of the other terms.)

A patient in her first trimester of pregnancy is diagnosed with rubella. Which of these statements is correct regarding the significance of this in relation to the infant's hearing? a. Rubella may affect the mother's hearing but not the infant's. b. Rubella can damage the infant's organ of Corti, which will impair hearing. c. Rubella is only dangerous to the infant in the second trimester of pregnancy. d. Rubella can impair the development of CN VIII and thus affect hearing.

B If maternal rubella infection occurs during the first trimester, then it can damage the organ of Corti and impair hearing.

A mother has noticed that her son, who has been to a new babysitter, has some blisters and scabs on his face and buttocks. On examination, the nurse notices moist, thin-roofed vesicles with a thin erythematous base and suspects: a. Eczema. b. Impetigo. c. Herpes zoster. d. Diaper dermatitis.

B Impetigo is moist, thin-roofed vesicles with a thin erythematous base and is a contagious bacterial infection of the skin and most common in infants and children. Eczema is characterized by erythematous papules and vesicles with weeping, oozing, and crusts. Herpes zoster (i.e., chickenpox or varicella) is characterized by small, tight vesicles that are shiny with an erythematous base. Diaper dermatitis is characterized by red, moist maculopapular patches with poorly defined borders.

The nurse is assessing the vital signs of a 20-year-old male marathon runner and documents the following vital signs: temperature-36° C; pulse-48 beats per minute; respirations-14 breaths per minute; blood pressure-104/68 mm Hg. Which statement is true concerning these results? a. The patient is experiencing tachycardia. b. These are normal vital signs for a healthy, athletic adult. c. The patient's pulse rate is not normal—his physician should be notified. d. On the basis of these readings, the patient should return to the clinic in 1 week.

B In the adult, a heart rate less than 50 beats per minute is called bradycardia, which normally occurs in the well-trained athlete whose heart muscle develops along with the skeletal muscles.

When examining the mouth of an older patient, the nurse recognizes which finding is due to the aging process? a. Teeth appearing shorter b. Tongue that looks smoother in appearance c. Buccal mucosa that is beefy red in appearance d. Small, painless lump on the dorsum of the tongue

B In the aging adult, the tongue looks smoother because of papillary atrophy. The teeth are slightly yellowed and appear longer because of the recession of gingival margins.

The nurse will perform a palpated pressure before auscultating blood pressure. The reason for this is to: a. More clearly hear the Korotkoff sounds. b. Detect the presence of an auscultatory gap. c. Avoid missing a falsely elevated blood pressure. d. More readily identify phase IV of the Korotkoff sounds.

B Inflation of the cuff 20 to 30 mm Hg beyond the point at which a palpated pulse disappears will avoid missing an auscultatory gap, which is a period when the Korotkoff sounds disappear during auscultation.

The adipose tissue in the hypodermis serves to a. provide sensory input. b. generate heat and insulate. c. create tensile strength. d. restrict water loss. e. secrete collagen.

B - generate heat and insulate.

The nurse is assessing the skin of a patient who has acquired immunodeficiency syndrome (AIDS) and notices multiple patchlike lesions on the temple and beard area that are faint pink in color. The nurse recognizes these lesions as: a. Measles (rubeola). b. Kaposi's sarcoma. c. Angiomas. d. Herpes zoster.

B Kaposi's sarcoma is a vascular tumor that, in the early stages, appears as multiple, patchlike, faint pink lesions over the patient's temple and beard areas. Measles is characterized by a red-purple maculopapular blotchy rash that appears on the third or fourth day of illness. The rash is first observed behind the ears, spreads over the face, and then spreads over the neck, trunk, arms, and legs. Cherry (senile) angiomas are small (1 to 5 mm), smooth, slightly raised bright red dots that commonly appear on the trunk in all adults over 30 years old. Herpes zoster causes vesicles up to 1 cm in size that are elevated with a cavity containing clear fluid.

A 21-year-old woman has been on a low-protein liquid diet for the past 2 months. She has had adequate intake of calories and appears well nourished. After further assessment, what would the nurse expect to find? a. Poor skin turgor b. Decreased serum albumin c. Increased lymphocyte count d. Triceps skinfold less than standard

B Kwashiorkor (protein malnutrition) is due to diets that may be high in calories but contain little or no protein (e.g., low-protein liquid diets, fad diets, and long-term use of dextrose-containing intravenous fluids). The serum albumin would be less than 3.5 g/dL.

A few days after a summer hiking trip, a 25-year-old man comes to the clinic with a rash. On examination, the nurse notes that the rash is red, macular, with a bull's eye pattern across his midriff and behind his knees. The nurse suspects: a. Rubeola. b. Lyme disease. c. Allergy to mosquito bites. d. Rocky Mountain spotted fever.

B Lyme disease occurs in people who spend time outdoors in May through September. The first disease state exhibits the distinctive bull's eye and a red macular or papular rash that radiates from the site of the tick bite with some central clearing. The rash spreads 5 cm or larger, and is usually in the axilla, midriff, inguinal, or behind the knee, with regional lymphadenopathy.

The nurse has just completed a lymph node assessment on a 60-year-old healthy female patient. The nurse knows that most lymph nodes in healthy adults are normally: a. Shotty. b. Nonpalpable. c. Large, firm, and fixed to the tissue. d. Rubbery, discrete, and mobile.

B Most lymph nodes are nonpalpable in adults. The palpability of lymph nodes decreases with age. Normal nodes feel movable, discrete, soft, and nontender.

A woman who is in the second trimester of pregnancy mentions that she has had "more nosebleeds than ever" since she became pregnant. The nurse recognizes that this is a result of: a. A problem with the patient's coagulation system. b. Increased vascularity in the upper respiratory tract as a result of the pregnancy. c. Increased susceptibility to colds and nasal irritation. d. Inappropriate use of nasal sprays.

B Nasal stuffiness and epistaxis may occur during pregnancy as a result of increased vascularity in the upper respiratory tract.

A 70-year-old man has a blood pressure of 150/90 mm Hg in a lying position, 130/80 mm Hg in a sitting position, and 100/60 mm Hg in a standing position. How should the nurse evaluate these findings? a. These readings are a normal response and attributable to changes in the patient's position. b. The change in blood pressure readings is called orthostatic hypotension. c. The blood pressure reading in the lying position is within normal limits. d. The change in blood pressure readings is considered within normal limits for the patient's age.

B Orthostatic hypotension is a drop in systolic pressure of more than 20 mm Hg, which occurs with a quick change to a standing position. Aging people have the greatest risk of this problem.

A patient says that she has recently noticed a lump in the front of her neck below her "Adam's apple" that seems to be getting bigger. During the assessment, the finding that leads the nurse to suspect that this may not be a cancerous thyroid nodule is that the lump (nodule): a. Is tender. b. Is mobile and not hard. c. Disappears when the patient smiles. d. Is hard and fixed to the surrounding structures.

B Painless, rapidly growing nodules may be cancerous, especially the appearance of a single nodule in a young person. However, cancerous nodules tend to be hard and fixed to surrounding structures, not mobile.

A patient has suddenly developed shortness of breath and appears to be in significant respiratory distress. After calling the physician and placing the patient on oxygen, which of these actions is the best for the nurse to take when further assessing the patient? a. Count the patient's respirations. b. Bilaterally percuss the thorax, noting any differences in percussion tones. c. Call for a chest x-ray study, and wait for the results before beginning an assessment. d. Inspect the thorax for any new masses and bleeding associated with respirations.

B Percussion is always available, portable, and offers instant feedback regarding changes in underlying tissue density, which may yield clues of the patient's physical status.

The nurse has collected the following information on a patient: palpated blood pressure-180 mm Hg; auscultated blood pressure-170/100 mm Hg; apical pulse-60 beats per minute; radial pulse-70 beats per minute. What is the patient's pulse pressure? a. 10 b. 70 c. 80 d. 100

B Pulse pressure is the difference between systolic and diastolic blood pressure (170 - 100 = 70) and reflects the stroke volume.

The nurse is performing an eye assessment on an 80-year-old patient. Which of these findings is considered abnormal? a. Decrease in tear production b. Unequal pupillary constriction in response to light c. Presence of arcus senilis observed around the cornea d. Loss of the outer hair on the eyebrows attributable to a decrease in hair follicles

B Pupils are small in the older adult, and the pupillary light reflex may be slowed, but pupillary constriction should be symmetric. The assessment findings in the other responses are considered normal in older persons.

A 31-year-old patient tells the nurse that he has noticed pain in his left ear when people speak loudly to him. The nurse knows that this finding: a. Is normal for people of his age. b. Is a characteristic of recruitment. c. May indicate a middle ear infection. d. Indicates that the patient has a cerumen impaction.

B Recruitment is significant hearing loss occurring when speech is at low intensity, but sound actually becomes painful when the speaker repeats at a louder volume. The other responses are not correct.

Skin turgor checks are performed to determine a. the temperature of the skin. b. hydration status. c. skin texture. d. the extent of an ecchymosis. e. skin moisture.

B - hydration status

The nurse keeps in mind that the most important reason to share information and to offer brief teaching while performing the physical examination is to help the: a. Examiner feel more comfortable and to gain control of the situation. b. Examiner to build rapport and to increase the patient's confidence in him or her. c. Patient understand his or her disease process and treatment modalities. d. Patient identify questions about his or her disease and the potential areas of patient education.

B Sharing information builds rapport and increases the patient's confidence in the examiner. It also gives the patient a little more control in a situation during which feeling completely helpless is often present

The nurse is reviewing the nutritional assessment of an 82-year-old patient. Which of these factors will most likely affect the nutritional status of an older adult? a. Increase in taste and smell b. Living alone on a fixed income c. Change in cardiovascular status d. Increase in gastrointestinal motility and absorption

B Socioeconomic conditions frequently affect the nutritional status of the aging adult; these factors should be closely evaluated. Physical limitations, income, and social isolation are frequent problems that interfere with the acquisition of a balanced diet. A decrease in taste and smell and decreased gastrointestinal motility and absorption occur with aging. Cardiovascular status is not a factor that affects an older adult's nutritional status.

A patient has come in for an examination and states, "I have this spot in front of my ear lobe on my cheek that seems to be getting bigger and is tender. What do you think it is?" The nurse notes swelling below the angle of the jaw and suspects that it could be an inflammation of his: a. Thyroid gland. b. Parotid gland. c. Occipital lymph node. d. Submental lymph node.

B Swelling of the parotid gland is evident below the angle of the jaw and is most visible when the head is extended. Painful inflammation occurs with mumps, and swelling also occurs with abscesses or tumors. Swelling occurs anterior to the lower ear lobe.

A 40-year-old woman reports a change in mole size, accompanied by color changes, itching, burning, and bleeding over the past month. She has a dark complexion and has no family history of skin cancer, but she has had many blistering sunburns in the past. The nurse would: a. Tell the patient to watch the lesion and report back in 2 months. b. Refer the patient because of the suggestion of melanoma on the basis of her symptoms. c. Ask additional questions regarding environmental irritants that may have caused this condition. d. Tell the patient that these signs suggest a compound nevus, which is very common in young to middle-aged adults.

B The ABCD danger signs of melanoma are asymmetry, border irregularity, color variation, and diameter. In addition, individuals may report a change in size, the development of itching, burning, and bleeding, or a new-pigmented lesion. Any one of these signs raises the suggestion of melanoma and warrants immediate referral.

The nurse knows that a normal finding when assessing the respiratory system of an older adult is: a. Increased thoracic expansion. b. Decreased mobility of the thorax. c. Decreased anteroposterior diameter. d. Bronchovesicular breath sounds throughout the lungs.

B The costal cartilages become calcified with aging, resulting in a less mobile thorax. Chest expansion may be somewhat decreased, and the chest cage commonly shows an increased anteroposterior diameter.

The nurse is assessing a patient's skin during an office visit. What part of the hand and technique should be used to best assess the patient's skin temperature? a. Fingertips; they are more sensitive to small changes in temperature. b. Dorsal surface of the hand; the skin is thinner on this surface than on the palms. c. Ulnar portion of the hand; increased blood supply in this area enhances temperature sensitivity. d. Palmar surface of the hand; this surface is the most sensitive to temperature variations because of its increased nerve supply in this area.

B The dorsa (backs) of the hands and fingers are best for determining temperature because the skin is thinner on the dorsal surfaces than on the palms. Fingertips are best for fine, tactile discrimination. The other responses are not useful for palpation.

A patient comes to the clinic complaining of neck and shoulder pain and is unable to turn her head. The nurse suspects damage to CN ______ and proceeds with the examination by _____________. a. XI; palpating the anterior and posterior triangles b. XI; asking the patient to shrug her shoulders against resistance c. XII; percussing the sternomastoid and submandibular neck muscles d. XII; assessing for a positive Romberg sign

B The major neck muscles are the sternomastoid and the trapezius. They are innervated by CN XI, the spinal accessory. The innervated muscles assist with head rotation and head flexion, movement of the shoulders, and extension and turning of the head.

A 1-month-old infant has a head measurement of 34 cm and has a chest circumference of 32 cm. Based on the interpretation of these findings, the nurse would: a. Refer the infant to a physician for further evaluation. b. Consider these findings normal for a 1-month-old infant. c. Expect the chest circumference to be greater than the head circumference. d. Ask the parent to return in 2 weeks to re-evaluate the head and chest circumferences.

B The newborn's head measures approximately 32 to 38 cm and is approximately 2 cm larger than the chest circumference. Between 6 months and 2 years, both measurements are approximately the same, and after age 2 years, the chest circumference is greater than the head circumference.

A 4-month-old child is at the clinic for a well-baby check-up and immunizations. Which of these actions is most appropriate when the nurse is assessing an infant's vital signs? a. The infant's radial pulse should be palpated, and the nurse should notice any fluctuations resulting from activity or exercise. b. The nurse should auscultate an apical rate for 1 minute and then assess for any normal irregularities, such as sinus arrhythmia. c. The infant's blood pressure should be assessed by using a stethoscope with a large diaphragm piece to hear the soft muffled Korotkoff sounds. d. The infant's chest should be observed and the respiratory rate counted for 1 minute; the respiratory pattern may vary significantly.

B The nurse palpates or auscultates an apical rate with infants and toddlers. The pulse should be counted for 1 full minute to account for normal irregularities, such as sinus arrhythmia. Children younger than 3 years of age have such small arm vessels; consequently, hearing Korotkoff sounds with a stethoscope is difficult. The nurse should use either an electronic blood pressure device that uses oscillometry or a Doppler ultrasound device to amplify the sounds.

A flat, nonpalpable lesion is described as a macule if the diameter is a. greater than 1 cm. b. less than 1 cm. c. greater than 2 cm. d. too irregular to measure. e. exactly 5 mm.

B - less than 1 cm.

A mother brings her newborn in for an assessment and asks, "Is there something wrong with my baby? His head seems so big." Which statement is true regarding the relative proportions of the head and trunk of the newborn? a. At birth, the head is one fifth the total length. b. Head circumference should be greater than chest circumference at birth. c. The head size reaches 90% of its final size when the child is 3 years old. d. When the anterior fontanel closes at 2 months, the head will be more proportioned to the body.

B The nurse recognizes that during the fetal period, head growth predominates. Head size is greater than chest circumference at birth, and the head size grows during childhood, reaching 90% of its final size when the child is age 6 years.

When performing a physical examination, safety must be considered to protect the examiner and the patient against the spread of infection. Which of these statements describes the most appropriate action the nurse should take when performing a physical examination? a. Washing one's hands after removing gloves is not necessary, as long as the gloves are still intact. b. Hands are washed before and after every physical patient encounter. c. Hands are washed before the examination of each body system to prevent the spread of bacteria from one part of the body to another. d. Gloves are worn throughout the entire examination to demonstrate to the patient concern regarding the spread of infectious diseases.

B The nurse should wash his or her hands before and after every physical patient encounter; after contact with blood, body fluids, secretions, and excretions; after contact with any equipment contaminated with body fluids; and after removing gloves. Hands should be washed after gloves have been removed, even if the gloves appear to be intact. Gloves should be worn when potential contact with any body fluids is present.

The nurse is palpating the sinus areas. If the findings are normal, then the patient should report which sensation? a. No sensation b. Firm pressure c. Pain during palpation d. Pain sensation behind eyes

B The person should feel firm pressure but no pain. Sinus areas are tender to palpation in persons with chronic allergies or an acute infection (sinusitis).

When performing a physical assessment, the first technique the nurse will always use is: a. Palpation. b. Inspection. c. Percussion. d. Auscultation.

B The skills requisite for the physical examination are inspection, palpation, percussion, and auscultation. The skills are performed one at a time and in this order (with the exception of the abdominal assessment, during which auscultation takes place before palpation and percussion). The assessment of each body system begins with inspection. A focused inspection takes time and yields a surprising amount of information.

The nurse keeps in mind that a thorough skin assessment is extremely important because the skin holds information about a person's: a. Support systems. b. Circulatory status. c. Socioeconomic status. d. Psychological wellness.

B The skin holds information about the body's circulation, nutritional status, and signs of systemic diseases, as well as topical data on the integumentary system itself.

During an examination of the anterior thorax, the nurse is aware that the trachea bifurcates anteriorly at the: a. Costal angle. b. Sternal angle. c. Xiphoid process. d. Suprasternal notch.

B The sternal angle marks the site of tracheal bifurcation into the right and left main bronchi; it corresponds with the upper borders of the atria of the heart, and it lies above the fourth thoracic vertebra on the back.

The nurse is teaching a class on basic assessment skills. Which of these statements is true regarding the stethoscope and its use? a. Slope of the earpieces should point posteriorly (toward the occiput). b. Although the stethoscope does not magnify sound, it does block out extraneous room noise. c. Fit and quality of the stethoscope are not as important as its ability to magnify sound. d. Ideal tubing length should be 22 inches to dampen the distortion of sound.

B The stethoscope does not magnify sound, but it does block out extraneous room sounds. The slope of the earpieces should point forward toward the examiner's nose. Long tubing will distort sound. The fit and quality of the stethoscope are both important.

Which of these actions illustrates the correct technique the nurse should use when assessing oral temperature with a mercury thermometer? a. Wait 30 minutes if the patient has ingested hot or iced liquids. b. Leave the thermometer in place 3 to 4 minutes if the patient is afebrile. c. Place the thermometer in front of the tongue, and ask the patient to close his or her lips. d. Shake the mercury-in-glass thermometer down to below 36.6° C before taking the temperature.

B The thermometer should be left in place 3 to 4 minutes if the person is afebrile and up to 8 minutes if the person is febrile. The nurse should wait 15 minutes if the person has just ingested hot or iced liquids and 2 minutes if he or she has just smoked.

In assessing the tonsils of a 30 year old, the nurse notices that they are involuted, granular in appearance, and appear to have deep crypts. What is correct response to these findings? a. Refer the patient to a throat specialist. b. No response is needed; this appearance is normal for the tonsils. c. Continue with the assessment, looking for any other abnormal findings. d. Obtain a throat culture on the patient for possible streptococcal (strep) infection.

B The tonsils are the same color as the surrounding mucous membrane, although they look more granular and their surface shows deep crypts. Tonsillar tissue enlarges during childhood until puberty and then involutes.

The physician reports that a patient with a neck tumor has a tracheal shift. The nurse is aware that this means that the patient's trachea is: a. Pulled to the affected side. b. Pushed to the unaffected side. c. Pulled downward. d. Pulled downward in a rhythmic pattern.

B The trachea is pushed to the unaffected side with an aortic aneurysm, a tumor, unilateral thyroid lobe enlargement, or a pneumothorax. The trachea is pulled to the affected side with large atelectasis, pleural adhesions, or fibrosis. Tracheal tug is a rhythmic downward pull that is synchronous with systole and occurs with aortic arch aneurysm.

When examining the ear with an otoscope, the nurse notes that the tympanic membrane should appear: a. Light pink with a slight bulge. b. Pearly gray and slightly concave. c. Pulled in at the base of the cone of light. d. Whitish with a small fleck of light in the superior portion.

B The tympanic membrane is a translucent membrane with a pearly gray color and a prominent cone of light in the anteroinferior quadrant, which is the reflection of the otoscope light. The tympanic membrane is oval and slightly concave, pulled in at its center by the malleus, which is one of the middle ear ossicles.

The nurse is assessing a patient who has liver disease for jaundice. Which of these assessment findings is indicative of true jaundice? a. Yellow patches in the outer sclera b. Yellow color of the sclera that extends up to the iris c. Skin that appears yellow when examined under low light d. Yellow deposits on the palms and soles of the feet where jaundice first appears

B The yellow sclera of jaundice extends up to the edge of the iris. Calluses on the palms and soles of the feet often appear yellow but are not classified as jaundice. Scleral jaundice should not be confused with the normal yellow subconjunctival fatty deposits that are common in the outer sclera of dark-skinned persons.

During an assessment of an adult, the nurse has noted unequal chest expansion and recognizes that this occurs in which situation? a. In an obese patient b. When part of the lung is obstructed or collapsed c. When bulging of the intercostal spaces is present d. When accessory muscles are used to augment respiratory effort

B Unequal chest expansion occurs when part of the lung is obstructed or collapsed, as with pneumonia, or when guarding to avoid postoperative incisional pain.

The nurse notices a colleague is preparing to check the blood pressure of a patient who is obese by using a standard-sized blood pressure cuff. The nurse should expect the reading to: a. Yield a falsely low blood pressure. b. Yield a falsely high blood pressure. c. Be the same, regardless of cuff size. d. Vary as a result of the technique of the person performing the assessment.

B Using a cuff that is too narrow yields a falsely high blood pressure because it takes extra pressure to compress the artery.

When assessing a patient's pain, the nurse knows that an example of visceral pain would be: a. Hip fracture. b. Cholecystitis. c. Second-degree burns. d. Pain after a leg amputation.

B Visceral pain originates from the larger interior organs, such as the gallbladder, liver, or kidneys.

The nurse is counting an infant's respirations. Which technique is correct? a. Watching the chest rise and fall b. Watching the abdomen for movement c. Placing a hand across the infant's chest d. Using a stethoscope to listen to the breath sounds

B Watching the abdomen for movement is the correct technique because the infant's respirations are normally more diaphragmatic than thoracic. The other responses do not reflect correct techniques.

With which of these patients would it be most appropriate for the nurse to use games during the assessment, such as having the patient "blow out" the light on the penlight? a. Infant b. Preschool child c. School-age child d. Adolescent

B When assessing preschool children, using games or allowing them to play with the equipment to reduce their fears can be helpful. Such games are not appropriate for the other age groups.

While obtaining a health history, a patient tells the nurse that he has frequent nosebleeds and asks the best way to get them to stop. What would be the nurse's best response? a. "While sitting up, place a cold compress over your nose." b. "Sit up with your head tilted forward and pinch your nose." c. "Just allow the bleeding to stop on its own, but don't blow your nose." d. "Lie on your back with your head tilted back and pinch your nose."

B With a nosebleed, the person should sit up with the head tilted forward and pinch the nose between the thumb and forefinger for 5 to 15 minutes.

A teenage patient comes to the emergency department with complaints of an inability to breathe and a sharp pain in the left side of his chest. The assessment findings include cyanosis, tachypnea, tracheal deviation to the right, decreased tactile fremitus on the left, hyperresonance on the left, and decreased breath sounds on the left. The nurse interprets that these assessment findings are consistent with: a. Bronchitis. b. Pneumothorax. c. Acute pneumonia. d. Asthmatic attack.

B With a pneumothorax, free air in the pleural space causes partial or complete lung collapse. If the pneumothorax is large, then tachypnea and cyanosis are evident. Unequal chest expansion, decreased or absent tactile fremitus, tracheal deviation to the unaffected side, decreased chest expansion, hyperresonant percussion tones, and decreased or absent breath sounds are found with the presence of pneumothorax. (See Table 18-8 for descriptions of the other conditions.)

An older adult woman is brought to the emergency department after being found lying on the kitchen floor for 2 days; she is extremely dehydrated. What would the nurse expect to see during the examination? a. Smooth mucous membranes and lips b. Dry mucous membranes and cracked lips c. Pale mucous membranes d. White patches on the mucous membranes

B With dehydration, mucous membranes appear dry and the lips look parched and cracked. The other responses are not found in dehydration.

A 17-year-old student is a swimmer on her high school's swim team. She has had three bouts of otitis externa this season and wants to know what to do to prevent it. The nurse instructs her to: a. Use a cotton-tipped swab to dry the ear canals thoroughly after each swim. b. Use rubbing alcohol or 2% acetic acid eardrops after every swim. c. Irrigate the ears with warm water and a bulb syringe after each swim. d. Rinse the ears with a warmed solution of mineral oil and hydrogen peroxide.

B With otitis externa (swimmer's ear), swimming causes the external canal to become waterlogged and swell; skinfolds are set up for infection. Otitis externa can be prevented by using rubbing alcohol or 2% acetic acid eardrops after every swim.

The nurse is preparing to examine a 4-year-old child. Which action is appropriate for this age group? a. Explain the procedures in detail to alleviate the child's anxiety. b. Give the child feedback and reassurance during the examination. c. Do not ask the child to remove his or her clothes because children at this age are usually very private. d. Perform an examination of the ear, nose, and throat first, and then examine the thorax and abdomen.

B With preschool children, short, simple explanations should be used. Children at this age are usually willing to undress. An examination of the head should be performed last. During the examination, needed feedback and reassurance should be given to the preschooler.

A patient comes to the emergency department after a boxing match, and his left eye is swollen almost shut. He has bruises on his face and neck. He says he is worried because he "can't see well" from his left eye. The physician suspects retinal damage. The nurse recognizes that signs of retinal detachment include: a. Loss of central vision. b. Shadow or diminished vision in one quadrant or one half of the visual field. c. Loss of peripheral vision. d. Sudden loss of pupillary constriction and accommodation.

B With retinal detachment, the person has shadows or diminished vision in one quadrant or one half of the visual field. The other responses are not signs of retinal detachment.

In performing a voice test to assess hearing, which of these actions would the nurse perform? a. Shield the lips so that the sound is muffled. b. Whisper a set of random numbers and letters, and then ask the patient to repeat them. c. Ask the patient to place his finger in his ear to occlude outside noise. d. Stand approximately 4 feet away to ensure that the patient can really hear at this distance.

B With the head 30 to 60 cm (1 to 2 feet) from the patient's ear, the examiner exhales and slowly whispers a set of random numbers and letters, such as "5, B, 6." Normally, the patient is asked to repeat each number and letter correctly after hearing the examiner say them.

Which disorder is a dysfunction of cell-mediated immunity? a. Acute idiopathic polyneuritis b. Acquired immunodeficiency syndrome (AIDS) c. Epstein-Barr virus (EBV) mononucleosis d. Systemic lupus erythematosus (SLE) e. Toxoplasmosis

B - Acquired immunodeficiency syndrome (AIDS)

Which cultural group has the lowest incidence of nevi? a. Native Americans b. African Americans c. Mexican Americans d. Asians e. Eastern Europeans

B - African Americans

Mr. L has cyanotic lips and nail beds. His lips are pursed, and he has nasal flaring. You susp difficulty. What additional sign would correspond with your impression? a. Callus formation on the heels b. Clubbing of the fingers c. Graying of the hair d. Swollen toes and ankles e. Positioning of the head

B - Clubbing of the fingers

Epiglottitis has frequently associated with infection by which organism? a. Respiratory syncytial virus b. Haemophilus influenzae type B c. Adenovirus d. Parainfluenza virus e. Human metapneumovirus

B - Haemophilus influenzae type B

Nancy is a 16-year-old young woman who presents to the clinic with complaints of severe, a Nancy, apart from occasional colds, is not prone to respiratory problems. What potential risk concerning Nancy's present problem? a. Anorexia symptoms b. Illegal drug use c. Last menses d. Signs of rheumatic fever e. Sexual activity

B - Illegal drug use

During history taking, a mother states that her son awoke in the middle of the night complain your inspection reveals honey-colored exudate from the vesicular rash on his legs. Which co findings? a. Exanthem b. Impetigo c. Solar keratoses d. Trichotillomania e. Drug eruption

B - Impetigo Impetigo causes intense pruritus, regional lymphadenopathy, and honey-colored exudative cr and dry.

A 17-year-old student complains of a "rash for 3 days." You note pale, erythematous oval pla scales and are arranged in a fernlike pattern with parallel alignment. What is the nurse's next a. Teach infectious control measures. b. Inquire about another recent skin lesion. c. Inspect the palms and the soles. d. Inform the patient that this will resolve within 1 week. e. Ask about sexual contacts.

B - Inquire about another recent skin lesion The described rash is the typical presentation of pityriasis rosea. The rash is not infectious o palms and soles, and usually lasts for several weeks. Pityriasis rosea begins with a sudden generalized eruption to the trunk and extremities following 1 to 3 weeks later.

In what position can the mediastinal crunch (Hamman sign) be heard best? a. In a supine position b. Lying on the left side c. Sitting completely upright d. With the head elevated 30 degrees e. In a prone position

B - Lying on the left side

Which chest structure contains all the thoracic viscera except the lungs? a. Manubrium b. Mediastinum c. Sternum d. Xiphoid e. Pleural cavities

B - Mediastinum

Which type of apnea requires immediate action? a. Primary apnea b. Secondary apnea c. Sleep apnea d. Periodic apnea of the newborn e. Apnea of prematurity

B - Secondary apnea

Laryngeal obstructions would elicit which breath sound? a. Fremitus b. Stridor c. Rhonchi d. Crepitus e. Wheezing

B - Stridor

The nurse inspects an annular lesion. What type of additional lighting source should be used a. Florescent lighting b. Wood's lamp c. Goose-neck lamp d. Sunlight e. Cobalt blue

B - Wood's lamp

The nurse blanches over a vascular lesion on a pregnant patient. The site blanches and refill nurse documents this lesion as a. telangiectasia. b. a spider angioma. c. petechiae. d. purpura. e. ecchymosis.

B - a spider angioma.

Bradypnea may accompany a. pneumothorax. b. an excellent level of cardiovascular fitness. c. ascites. d. a pulmonary embolus. e. anxiety.

B - an excellent level of cardiovascular fitness.

Equipment for examining the lymphatic system includes a a. caliper. b. centimeter ruler. c. goniometer. d. syringe and needle. e. stethoscope.

B - centimeter ruler. The centimeter ruler and the marking pencil are the only equipment needed for this system e and outline the borders of the nodes

Large palatine tonsils are common in a. females. b. children. c. adolescents. d. young adults. e. older adults.

B - children

To find the infection site associated with acute lymphangitis, the examiner should look _____ a. proximal b. distal c. contralateral d. anterior e. posterior

B - distal

Transillumination is done during an examination of the lymphatic system to a. detect lymphatic pulsation. b. distinguish nodes from cysts. c. evaluate nodal contours. d. observe erythematous lesions. e. distinguish blood vessels from nodes

B - distinguish nodes from cysts

Both pleural effusion and lobar pneumonia are characterized by _____ percussion. a. tympany heard with b. dullness heard on c. resonance heard on d. hyperresonance heard on e. occasional hyperresonance heard on

B - dullness heard on

The most common inflammatory skin condition is a. cutis marmorata. b. eczematous dermatitis. c. intradermal nevus. d. pityriasis rosea. e. psoriasis.

B - eczematous dermatitis The most common inflammatory skin disorder is eczematous dermatitis. Cutis marmorata an pityriasis rosea is not as common as eczema, and psoriasis is a disease of keratin synthesi

The type and brand of grooming products used are important to the health history of a. adolescents. b. everyone. c. older adults. d. persons with rashes. e. children.

B - everyone.

To palpate the inguinal nodes, you should have the patient a. bend over a table and cough. b. lie supine, with the knees slightly flexed. c. lie supine, with the legs extended. d. stand with the back extended. e. lie prone, with the knees slightly flexed.

B - lie supine, with the knees slightly flexed

A red streak that follows the course of the lymphatic collecting duct is a finding associated w a. Hodgkin lymphoma. b. lymphangitis. c. lymphedema. d. non-Hodgkin lymphoma. e. roseola infantum.

B - lymphangitis.

An increased number of lymphocytes in the blood represents a systemic response to _____ a. most bacterial b. most viral c. all parasitic d. HIV e. all fungal

B - most viral An increased number of lymphocytes in the blood represents a systemic response to most v infections.

Transient mottling of the patient's skin in a cool room is a common finding in a. menopausal women. b. newborn infants. c. pregnant women. d. sedentary adults. e. older adults.

B - newborn infants.

Small, less than 0.5-cm in diameter, red-purple nonblanchable discolorations of the skin are a. ecchymoses. b. petechiae. c. spider veins. d. telangiectasias. e. purpura.

B - petechiae.

Assessment of poor hygiene, healed fractures with deformity, or unexplained trauma in older a. sexual abuse. b. physical neglect. c. psychologic abuse. d. violated rights. e. financial abuse.

B - physical neglect. The question describes is the most common form of elder abuse—physical neglect.

Nodes located over the mastoid process are the _____ nodes. a. preauricular b. postauricular c. occipital d. epitrochlear e. posterior cervical

B - postauricular

Risk factors for skin cancer include a. an olive complexion. b. repeated trauma or irritation to the skin. c. a history of allergic reactions to sunscreen. d. dark eyes and hair. e. pigmented bands in the nails.

B - repeated trauma or irritation to the skin.

Nodes located nearest the palatine tonsils are the a. supraclavicular. b. retropharyngeal. c. submental. d. submandibular. e. posterior cervical.

B - retropharyngeal. The retropharyngeal (tonsillar) nodes and the parotid nodes lie at the angle of the mandible. T lie more distal to the mandible. The posterior cervical nodes lie posterior to the sternocleidom supraclavicular nodes lie superior to the clavicles.

A congenital defect in the immune system of a 2-week-old infant may be suspected if a. there are small, palpable inguinal nodes. b. the umbilical cord has not yet dropped off. c. the tonsils are visible. d. the thymus is visible on a chest radiograph. e. the small postauricular nodes are palpable.

B - the umbilical cord has not yet dropped off. In some infants, delayed separation of the umbilical cord has been associated with abnorma

During an admission assessment of a patient with dementia, the nurse assesses for pain because the patient has recently had several falls. Which of these are appropriate for the nurse to assess in a patient with dementia? Select all that apply. a. Ask the patient, "Do you have pain?" b. Assess the patient's breathing independent of vocalization. c. Note whether the patient is calling out, groaning, or crying. d. Have the patient rate pain on a 1-to-10 scale. e. Observe the patient's body language for pacing and agitation.

B, C, E Patients with dementia may say "no" when, in reality, they are very uncomfortable because words have lost their meaning. Patients with dementia become less able to identify and describe pain over time, although pain is still present. People with dementia communicate pain through their behaviors. Agitation, pacing, and repetitive yelling may indicate pain and not a worsening of the dementia. (See Figure 10-10 for the Pain Assessment in Advanced Dementia [PAINAD] scale, which may also be used to assess pain in persons with dementia.)

The nurse is testing the hearing of a 78-year-old man and is reminded of the changes in hearing that occur with aging that include which of the following? Select all that apply. a. Hearing loss related to aging begins in the mid 40s. b. Progression of hearing loss is slow. c. The aging person has low-frequency tone loss. d. The aging person may find it harder to hear consonants than vowels. e. Sounds may be garbled and difficult to localize. f. Hearing loss reflects nerve degeneration of the middle ear.

B, D, E Presbycusis is a type of hearing loss that occurs with aging and is found in 60% of those older than 65 years. It is a gradual sensorineural loss caused by nerve degeneration in the inner ear or auditory nerve, and it slowly progresses after the age of 50 years. The person first notices a high-frequency tone loss; it is harder to hear consonants (high-pitched components of speech) than vowels, which makes words sound garbled. The ability to localize sound is also impaired.

During an assessment, a patient mentions that "I just can't smell like I used to. I can barely smell the roses in my garden. Why is that?" For which possible causes of changes in the sense of smell will the nurse assess? Select all that apply. a. Chronic alcohol use b. Cigarette smoking c. Frequent episodes of strep throat d. Chronic allergies e. Aging f. Herpes simplex virus I

B, D, E Sen The sense of smell diminishes with cigarette smoking, chronic allergies, and aging. Chronic alcohol use, a history of strep throat, and herpes simplex virus I are not associated with changes in the sense of smell.

The nurse is assessing a 1-month-old infant at his well-baby checkup. Which assessment findings are appropriate for this age? Select all that apply. a. Head circumference equal to chest circumference b. Head circumference greater than chest circumference c. Head circumference less than chest circumference d. Fontanels firm and slightly concave e. Absent tonic neck reflex f. Nonpalpable cervical lymph nodes

B, D, F An infant's head circumference is larger than the chest circumference. At age 2 years, both measurements are the same. During childhood, the chest circumference grows to exceed the head circumference by 5 to 7 cm. The fontanels should feel firm and slightly concave in the infant, and they should close by age 9 months. The tonic neck reflex is present until between 3 and 4 months of age, and cervical lymph nodes are normally nonpalpable in an infant.

During an examination, a patient states that she was diagnosed with open-angle glaucoma 2 years ago. The nurse assesses for characteristics of open-angle glaucoma. Which of these are characteristics of open-angle glaucoma? Select all that apply. a. Patient may experience sensitivity to light, nausea, and halos around lights. b. Patient experiences tunnel vision in the late stages. c. Immediate treatment is needed. d. Vision loss begins with peripheral vision. e. Open-angle glaucoma causes sudden attacks of increased pressure that cause blurred vision. f. Virtually no symptoms are exhibited.

B, D, F Open-angle glaucoma is the most common type of glaucoma; virtually no symptoms are exhibited. Vision loss begins with the peripheral vision, which often goes unnoticed because individuals learn to compensate intuitively by turning their heads. The other characteristics are those of closed-angle glaucoma

A patient has been admitted to a hospital after the staff in the nursing home noticed a pressure ulcer in his sacral area. The nurse examines the pressure ulcer and determines that it is a stage II ulcer. Which of these findings are characteristic of a stage II pressure ulcer? Select all that apply. a. Intact skin appears red but is not broken. b. Partial thickness skin erosion is observed with a loss of epidermis or dermis. c. Ulcer extends into the subcutaneous tissue. d. Localized redness in light skin will blanch with fingertip pressure. e. Open blister areas have a red-pink wound bed. f. Patches of eschar cover parts of the wound.

B, E Stage I pressure ulcers have intact skin that appears red but is not broken, and localized redness in intact skin will blanche with fingertip pressure. Stage II pressure ulcers have partial thickness skin erosion with a loss of epidermis or also the dermis; open blisters have a red-pink wound bed. Stage III pressure ulcers are full thickness, extending into the subcutaneous tissue; subcutaneous fat may be seen but not muscle, bone, or tendon. Stage IV pressure ulcers involve all skin layers and extend into supporting tissue, exposing muscle, bone, and tendon. Slough (stringy matter attached to the wound bed) or eschar (black or brown necrotic tissue) may be present.

The nurse is assessing a patient in the hospital who has received numerous antibiotics and notices that his tongue appears to be black and hairy. In response to his concern, what would the nurse say? a. "We will need to get a biopsy to determine the cause." b. "This is an overgrowth of hair and will go away in a few days." c. "Black, hairy tongue is a fungal infection caused by all the antibiotics you have received." d. "This is probably caused by the same bacteria you had in your lungs."

C A black, hairy tongue is not really hair but the elongation of filiform papillae and painless overgrowth of mycelial threads of fungus infection on the tongue. It occurs after the use of antibiotics, which inhibit normal bacteria and allow a proliferation of fungus.

A patient comes in for a physical examination and complains of "freezing to death" while waiting for her examination. The nurse notes that her skin is pale and cool and attributes this finding to: a. Venous pooling. b. Peripheral vasodilation. c. Peripheral vasoconstriction. d. Decreased arterial perfusion.

C A chilly or air-conditioned environment causes vasoconstriction, which results in false pallor and coolness (see Table 12-1).

A patient has been shown to have a sensorineural hearing loss. During the assessment, it would be important for the nurse to: a. Speak loudly so the patient can hear the questions. b. Assess for middle ear infection as a possible cause. c. Ask the patient what medications he is currently taking. d. Look for the source of the obstruction in the external ear.

C A simple increase in amplitude may not enable the person to understand spoken words. Sensorineural hearing loss may be caused by presbycusis, which is a gradual nerve degeneration that occurs with aging and by ototoxic drugs, which affect the hair cells in the cochlea.

During an admission assessment, the nurse notices that a male patient has an enlarged and rather thick skull. The nurse suspects acromegaly and would further assess for: a. Exophthalmos. b. Bowed long bones. c. Coarse facial features. d. Acorn-shaped cranium.

C Acromegaly is excessive secretion of growth hormone that creates an enlarged skull and thickened cranial bones. Patients will have elongated heads, massive faces, prominent noses and lower jaws, heavy eyebrow ridges, and coarse facial features. Exophthalmos is associated with hyperthyroidism. Bowed long bones and an acorn-shaped cranium result from Paget disease.

When evaluating a patient's pain, the nurse knows that an example of acute pain would be: a. Arthritic pain. b. Fibromyalgia. c. Kidney stones. d. Low back pain.

C Acute pain is short-term and dissipates after an injury heals, such as with kidney stones. The other conditions are examples of chronic pain during which the pain continues for 6 months or longer and does not stop when the injury heals.

During an examination of a female patient, the nurse notes lymphadenopathy and suspects an acute infection. Acutely infected lymph nodes would be: a. Clumped. b. Unilateral. c. Firm but freely movable. d. Firm and nontender.

C Acutely infected lymph nodes are bilateral, enlarged, warm, tender, and firm but freely movable. Unilaterally enlarged nodes that are firm and nontender may indicate cancer.

A patient with a middle ear infection asks the nurse, "What does the middle ear do?" The nurse responds by telling the patient that the middle ear functions to: a. Maintain balance. b. Interpret sounds as they enter the ear. c. Conduct vibrations of sounds to the inner ear. d. Increase amplitude of sound for the inner ear to function.

C Among its other functions, the middle ear conducts sound vibrations from the outer ear to the central hearing apparatus in the inner ear. The other responses are not functions of the middle ear.

The nurse is taking an initial blood pressure reading on a 72-year-old patient with documented hypertension. How should the nurse proceed? a. Cuff should be placed on the patient's arm and inflated 30 mm Hg above the patient's pulse rate. b. Cuff should be inflated to 200 mm Hg in an attempt to obtain the most accurate systolic reading. c. Cuff should be inflated 30 mm Hg above the point at which the palpated pulse disappears. d. After confirming the patient's previous blood pressure readings, the cuff should be inflated 30 mm Hg above the highest systolic reading recorded.

C An auscultatory gap occurs in approximately 5% of the people, most often in those with hypertension. To check for the presence of an auscultatory gap, the cuff should be inflated 20 to 30 mm Hg beyond the point at which the palpated pulse disappears.

The nurse is performing an assessment on a 65-year-old man. He reports a crusty nodule behind the pinna. It intermittently bleeds and has not healed over the past 6 months. On physical assessment, the nurse finds an ulcerated crusted nodule with an indurated base. The preliminary analysis in this situation is that this: a. Is most likely a benign sebaceous cyst. b. Is most likely a keloid. c. Could be a potential carcinoma, and the patient should be referred for a biopsy. d. Is a tophus, which is common in the older adult and is a sign of gout.

C An ulcerated crusted nodule with an indurated base that fails to heal is characteristic of a carcinoma. These lesions fail to heal and intermittently bleed. Individuals with such symptoms should be referred for a biopsy (see Table 15-2). The other responses are not correct.

A 60-year-old male patient has been treated for pneumonia for the past 6 weeks. He is seen today in the clinic for an "unexplained" weight loss of 10 pounds over the last 6 weeks. The nurse knows that: a. Weight loss is probably the result of unhealthy eating habits. b. Chronic diseases such as hypertension cause weight loss. c. Unexplained weight loss often accompanies short-term illnesses. d. Weight loss is probably the result of a mental health dysfunction.

C An unexplained weight loss may be a sign of a short-term illness or a chronic illness such as endocrine disease, malignancy, depression, anorexia nervosa, or bulimia.

A patient has been admitted after an accident at work. During the assessment, the patient is having trouble hearing and states, "I don't know what the matter is. All of a sudden, I can't hear you out of my left ear!" What should the nurse do next? a. Make note of this finding for the report to the next shift. b. Prepare to remove cerumen from the patient's ear. c. Notify the patient's health care provider. d. Irrigate the ear with rubbing alcohol.

C Any sudden loss of hearing in one or both ears that is not associated with an upper respiratory infection needs to be reported at once to the patient's health care provider. Hearing loss associated with trauma is often sudden. Irrigating the ear or removing cerumen is not appropriate at this time.

A 13-year-old girl is interested in obtaining information about the cause of her acne. The nurse should share with her that acne: a. Is contagious. b. Has no known cause. c. Is caused by increased sebum production. d. Has been found to be related to poor hygiene.

C Approximately 90% of males and 80% of females will develop acne; causes are increased sebum production and epithelial cells that do not desquamate normally.

During an assessment of a 20-year-old Asian patient, the nurse notices that he has dry, flaky cerumen in his canal. What is the significance of this finding? This finding: a. Is probably the result of lesions from eczema in his ear. b. Represents poor hygiene. c. Is a normal finding, and no further follow-up is necessary. d. Could be indicative of change in cilia; the nurse should assess for hearing loss.

C Asians and Native Americans are more likely to have dry cerumen, whereas Blacks and Whites usually have wet cerumen.

The nurse is assessing for inflammation in a dark-skinned person. Which technique is the best? a. Assessing the skin for cyanosis and swelling b. Assessing the oral mucosa for generalized erythema c. Palpating the skin for edema and increased warmth d. Palpating for tenderness and local areas of ecchymosis

C Because inflammation cannot be seen in dark-skinned persons, palpating the skin for increased warmth, for taut or tightly pulled surfaces that may be indicative of edema, and for a hardening of deep tissues or blood vessels is often necessary.

The nurse is reviewing the characteristics of breath sounds. Which statement about bronchovesicular breath sounds is true? Bronchovesicular breath sounds are: a. Musical in quality. b. Usually caused by a pathologic disease. c. Expected near the major airways. d. Similar to bronchial sounds except shorter in duration.

C Bronchovesicular breath sounds are heard over major bronchi where fewer alveoli are located posteriorly—between the scapulae, especially on the right; and anteriorly, around the upper sternum in the first and second intercostal spaces. The other responses are not correct.

When assessing the respiratory system of a 4-year-old child, which of these findings would the nurse expect? a. Crepitus palpated at the costochondral junctions b. No diaphragmatic excursion as a result of a child's decreased inspiratory volume c. Presence of bronchovesicular breath sounds in the peripheral lung fields d. Irregular respiratory pattern and a respiratory rate of 40 breaths per minute at rest

C Bronchovesicular breath sounds in the peripheral lung fields of the infant and young child up to age 5 or 6 years are normal findings. Their thin chest walls with underdeveloped musculature do not dampen the sound, as do the thicker chest walls of adults; therefore, breath sounds are loud and harsh.

A mother brings in her newborn infant for an assessment and tells the nurse that she has noticed that whenever her newborn's head is turned to the right side, she straightens out the arm and leg on the same side and flexes the opposite arm and leg. After observing this on examination, the nurse tells her that this reflex is: a. Abnormal and is called the atonic neck reflex. b. Normal and should disappear by the first year of life. c. Normal and is called the tonic neck reflex, which should disappear between 3 and 4 months of age. d. Abnormal. The baby should be flexing the arm and leg on the right side of his body when the head is turned to the right.

C By 2 weeks, the infant shows the tonic neck reflex when supine and the head is turned to one side (extension of same arm and leg, flexion of opposite arm and leg). The tonic neck reflex disappears between 3 and 4 months of age.

When measuring a patient's body temperature, the nurse keeps in mind that body temperature is influenced by: a. Constipation. b. Patient's emotional state. c. Diurnal cycle. d. Nocturnal cycle.

C Normal temperature is influenced by the diurnal cycle, exercise, and age. The other responses do not influence body temperature.

The nurse is assessing an 80-year-old male patient. Which assessment findings would be considered normal? a. Increase in body weight from his younger years b. Additional deposits of fat on the thighs and lower legs c. Presence of kyphosis and flexion in the knees and hips d. Change in overall body proportion, including a longer trunk and shorter extremities

C Changes that occur in the aging person include more prominent bony landmarks, decreased body weight (especially in men), a decrease in subcutaneous fat from the face and periphery, and additional fat deposited on the abdomen and hips. Postural changes of kyphosis and slight flexion in the knees and hips also occur.

A 42-year-old woman complains that she has noticed several small, slightly raised, bright red dots on her chest. On examination, the nurse expects that the spots are probably: a. Anasarca. b. Scleroderma. c. Senile angiomas. d. Latent myeloma.

C Cherry (senile) angiomas are small, smooth, slightly raised bright red dots that commonly appear on the trunk of adults over 30 years old.

The nurse is assessing a 3 year old for "drainage from the nose." On assessment, a purulent drainage that has a very foul odor is noted from the left naris and no drainage is observed from the right naris. The child is afebrile with no other symptoms. What should the nurse do next? a. Refer to the physician for an antibiotic order. b. Have the mother bring the child back in 1 week. c. Perform an otoscopic examination of the left nares. d. Tell the mother that this drainage is normal for a child of this age.

C Children are prone to put an object up the nose, producing unilateral purulent drainage with a foul odor. Because some risk for aspiration exists, removal should be prompt.

During an assessment of a 26 year old at the clinic for "a spot on my lip I think is cancer," the nurse notices a group of clear vesicles with an erythematous base around them located at the lip-skin border. The patient mentions that she just returned from Hawaii. What would be the most appropriate response by the nurse? a. Tell the patient she needs to see a skin specialist. b. Discuss the benefits of having a biopsy performed on any unusual lesion. c. Tell the patient that these vesicles are indicative of herpes simplex I or cold sores and that they will heal in 4 to 10 days. d. Tell the patient that these vesicles are most likely the result of a riboflavin deficiency and discuss nutrition.

C Cold sores are groups of clear vesicles with a surrounding erythematous base. These evolve into pustules or crusts and heal in 4 to 10 days. The most likely site is the lip-skin junction. Infection often recurs in the same site. Recurrent herpes infections may be precipitated by sunlight, fever, colds, or allergy.

During an examination, the nurse notices that a female patient has a round "moon" face, central trunk obesity, and a cervical hump. Her skin is fragile with bruises. The nurse determines that the patient has which condition? a. Marfan syndrome b. Gigantism c. Cushing syndrome d. Acromegaly

C Cushing syndrome is characterized by weight gain and edema with central trunk and cervical obesity (buffalo hump) and round plethoric face (moon face). Excessive catabolism causes muscle wasting; weakness; thin arms and legs; reduced height; and thin, fragile skin with purple abdominal striae, bruising, and acne. (See Table 9-5, Abnormalities in Body Height and Proportion, for the definitions of the other conditions.)

Immediately after birth, the nurse is unable to suction the nares of a newborn. An attempt is made to pass a catheter through both nasal cavities with no success. What should the nurse do next? a. Attempt to suction again with a bulb syringe. b. Wait a few minutes, and try again once the infant stops crying. c. Recognize that this situation requires immediate intervention. d. Contact the physician to schedule an appointment for the infant at his or her next hospital visit.

C Determining the patency of the nares in the immediate newborn period is essential because most newborns are obligate nose breathers. Nares blocked with amniotic fluid are gently suctioned with a bulb syringe. If obstruction is suspected, then a small lumen (5 to 10 Fr) catheter is passed down each naris to confirm patency. The inability to pass a catheter through the nasal cavity indicates choanal atresia, which requires immediate intervention.

The nurse is unable to palpate the right radial pulse on a patient. The best action would be to: a. Auscultate over the area with a fetoscope. b. Use a goniometer to measure the pulsations. c. Use a Doppler device to check for pulsations over the area. d. Check for the presence of pulsations with a stethoscope.

C Doppler devices are used to augment pulse or blood pressure measurements. Goniometers measure joint range of motion. A fetoscope is used to auscultate fetal heart tones. Stethoscopes are used to auscultate breath, bowel, and heart sounds.

A woman in her 26th week of pregnancy states that she is "not really short of breath" but feels that she is aware of her breathing and the need to breathe. What is the nurse's best reply? a. "The diaphragm becomes fixed during pregnancy, making it difficult to take in a deep breath." b. "The increase in estrogen levels during pregnancy often causes a decrease in the diameter of the rib cage and makes it difficult to breathe." c. "What you are experiencing is normal. Some women may interpret this as shortness of breath, but it is a normal finding and nothing is wrong." d. "This increased awareness of the need to breathe is normal as the fetus grows because of the increased oxygen demand on the mother's body, which results in an increased respiratory rate."

C During pregnancy, the woman may develop an increased awareness of the need to breathe. Some women may interpret this as dyspnea, although structurally nothing is wrong. Increases in estrogen relax the chest cage ligaments, causing an increase in the transverse diameter. Although the growing fetus increases the oxygen demand on the mother's body, this increased demand is easily met by the increasing tidal volume (deeper breathing). Little change occurs in the respiratory rate.

The nurse is preparing to perform an otoscopic examination of a newborn infant. Which statement is true regarding this examination? a. Immobility of the drum is a normal finding. b. An injected membrane would indicate an infection. c. The normal membrane may appear thick and opaque. d. The appearance of the membrane is identical to that of an adult.

C During the first few days after the birth, the tympanic membrane of a newborn often appears thickened and opaque. It may look injected and have a mild redness from increased vascularity. The other statements are not correct

A 92-year-old patient has had a stroke. The right side of his face is drooping. The nurse might also suspect which of these assessment findings? a. Epistaxis b. Rhinorrhea c. Dysphagia d. Xerostomia

C Dysphagia is difficulty with swallowing and may occur with a variety of disorders, including stroke and other neurologic diseases. Rhinorrhea is a runny nose, epistaxis is a bloody nose, and xerostomia is a dry mouth.

A visitor from Poland who does not speak English seems to be somewhat apprehensive about the nurse examining his neck. He would probably be more comfortable with the nurse examining his thyroid gland from: a. Behind with the nurse's hands placed firmly around his neck. b. The side with the nurse's eyes averted toward the ceiling and thumbs on his neck. c. The front with the nurse's thumbs placed on either side of his trachea and his head tilted forward. d. The front with the nurse's thumbs placed on either side of his trachea and his head tilted backward.

C Examining this patient's thyroid gland from the back may be unsettling for him. It would be best to examine his thyroid gland using the anterior approach, asking him to tip his head forward and to the right and then to the left.

In a patient with acromegaly, the nurse will expect to discover which assessment findings? a. Heavy, flattened facial features b. Growth retardation and a delayed onset of puberty c. Overgrowth of bone in the face, head, hands, and feet d. Increased height and weight and delayed sexual development

C Excessive secretions of growth hormone in adulthood after normal completion of body growth causes an overgrowth of the bones in the face, head, hands, and feet but no change in height.

A mother asks when her newborn infant's eyesight will be developed. The nurse should reply: a. "Vision is not totally developed until 2 years of age." b. "Infants develop the ability to focus on an object at approximately 8 months of age." c. "By approximately 3 months of age, infants develop more coordinated eye movements and can fixate on an object." d. "Most infants have uncoordinated eye movements for the first year of life."

C Eye movements may be poorly coordinated at birth, but by 3 to 4 months of age, the infant should establish binocularity and should be able to fixate simultaneously on a single image with both eyes.

The nurse notices that a patient's palpebral fissures are not symmetric. On examination, the nurse may find that damage has occurred to which cranial nerve (CN)? a. III b. V c. VII d. VIII

C Facial muscles are mediated by CN VII; asymmetry of palpebral fissures may be attributable to damage to CN VII (Bell palsy).

The nurse is auscultating the chest in an adult. Which technique is correct? a. Instructing the patient to take deep, rapid breaths b. Instructing the patient to breathe in and out through his or her nose c. Firmly holding the diaphragm of the stethoscope against the chest d. Lightly holding the bell of the stethoscope against the chest to avoid friction

C Firmly holding the diaphragm of the stethoscope against the chest is the correct way to auscultate breath sounds. The patient should be instructed to breathe through his or her mouth, a little deeper than usual, but not to hyperventilate.

A 16-year-old girl is being seen at the clinic for gastrointestinal complaints and weight loss. The nurse determines that many of her complaints may be related to erratic eating patterns, eating predominantly fast foods, and high caffeine intake. In this situation, which is most appropriate when collecting current dietary intake information? a. Scheduling a time for direct observation of the adolescent during meals b. Asking the patient for a 24-hour diet recall, and assuming it to be reflective of a typical day for her c. Having the patient complete a food diary for 3 days, including 2 weekdays and 1 weekend day d. Using the food frequency questionnaire to identify the amount of intake of specific foods

C Food diaries require the individual to write down everything consumed for a certain time period. Because of the erratic eating patterns of this individual, assessing dietary intake over a few days would produce more accurate information regarding eating patterns. Direct observation is best used with young children or older adults.

The nurse is reviewing the technique of palpating for tactile fremitus with a new graduate. Which statement by the graduate nurse reflects a correct understanding of tactile fremitus? "Tactile fremitus: a. "Is caused by moisture in the alveoli." b. "Indicates that air is present in the subcutaneous tissues." c. "Is caused by sounds generated from the larynx." d. "Reflects the blood flow through the pulmonary arteries."

C Fremitus is a palpable vibration. Sounds generated from the larynx are transmitted through patent bronchi and the lung parenchyma to the chest wall where they are felt as vibrations. Crepitus is the term for air in the subcutaneous tissues.

The nurse is assessing a 30-year-old unemployed immigrant from an underdeveloped country who has been in the United States for 1 month. Which of these problems related to his nutritional status might the nurse expect to find? a. Obesity b. Hypotension c. Osteomalacia (softening of the bones) d. Coronary artery disease

C General undernutrition, hypertension, diarrhea, lactose intolerance, osteomalacia, scurvy, and dental caries are among the more common nutrition-related problems of new immigrants from developing countries.

During a well-baby checkup, the nurse notices that a 1-week-old infant's face looks small compared with his cranium, which seems enlarged. On further examination, the nurse also notices dilated scalp veins and downcast or "setting sun" eyes. The nurse suspects which condition? a. Craniotabes b. Microcephaly c. Hydrocephalus d. Caput succedaneum

C Hydrocephalus occurs with the obstruction of drainage of cerebrospinal fluid that results in excessive accumulation, increasing intracranial pressure, and an enlargement of the head. The face looks small, compared with the enlarged cranium, and dilated scalp veins and downcast or "setting sun" eyes are noted. Craniotabes is a softening of the skull's outer layer. Microcephaly is an abnormally small head. A caput succedaneum is edematous swelling and ecchymosis of the presenting part of the head caused by birth trauma.

A patient has been admitted to the emergency department for a suspected drug overdose. His respirations are shallow, with an irregular pattern, with a rate of 12 respirations per minute. The nurse interprets this respiration pattern as which of the following? a. Bradypnea b. Cheyne-Stokes respirations c. Hypoventilation d. Chronic obstructive breathing

C Hypoventilation is characterized by an irregular, shallow pattern, and can be caused by an overdose of narcotics or anesthetics. Bradypnea is slow breathing, with a rate less than 10 respirations per minute. (See Table 18-4 for descriptions of Cheyne-Stokes respirations and chronic obstructive breathing.)

During an assessment, the nurse knows that expected assessment findings in the normal adult lung include the presence of: a. Adventitious sounds and limited chest expansion. b. Increased tactile fremitus and dull percussion tones. c. Muffled voice sounds and symmetric tactile fremitus. d. Absent voice sounds and hyperresonant percussion tones.

C Normal lung findings include symmetric chest expansion, resonant percussion tones, vesicular breath sounds over the peripheral lung fields, muffled voice sounds, and no adventitious sounds.

The nurse knows that which statement is true regarding the pain experienced by infants? a. Pain in infants can only be assessed by physiologic changes, such as an increased heart rate. b. The FPS-R can be used to assess pain in infants. c. A procedure that induces pain in adults will also induce pain in the infant. d. Infants feel pain less than do adults.

C If a procedure or disease process causes pain in an adult, then it will also cause pain in an infant. Physiologic changes cannot be exclusively used to confirm or deny pain because other factors, such as medications, fluid status, or stress may cause physiologic changes. The FPS-R can be used starting at age 4 years.

What type of blood pressure measurement error is most likely to occur if the nurse does not check for the presence of an auscultatory gap? a. Diastolic blood pressure may not be heard. b. Diastolic blood pressure may be falsely low. c. Systolic blood pressure may be falsely low. d. Systolic blood pressure may be falsely high.

C If an auscultatory gap is undetected, then a falsely low systolic or falsely high diastolic reading may result, which is common in patients with hypertension.

A patient is seen in the clinic for complaints of "fainting episodes that started last week." How should the nurse proceed with the examination? a. Blood pressure readings are taken in both the arms and the thighs. b. The patient is assisted to a lying position, and his blood pressure is taken. c. His blood pressure is recorded in the lying, sitting, and standing positions. d. His blood pressure is recorded in the lying and sitting positions; these numbers are then averaged to obtain a mean blood pressure.

C If the person is known to have hypertension, is taking antihypertensive medications, or reports a history of fainting or syncope, then the blood pressure reading should be taken in three positions: lying, sitting, and standing.

A patient's thyroid gland is enlarged, and the nurse is preparing to auscultate the thyroid gland for the presence of a bruit. A bruit is a __________ sound that is heard best with the __________ of the stethoscope. a. Low gurgling; diaphragm b. Loud, whooshing, blowing; bell c. Soft, whooshing, pulsatile; bell d. High-pitched tinkling; diaphragm

C If the thyroid gland is enlarged, then the nurse should auscultate it for the presence of a bruit, which is a soft, pulsatile, whooshing, blowing sound heard best with the bell of the stethoscope.

Which of these interventions is most appropriate when the nurse is planning nutritional interventions for a healthy, active 74-year-old woman? a. Decreasing the amount of carbohydrates to prevent lean muscle catabolism b. Increasing the amount of soy and tofu in her diet to promote bone growth and reverse osteoporosis c. Decreasing the number of calories she is eating because of the decrease in energy requirements from the loss of lean body mass d. Increasing the number of calories she is eating because of the increased energy needs of the older adult

C Important nutritional features of the older years are a decrease in energy requirements as a result of loss of lean body mass, the most metabolically active tissue, and an increase in fat mass.

A 35-year-old pregnant woman comes to the clinic for a monthly appointment. During the assessment, the nurse notices that she has a brown patch of hyperpigmentation on her face. The nurse continues the skin assessment aware that another finding may be: a. Keratoses. b. Xerosis. c. Chloasma. d. Acrochordons.

C In pregnancy, skin changes can include striae, linea nigra (a brownish-black line down the midline), chloasma (brown patches of hyperpigmentation), and vascular spiders. Keratoses are raised, thickened areas of pigmentation that look crusted, scaly, and warty. Xerosis is dry skin. Acrochordons, or skin tags, occur more often in the aging adult.

A patient comes to the clinic and tells the nurse that he has been confined to his recliner chair for approximately 3 days with his feet down and he asks the nurse to evaluate his feet. During the assessment, the nurse might expect to find: a. Pallor b. Coolness c. Distended veins d. Prolonged capillary filling time

C Keeping the feet in a dependent position causes venous pooling, resulting in redness, warmth, and distended veins. Prolonged elevation would cause pallor and coolness. Immobilization or prolonged inactivity would cause prolonged capillary filling time (see Table 12-1).

A 70-year-old woman who loves to garden has small, flat, brown macules over her arms and hands. She asks, "What causes these liver spots?" The nurse tells her, "They are: a. "Signs of decreased hematocrit related to anemia." b. "Due to the destruction of melanin in your skin from exposure to the sun." c. "Clusters of melanocytes that appear after extensive sun exposure." d. "Areas of hyperpigmentation related to decreased perfusion and vasoconstriction."

C Liver spots, or senile lentigines, are clusters of melanocytes that appear on the forearms and dorsa of the hands after extensive sun exposure. The other responses are not correct.

When considering the concepts related to blood pressure, the nurse knows that the concept of mean arterial pressure (MAP) is best described by which statement? a. MAP is the pressure of the arterial pulse. b. MAP reflects the stroke volume of the heart. c. MAP is the pressure forcing blood into the tissues, averaged over the cardiac cycle. d. MAP is an average of the systolic and diastolic blood pressures and reflects tissue perfusion.

C MAP is the pressure that forces blood into the tissues, averaged over the cardiac cycle. Stroke volume is reflected by the blood pressure. MAP is not an arithmetic average of systolic and diastolic pressures because diastole lasts longer; rather, it is a value closer to diastolic pressure plus one third of the pulse pressure.

Which of these conditions is due to an inadequate intake of both protein and calories? a. Obesity b. Bulimia c. Marasmus d. Kwashiorkor

C Marasmus, protein-calorie malnutrition, is due to an inadequate intake of protein and calories or prolonged starvation. Obesity is due to caloric excess; bulimia is an eating disorder. Kwashiorkor is protein malnutrition.

A woman comes to the clinic and states, "I've been sick for so long! My eyes have gotten so puffy, and my eyebrows and hair have become coarse and dry." The nurse will assess for other signs and symptoms of: a. Cachexia. b. Parkinson syndrome. c. Myxedema. d. Scleroderma.

C Myxedema (hypothyroidism) is a deficiency of thyroid hormone that, when severe, causes a nonpitting edema or myxedema. The patient will have a puffy edematous face, especially around the eyes (periorbital edema); coarse facial features; dry skin; and dry, coarse hair and eyebrows. (See Table 13-4, Abnormal Facial Appearances with Chronic Illnesses, for descriptions of the other responses.)

A 50-year-old patient has been brought to the emergency department after a housemate found that the patient could not get out of bed alone. He has lived in a group home for years but for several months has not participated in the activities and has stayed in his room. The nurse assesses for signs of undernutrition, and an x-ray study reveals that he has osteomalacia, which is a deficiency of: a. Iron. b. Riboflavin. c. Vitamin D and calcium. d. Vitamin C.

C Osteomalacia results from a deficiency of vitamin D and calcium in adults. Iron deficiency would result in anemia, riboflavin deficiency would result in magenta tongue, and vitamin C deficiency would result in scurvy.

Which statement indicates that the nurse understands the pain experienced by an older adult? a. "Older adults must learn to tolerate pain." b. "Pain is a normal process of aging and is to be expected." c. "Pain indicates a pathologic condition or an injury and is not a normal process of aging." d. "Older individuals perceive pain to a lesser degree than do younger individuals."

C Pain indicates a pathologic condition or an injury and should never be considered something that an older adult should expect or tolerate. Pain is not a normal process of aging, and no evidence suggests that pain perception is reduced with aging.

A black patient is in the intensive care unit because of impending shock after an accident. The nurse expects to find what characteristics in this patient's skin? a. Ruddy blue. b. Generalized pallor. c. Ashen, gray, or dull. d. Patchy areas of pallor.

C Pallor attributable to shock, with decreased perfusion and vasoconstriction, in black-skinned people will cause the skin to appear ashen, gray, or dull (see Table 12-2).

While performing a well-child assessment on a 5 year old, the nurse notes the presence of palpable, bilateral, cervical, and inguinal lymph nodes. They are approximately 0.5 cm in size, round, mobile, and nontender. The nurse suspects that this child: a. Has chronic allergies. b. May have an infection. c. Is exhibiting a normal finding for a well child of this age. d. Should be referred for additional evaluation.

C Palpable lymph nodes are normal in children until puberty when the lymphoid tissue begins to atrophy. Lymph nodes may be up to 1 cm in size in the cervical and inguinal areas but are discrete, movable, and nontender.

The nurse is preparing to percuss the abdomen of a patient. The purpose of the percussion is to assess the __________ of the underlying tissue. a. Turgor b. Texture c. Density d. Consistency

C Percussion yields a sound that depicts the location, size, and density of the underlying organ. Turgor and texture are assessed with palpation.

The nurse notices the presence of periorbital edema when performing an eye assessment on a 70-year-old patient. The nurse should: a. Check for the presence of exophthalmos. b. Suspect that the patient has hyperthyroidism. c. Ask the patient if he or she has a history of heart failure. d. Assess for blepharitis, which is often associated with periorbital edema.

C Periorbital edema occurs with local infections, crying, and systemic conditions such as heart failure, renal failure, allergy, and hypothyroidism. Periorbital edema is not associated with blepharitis.

A 70-year-old patient tells the nurse that he has noticed that he is having trouble hearing, especially in large groups. He says that he "can't always tell where the sound is coming from" and the words often sound "mixed up." What might the nurse suspect as the cause for this change? a. Atrophy of the apocrine glands b. Cilia becoming coarse and stiff c. Nerve degeneration in the inner ear d. Scarring of the tympanic membrane

C Presbycusis is a type of hearing loss that occurs in 60% of those older than 65 years of age, even in those living in a quiet environment. This sensorineural loss is gradual and caused by nerve degeneration in the inner ear. Words sound garbled, and the ability to localize sound is also impaired. This communication dysfunction is accentuated when background noise is present.

A 60-year-old man is at the clinic for an eye examination. The nurse suspects that he has ptosis of one eye. How should the nurse check for this? a. Perform the confrontation test. b. Assess the individual's near vision. c. Observe the distance between the palpebral fissures. d. Perform the corneal light test, and look for symmetry of the light reflex.

C Ptosis is a drooping of the upper eyelid that would be apparent by observing the distance between the upper and lower eyelids. The confrontation test measures peripheral vision. Measuring near vision or the corneal light test does not check for ptosis.

A physician has diagnosed a patient with purpura. After leaving the room, a nursing student asks the nurse what the physician saw that led to that diagnosis. The nurse should say, "The physician is referring to the: a. "Blue dilation of blood vessels in a star-shaped linear pattern on the legs." b. "Fiery red, star-shaped marking on the cheek that has a solid circular center." c. "Confluent and extensive patch of petechiae and ecchymoses on the feet." d. "Tiny areas of hemorrhage that are less than 2 mm, round, discrete, and dark red in color."

C Purpura is a confluent and extensive patch of petechiae and ecchymoses and a flat macular hemorrhage observed in generalized disorders such as thrombocytopenia and scurvy. The blue dilation of blood vessels in a star-shaped linear pattern on the legs describes a venous lake. The fiery red, star-shaped marking on the cheek that has a solid circular center describes a spider or star angioma. The tiny areas of hemorrhage that are less than 2 mm, round, discrete, and dark red in color describes petechiae.

7. The nurse should measure rectal temperatures in which of these patients? a. School-age child b. Older adult c. Comatose adult d. Patient receiving oxygen by nasal cannula

C Rectal temperatures should be taken when the other routes are impractical, such as for comatose or confused persons, for those in shock, or for those who cannot close the mouth because of breathing or oxygen tubes, a wired mandible, or other facial dysfunctions.

When examining the nares of a 45-year-old patient who has complaints of rhinorrhea, itching of the nose and eyes, and sneezing, the nurse notices the following: pale turbinates, swelling of the turbinates, and clear rhinorrhea. Which of these conditions is most likely the cause? a. Nasal polyps b. Acute sinusitis c. Allergic rhinitis d. Acute rhinitis

C Rhinorrhea, itching of the nose and eyes, and sneezing are present with allergic rhinitis. On physical examination, serous edema is noted, and the turbinates usually appear pale with a smooth, glistening surface. (See Table 16-1 for descriptions of the other conditions.)

A patient has been admitted for severe psoriasis. The nurse expects to see what finding in the patient's fingernails? a. Splinter hemorrhages b. Paronychia c. Pitting d. Beau lines

C Sharply defined pitting and crumbling of the nails, each with distal detachment characterize pitting nails and are associated with psoriasis. (See Table 12-13 for descriptions of the other terms.)

A patient comes into the clinic complaining of facial pain, fever, and malaise. On examination, the nurse notes swollen turbinates and purulent discharge from the nose. The patient also complains of a dull, throbbing pain in his cheeks and teeth on the right side and pain when the nurse palpates the areas. The nurse recognizes that this patient has: a. Posterior epistaxis. b. Frontal sinusitis. c. Maxillary sinusitis. d. Nasal polyps.

C Signs of maxillary sinusitis include facial pain after upper respiratory infection, red swollen nasal mucosa, swollen turbinates, and purulent discharge. The person also has fever, chills, and malaise. With maxillary sinusitis, dull throbbing pain occurs in the cheeks and teeth on the same side, and pain with palpation is present. With frontal sinusitis, pain is above the supraorbital ridge.

During an oral assessment of a 30-year-old Black patient, the nurse notices bluish lips and a dark line along the gingival margin. What action would the nurse perform in response to this finding? a. Check the patient's hemoglobin for anemia. b. Assess for other signs of insufficient oxygen supply. c. Proceed with the assessment, knowing that this appearance is a normal finding. d. Ask if he has been exposed to an excessive amount of carbon monoxide.

C Some Blacks may have bluish lips and a dark line on the gingival margin; this appearance is a normal finding.

A 35-year-old recent immigrant is being seen in the clinic for complaints of a cough that is associated with rust-colored sputum, low-grade afternoon fevers, and night sweats for the past 2 months. The nurse's preliminary analysis, based on this history, is that this patient may be suffering from: a. Bronchitis. b. Pneumonia. c. Tuberculosis. d. Pulmonary edema.

C Sputum is not diagnostic alone, but some conditions have characteristic sputum production. Tuberculosis often produces rust-colored sputum in addition to other symptoms of night sweats and low-grade afternoon fevers (see Table 18-8).

Which of these statements is true regarding the use of Standard Precautions in the health care setting? a. Standard Precautions apply to all body fluids, including sweat. b. Use alcohol-based hand rub if hands are visibly dirty. c. Standard Precautions are intended for use with all patients, regardless of their risk or presumed infection status. d. Standard Precautions are to be used only when nonintact skin, excretions containing visible blood, or expected contact with mucous membranes is present.

C Standard Precautions are designed to reduce the risk of transmission of microorganisms from both recognized and unrecognized sources and are intended for use for all patients, regardless of their risk or presumed infection status. Standard Precautions apply to blood and all other body fluids, secretions and excretions except sweat—regardless of whether they contain visible blood, nonintact skin, or mucous membranes. Hands should be washed with soap and water if visibly soiled with blood or body fluids. Alcohol-based hand rubs can be used if hands are not visibly soiled.

When examining a patient's eyes, the nurse recalls that stimulation of the sympathetic branch of the autonomic nervous system: a. Causes pupillary constriction. b. Adjusts the eye for near vision. c. Elevates the eyelid and dilates the pupil. d. Causes contraction of the ciliary body.

C Stimulation of the sympathetic branch of the autonomic nervous system dilates the pupil and elevates the eyelid. Parasympathetic nervous system stimulation causes the pupil to constrict. The muscle fibers of the iris contract the pupil in bright light to accommodate for near vision. The ciliary body controls the thickness of the lens.

The nurse is assessing color vision of a male child. Which statement is correct? The nurse should: a. Check color vision annually until the age of 18 years. b. Ask the child to identify the color of his or her clothing. c. Test for color vision once between the ages of 4 and 8 years. d. Begin color vision screening at the child's 2-year checkup.

C Test boys only once for color vision between the ages of 4 and 8 years. Color vision is not tested in girls because it is rare in girls. Testing is performed with the Ishihara test, which is a series of polychromatic cards.

A physician tells the nurse that a patient's vertebra prominens is tender and asks the nurse to reevaluate the area in 1 hour. The area of the body the nurse will assess is: a. Just above the diaphragm. b. Just lateral to the knee cap. c. At the level of the C7 vertebra. d. At the level of the T11 vertebra.

C The C7 vertebra has a long spinous process, called the vertebra prominens, which is palpable when the head is flexed.

The nurse is preparing to assess the visual acuity of a 16-year-old patient. How should the nurse proceed? a. Perform the confrontation test. b. Ask the patient to read the print on a handheld Jaeger card. c. Use the Snellen chart positioned 20 feet away from the patient. d. Determine the patient's ability to read newsprint at a distance of 12 to 14 inches.

C The Snellen alphabet chart is the most commonly used and most accurate measure of visual acuity. The confrontation test is a gross measure of peripheral vision. The Jaeger card or newspaper tests are used to test near vision.

The nurse is taking temperatures in a clinic with a TMT. Which statement is true regarding use of the TMT? a. A tympanic temperature is more time consuming than a rectal temperature. b. The tympanic method is more invasive and uncomfortable than the oral method. c. The risk of cross-contamination is reduced, compared with the rectal route. d. The tympanic membrane most accurately reflects the temperature in the ophthalmic artery.

C The TMT is a noninvasive, nontraumatic device that is extremely quick and efficient. The chance of cross-contamination with the TMT is minimal because the ear canal is lined with skin, not mucous membranes.

A 2-year-old child has been brought to the clinic for a well-child checkup. The best way for the nurse to begin the assessment is to: a. Ask the parent to place the child on the examining table. b. Have the parent remove all of the child's clothing before the examination. c. Allow the child to keep a security object such as a toy or blanket during the examination. d. Initially focus the interactions on the child, essentially ignoring the parent until the child's trust has been obtained.

C The best place to examine the toddler is on the parent's lap. Toddlers understand symbols; therefore, a security object is helpful. Initially, the focus is more on the parent, which allows the child to adjust gradually and to become familiar with you. A 2-year-old child does not like to take off his or her clothes. Therefore, ask the parent to undress one body part at a time.

During an assessment, the nurse notices that an older adult patient has tears rolling down his face from his left eye. Closer examination shows that the lower lid is loose and rolling outward. The patient complains of his eye feeling "dry and itchy." Which action by the nurse is correct? a. Assessing the eye for a possible foreign body b. Documenting the finding as ptosis c. Assessing for other signs of ectropion d. Contacting the prescriber; these are signs of basal cell carcinoma

C The condition described is known as ectropion, and it occurs in older adults and is attributable to atrophy of the elastic and fibrous tissues. The lower lid does not approximate to the eyeball, and, as a result, the puncta cannot effectively siphon tears; excessive tearing results. Ptosis is a drooping of the upper eyelid. These signs do not suggest the presence of a foreign body in the eye or basal cell carcinoma.

The nurse is examining a patient's ears and notices cerumen in the external canal. Which of these statements about cerumen is correct? a. Sticky honey-colored cerumen is a sign of infection. b. The presence of cerumen is indicative of poor hygiene. c. The purpose of cerumen is to protect and lubricate the ear. d. Cerumen is necessary for transmitting sound through the auditory canal.

C The ear is lined with glands that secrete cerumen, which is a yellow waxy material that lubricates and protects the ear.

A mother is concerned because her 18-month-old toddler has 12 teeth. She is wondering if this is normal for a child of this age. The nurse's best response would be: a. "How many teeth did you have at this age?" b. "All 20 deciduous teeth are expected to erupt by age 4 years." c. "This is a normal number of teeth for an 18 month old." d. "Normally, by age 2 years, 16 deciduous teeth are expected."

C The guidelines for the number of teeth for children younger than 2 years old are as follows: the child's age in months minus the number 6 should be equal to the expected number of deciduous teeth. Normally, all 20 teeth are in by 2 years old. In this instance, the child is 18 months old, minus 6, equals 12 deciduous teeth expected.

When assessing the force, or strength, of a pulse, the nurse recalls that the pulse: a. Is usually recorded on a 0- to 2-point scale. b. Demonstrates elasticity of the vessel wall. c. Is a reflection of the heart's stroke volume. d. Reflects the blood volume in the arteries during diastole.

C The heart pumps an amount of blood (the stroke volume) into the aorta. The force flares the arterial walls and generates a pressure wave, which is felt in the periphery as the pulse.

The projections in the nasal cavity that increase the surface area are called the: a. Meatus. b. Septum. c. Turbinates. d. Kiesselbach plexus.

C The lateral walls of each nasal cavity contain three parallel bony projections: the superior, middle, and inferior turbinates. These increase the surface area, making more blood vessels and mucous membrane available to warm, humidify, and filter the inhaled air.

The most important step that the nurse can take to prevent the transmission of microorganisms in the hospital setting is to: a. Wear protective eye wear at all times. b. Wear gloves during any and all contact with patients. c. Wash hands before and after contact with each patient. d. Clean the stethoscope with an alcohol swab between patients.

C The most important step to decrease the risk of microorganism transmission is to wash hands promptly and thoroughly before and after physical contact with each patient. Stethoscopes should also be cleansed with an alcohol swab before and after each patient contact. The best routine is to combine stethoscope rubbing with hand hygiene each time hand hygiene is performed.

A newborn infant is in the clinic for a well-baby checkup. The nurse observes the infant for the possibility of fluid loss because of which of these factors? a. Subcutaneous fat deposits are high in the newborn. b. Sebaceous glands are overproductive in the newborn. c. The newborn's skin is more permeable than that of the adult. d. The amount of vernix caseosa dramatically rises in the newborn.

C The newborn's skin is thin, smooth, and elastic and is relatively more permeable than that of the adult; consequently, the infant is at greater risk for fluid loss. The subcutaneous layer in the infant is inefficient, not thick, and the sebaceous glands are present but decrease in size and production. Vernix caseosa is not produced after birth.

The nurse notices that the mother of a 2-year-old boy brings him into the clinic quite frequently for various injuries and suspects there may be some child abuse involved. During an inspection of his mouth, the nurse should look for: a. Swollen, red tonsils. b. Ulcerations on the hard palate. c. Bruising on the buccal mucosa or gums. d. Small yellow papules along the hard palate.

C The nurse should notice any bruising or laceration on the buccal mucosa or gums of an infant or young child. Trauma may indicate child abuse from a forced feeding of a bottle or spoon.

A 65-year-old patient with a history of heart failure comes to the clinic with complaints of "being awakened from sleep with shortness of breath." Which action by the nurse is most appropriate? a. Obtaining a detailed health history of the patient's allergies and a history of asthma b. Telling the patient to sleep on his or her right side to facilitate ease of respirations c. Assessing for other signs and symptoms of paroxysmal nocturnal dyspnea d. Assuring the patient that paroxysmal nocturnal dyspnea is normal and will probably resolve within the next week

C The patient is experiencing paroxysmal nocturnal dyspnea—being awakened from sleep with shortness of breath and the need to be upright to achieve comfort.

A 50-year-old woman with elevated total cholesterol and triglyceride levels is visiting the clinic to find out about her laboratory results. What would be important for the nurse to include in patient teaching in relation to these tests? a. The risks of undernutrition should be included. b. Offer methods to reduce the stress in her life. c. Provide information regarding a diet low in saturated fat. d. This condition is hereditary; she can do nothing to change the levels.

C The patient with elevated cholesterol and triglyceride levels should be taught about eating a healthy diet that limits the intake of foods high in saturated fats or trans fats. Reducing dietary fats is part of the treatment for this condition. The other responses are not pertinent to her condition.

The nurse is performing an otoscopic examination on an adult. Which of these actions is correct? a. Tilting the person's head forward during the examination b. Once the speculum is in the ear, releasing the traction c. Pulling the pinna up and back before inserting the speculum d. Using the smallest speculum to decrease the amount of discomfort

C The pinna is pulled up and back on an adult or older child, which helps straighten the S-shape of the canal. Traction should not be released on the ear until the examination is completed and the otoscope is removed.

When a light is directed across the iris of a patient's eye from the temporal side, the nurse is assessing for: a. Drainage from dacryocystitis. b. Presence of conjunctivitis over the iris. c. Presence of shadows, which may indicate glaucoma. d. Scattered light reflex, which may be indicative of cataracts.

C The presence of shadows in the anterior chamber may be a sign of acute angle-closure glaucoma. The normal iris is flat and creates no shadows. This method is not correct for the assessment of dacryocystitis, conjunctivitis, or cataracts.

When performing a respiratory assessment on a patient, the nurse notices a costal angle of approximately 90 degrees. This characteristic is: a. Observed in patients with kyphosis. b. Indicative of pectus excavatum. c. A normal finding in a healthy adult. d. An expected finding in a patient with a barrel chest.

C The right and left costal margins form an angle where they meet at the xiphoid process. Usually, this angle is 90 degrees or less. The angle increases when the rib cage is chronically overinflated, as in emphysema.

The nurse is assessing an 80-year-old patient. Which of these findings would be expected for this patient? a. Hypertrophy of the gums b. Increased production of saliva c. Decreased ability to identify odors d. Finer and less prominent nasal hair

C The sense of smell may be reduced because of a decrease in the number of olfactory nerve fibers. Nasal hairs grow coarser and stiffer with aging. The gums may recede with aging, not hypertrophy, and saliva production decreases.

During the examination, offering some brief teaching about the patient's body or the examiner's findings is often appropriate. Which one of these statements by the nurse is most appropriate? a. "Your atrial dysrhythmias are under control." b. "You have pitting edema and mild varicosities." c. "Your pulse is 80 beats per minute, which is within the normal range." d. "I'm using my stethoscope to listen for any crackles, wheezes, or rubs."

C The sharing of some information builds rapport, as long as the patient is able to understand the terminology.

A patient's blood pressure is 118/82 mm Hg. He asks the nurse, "What do the numbers mean?" The nurse's best reply is: a. "The numbers are within the normal range and are nothing to worry about." b. "The bottom number is the diastolic pressure and reflects the stroke volume of the heart." c. "The top number is the systolic blood pressure and reflects the pressure of the blood against the arteries when the heart contracts." d. "The concept of blood pressure is difficult to understand. The primary thing to be concerned about is the top number, or the systolic blood pressure."

C The systolic pressure is the maximum pressure felt on the artery during left ventricular contraction, or systole. The diastolic pressure is the elastic recoil, or resting, pressure that the blood constantly exerts in between each contraction. The nurse should answer the patient's question and use terms he can understand.

The nurse needs to palpate the temporomandibular joint for crepitation. This joint is located just below the temporal artery and anterior to the: a. Hyoid bone. b. Vagus nerve. c. Tragus. d. Mandible.

C The temporomandibular joint is just below the temporal artery and anterior to the tragus.

The nurse is unable to identify any changes in sound when percussing over the abdomen of an obese patient. What should the nurse do next? a. Ask the patient to take deep breaths to relax the abdominal musculature. b. Consider this finding as normal, and proceed with the abdominal assessment. c. Increase the amount of strength used when attempting to percuss over the abdomen. d. Decrease the amount of strength used when attempting to percuss over the abdomen.

C The thickness of the person's body wall will be a factor. The nurse needs a stronger percussion stroke for persons with obese or very muscular body walls. The force of the blow determines the loudness of the note. The other actions are not correct.

During an examination of a patient in her third trimester of pregnancy, the nurse notices that the patient's thyroid gland is slightly enlarged. No enlargement had been previously noticed. The nurse suspects that the patient: a. Has an iodine deficiency. b. Is exhibiting early signs of goiter. c. Is exhibiting a normal enlargement of the thyroid gland during pregnancy. d. Needs further testing for possible thyroid cancer.

C The thyroid gland enlarges slightly during pregnancy because of hyperplasia of the tissue and increased vascularity.

During an examination, the patient states he is hearing a buzzing sound and says that it is "driving me crazy!" The nurse recognizes that this symptom indicates: a. Vertigo. b. Pruritus. c. Tinnitus. d. Cholesteatoma.

C Tinnitus is a sound that comes from within a person; it can be a ringing, crackling, or buzzing sound. It accompanies some hearing or ear disorders.

The nurse is preparing to measure the length, weight, chest, and head circumference of a 6-month-old infant. Which measurement technique is correct? a. Measuring the infant's length by using a tape measure b. Weighing the infant by placing him or her on an electronic standing scale c. Measuring the chest circumference at the nipple line with a tape measure d. Measuring the head circumference by wrapping the tape measure over the nose and cheekbones

C To measure the chest circumference, the tape is encircled around the chest at the nipple line. The length should be measured on a horizontal measuring board. Weight should be measured on a platform-type balance scale. Head circumference is measured with the tape around the head, aligned at the eyebrows, and at the prominent frontal and occipital bones—the widest span is correct

When assessing the pupillary light reflex, the nurse should use which technique? a. Shine a penlight from directly in front of the patient, and inspect for pupillary constriction. b. Ask the patient to follow the penlight in eight directions, and observe for bilateral pupil constriction. c. Shine a light across the pupil from the side, and observe for direct and consensual pupillary constriction. d. Ask the patient to focus on a distant object. Then ask the patient to follow the penlight to approximately 7 cm from the nose.

C To test the pupillary light reflex, the nurse should advance a light in from the side and note the direct and consensual pupillary constriction.

When examining the face of a patient, the nurse is aware that the two pairs of salivary glands that are accessible to examination are the ___________ and ___________ glands. a. Occipital; submental b. Parotid; jugulodigastric c. Parotid; submandibular d. Submandibular; occipital

C Two pairs of salivary glands accessible to examination on the face are the parotid glands, which are in the cheeks over the mandible, anterior to and below the ear; and the submandibular glands, which are beneath the mandible at the angle of the jaw. The parotid glands are normally nonpalpable

In a patient who has anisocoria, the nurse would expect to observe: a. Dilated pupils. b. Excessive tearing. c. Pupils of unequal size. d. Uneven curvature of the lens.

C Unequal pupil size is termed anisocoria. It normally exists in 5% of the population but may also be indicative of central nervous system disease.

A patient has been diagnosed with strep throat. The nurse is aware that without treatment, which complication may occur? a. Rubella b. Leukoplakia c. Rheumatic fever d. Scarlet fever

C Untreated strep throat may lead to rheumatic fever. When performing a health history, the patient should be asked whether his or her sore throat has been documented as streptococcal.

When auscultating the lungs of an adult patient, the nurse notes that low-pitched, soft breath sounds are heard over the posterior lower lobes, with inspiration being longer than expiration. The nurse interprets that these sounds are: a. Normally auscultated over the trachea. b. Bronchial breath sounds and normal in that location. c. Vesicular breath sounds and normal in that location. d. Bronchovesicular breath sounds and normal in that location.

C Vesicular breath sounds are low-pitched, soft sounds with inspiration being longer than expiration. These breath sounds are expected over the peripheral lung fields where air flows through smaller bronchioles and alveoli.

The nurse is helping another nurse to take a blood pressure reading on a patient's thigh. Which action is correct regarding thigh pressure? a. Either the popliteal or femoral vessels should be auscultated to obtain a thigh pressure. b. The best position to measure thigh pressure is the supine position with the knee slightly bent. c. If the blood pressure in the arm is high in an adolescent, then it should be compared with the thigh pressure. d. The thigh pressure is lower than the pressure in the arm, which is attributable to the distance away from the heart and the size of the popliteal vessels.

C When blood pressure measured at the arm is excessively high, particularly in adolescents and young adults, it is compared with thigh pressure to check for coarctation of the aorta. The popliteal artery is auscultated for the reading. Generally, thigh pressure is higher than that of the arm; however, if coarctation of the artery is present, then arm pressures are higher than thigh pressures.

How should the nurse perform a triceps skinfold assessment? a. After pinching the skin and fat, the calipers are vertically applied to the fat fold. b. The skin and fat on the front of the patient's arm are gently pinched, and then the calipers are applied. c. After applying the calipers, the nurse waits 3 seconds before taking a reading. After repeating the procedure three times, an average is recorded. d. The patient is instructed to stand with his or her back to the examiner and arms folded across the chest. The skin on the forearm is pinched.

C While holding the skinfold, the lever of the calipers is released. The nurse waits 3 seconds and then takes a reading. This procedure should be repeated three times, and an average of the three skinfold measurements is then recorded.

During an examination, the nurse finds that a patient's left temporal artery is tortuous and feels hardened and tender, compared with the right temporal artery. The nurse suspects which condition? a. Crepitation b. Mastoiditis c. Temporal arteritis d. Bell palsy

C With temporal arteritis, the artery appears more tortuous and feels hardened and tender. These assessment findings are not consistent with the other responses.

The respiratory rate of a newborn infant is expected to range from _____ breaths/min. a. 10 to 20 b. 20 to 30 c. 40 to 60 d. 30 to 80 e. greater than 80

C - 40 to 60

Which nail change found on examination would be most alarming? a. Dark bands seen in all fingernails of a dark-skinned person b. A yellow discoloration of the great toe of an older adult c. A single blue nail d. Pits in both index fingernails of an adult e. Longitudinal ridges in an older adult

C - A single blue nail

Which site of chest wall retractions indicates a more severe obstruction in a patient with asth a. Lower chest b. Along the anterior axillary line c. Above the clavicles d. At the nipple line e. Along the posterior axillary line

C - Above the clavicles

Tender nodes associated with cat scratch disease are most commonly found in which area? a. Epitrochlear b. Popliteal c. Axilla d. Inguinal e. Supraclavicular

C - Axilla

Which lymphatic tissue is normally visible during the physical examination? a. Adenoids b. Peyer patches c. Palatine tonsils d. Thymus e. Submandibular

C - Palatine tonsils The palatine tonsils are located at the back of the mouth on either side of the tongue and can adenoids. Submandibular nodes may be palpable but are not visible. Peyer patches are loca thymus is in the chest.

In which patient situation would you expect to assess tachypnea? a. Patient with depression b. Patient who abuses narcotics c. Patient with metabolic acidosis d. Patient with myasthenia gravis e. Patient with metabolic alkalosis

C - Patient with metabolic acidosis

Which finding suggests a minor structural variation? a. Barrel chest b. Clubbed fingers c. Pectus carinatum d. Retractions e. Tachypnea

C - Pectus carinatum

The patient that you are examining is complaining of pain near the spine. While palpating the the inferior border of the right scapula, the patient feels more intense pain. When viewing the at which rib? a. Right sixth rib b. Right seventh rib c. Right eighth rib d. Left seventh rib e. Left eighth rib

C - Right eighth rib

Which bronchial structure(s) is (are) most susceptible to aspiration of foreign bodies? a. Left mainstem bronchus b. Terminal bronchioles c. Right mainstem bronchus d. Right respiratory bronchioles e. Left respiratory bronchioles

C - Right mainstem bronchus

Which decubitus ulcer stage indicates damage into the subcutaneous tissue? a. Stage I b. Stage II c. Stage III d. Stage IV e. Stage V

C - Stage III Stage III describes damage through to the subcutaneous tissue.

Which landmark is the dividing line between the anterior and posterior cervical triangles? a. Clavicle b. Cervical spine c. Sternocleidomastoid d. Sternum e. Hyoid bone

C - Sternocleidomastoid The landmark dividing the anterior and posterior cervical triangles is the sternocleidomastoid

How is the sputum of a viral infection different from the sputum of a bacterial infection? a. There is more sputum production with viral conditions than bacterial infections. b. The sputum is odorous with viral conditions and nonodorous with bacterial infections. c. The sputum is yellow, green, or rust colored with bacterial infections and mucoid with vira d. The sputum is much thinner with bacterial infections and viscid with viral. e. Viral pneumonia sputum is never blood streaked.

C - The sputum is yellow, green, or rust colored with bacterial infections and mucoid w The more likely differentiating characteristic between viral and bacterial sputum is the color. produce mucoid sputum, bacterial infections produce yellow, green, or rust-colored sputum.

The best time to observe and count respirations is a. while the patient is answering questions. b. while weighing the patient. c. after palpating the pulse. d. when the patient is sleeping. e. after a short walk.

C - after palpating the pulse.

Skin lesions are transilluminated to determine a. vascular from nonvascular lesions. b. furuncles from folliculitis lesions. c. fluid-filled lesions in solid cysts or masses. d. herpes zoster from varicella. e. macules from papules.

C - fluid-filled lesions in solid cysts or masses

Expected hair distribution changes in the older adults include a. increased terminal hair follicles to the scalp. b. more prominent axillary and pubic hair production. c. increased terminal hair follicles to the tragus of men's ears. d. more prominent peripheral extremity hair production. e. women possibly developing less coarse facial hair.

C - increased terminal hair follicles to the tragus of men's ears.

Sweat glands, hair, and nails are all formed from a. basement membranes under cellular strata. b. closely packed squamous cells. c. invaginations of epidermis into dermis. d. papillae that penetrate the epidermis. e. evaginations of the hypodermis.

C - invaginations of epidermis into dermis

The harder and more discrete a node, the more likely a(n) a. innocent cause. b. infection. c. malignancy. d. metabolic disease. e. drug reaction.

C - malignancy

Enlarged tonsils and adenoids may obstruct the a. thoracic duct. b. esophagus. c. nasopharyngeal passageway. d. external auditory meatus. e. oral cavity.

C - nasopharyngeal passageway. The palatine tonsils are located on either side of the pharynx, and the adenoids (pharyngeal t the pharynx, superior to the soft palate. If these structures become enlarged, they block the nasal cavity.

Bronchovesicular breath sounds in young children that are loud and harsh are an indication o a. an accumulation of fluid. b. malignant tumors or solid masses. c. normal, thin chest wall structures. d. pus-filled abscesses and tumors. e. tension pneumothorax.

C - normal, thin chest wall structures.

Nodes located at the base of the skull are the _____ nodes. a. preauricular b. postauricular c. occipital d. epitrochlear e. parotid

C - occipital

Obstruction of the nasopharynx, pulmonary hypertension, and risk of sleep apnea may be as a. branchial cleft cysts. b. cystic hygromas. c. palatine tonsils. d. thyroglossal duct cysts. e. lymphangiomas.

C - palatine tonsils

You would expect to document the presence of a pleural friction rub for a patient being treate a. bronchitis. b. atelectasis. c. pleurisy. d. emphysema. e. pneumonia.

C - pleurisy.

Dullness to percussion in intercostal spaces is most consistent with the presence of a. asthma. b. empyema. c. pneumonia. d. sickle cell disease. e. pneumothorax.

C - pneumonia

Skin appendages are formed embryonically when the epidermis invaginates into the dermis. The secretory activity of the sebaceous glands is stimulated by a. body heat. b. ambient temperature. c. sex hormones. d. dietary protein. e. emotional stimuli.

C - sex hormones.

Lymph ducts merge into the venous system at the a. portal vein. b. pulmonic vein. c. subclavian veins. d. vena cava. e. jugular veins.

C - subclavian veins. The large ducts of the lymphatics merge into the venous system at the left and right subclav

Café au lait patches are numbered with each assessment of infants and young children beca a. their numbers are expected to increase each year. b. coalescent lesions are a more serious finding. c. the presence of more than six patches suggests neurofibromatosis. d. decreasing numbers are expected with growth. e. a higher number correlates with the development of erythema toxicum

C - the presence of more than six patches suggests neurofibromatosis.

An organ that is essential to the development of protective immune function in infants but ha adults is the a. spleen. b. appendix. c. thymus. d. pancreas. e. liver.

C - thymus. In adults, the thymus atrophies, and in older adults, it is replaced by fat and connective tissu

Serum sickness is usually characterized first by the appearance of: a. diffuse lymph node enlargement. b. joint pain. c. urticaria. d. fever. e. a single enlarged posterior cervical node.

C - urticaria.

Unusual white areas on the skin may be caused by: a. adrenal disease. b. polycythemia. c. vitiligo. d. Down syndrome. e. lentigo.

C - vitiligo

When assessing a patient's nutritional status, the nurse recalls that the best definition of optimal nutritional status is sufficient nutrients that: a. Are in excess of daily body requirements. b. Provide for the minimum body needs. c. Provide for daily body requirements but do not support increased metabolic demands. d. Provide for daily body requirements and support increased metabolic demands.

D Optimal nutritional status is achieved when sufficient nutrients are consumed to support day-to-day body needs and any increased metabolic demands resulting from growth, pregnancy, or illness.

A 60-year-old woman has developed reflexive sympathetic dystrophy after arthroscopic repair of her shoulder. A key feature of this condition is that the: a. Affected extremity will eventually regain its function. b. Pain is felt at one site but originates from another location. c. Patient's pain will be associated with nausea, pallor, and diaphoresis. d. Slightest touch, such as a sleeve brushing against her arm, causes severe and intense pain.

D A key feature of reflexive sympathetic dystrophy is that a typically innocuous stimulus can create a severe, intensely painful response. The affected extremity becomes less functional over time.

A mother brings her 4-month-old infant to the clinic with concerns regarding a small pad in the middle of the upper lip that has been there since 1 month of age. The infant has no health problems. On physical examination, the nurse notices a 0.5-cm, fleshy, elevated area in the middle of the upper lip. No evidence of inflammation or drainage is observed. What would the nurse tell this mother? a. "This area of irritation is caused from teething and is nothing to worry about." b. "This finding is abnormal and should be evaluated by another health care provider." c. "This area of irritation is the result of chronic drooling and should resolve within the next month or two." d. "This elevated area is a sucking tubercle caused from the friction of breastfeeding or bottle-feeding and is normal."

D A normal finding in infants is the sucking tubercle, a small pad in the middle of the upper lip from the friction of breastfeeding or bottle-feeding. This condition is not caused by irritation, teething, or excessive drooling, and evaluation by another health care provider is not warranted.

The nurse notices that a patient has a solid, elevated, circumscribed lesion that is less than 1 cm in diameter. When documenting this finding, the nurse reports this as a: a. Bulla. b. Wheal. c. Nodule. d. Papule.

D A papule is something one can feel, is solid, elevated, circumscribed, less than 1 cm in diameter, and is due to superficial thickening in the epidermis. A bulla is larger than 1 cm, superficial, and thin walled. A wheal is superficial, raised, transient, erythematous, and irregular in shape attributable to edema. A nodule is solid, elevated, hard or soft, and larger than 1 cm.

When measuring a patient's weight, the nurse is aware of which of these guidelines? a. The patient is always weighed wearing only his or her undergarments. b. The type of scale does not matter, as long as the weights are similar from day to day. c. The patient may leave on his or her jacket and shoes as long as these are documented next to the weight. d. Attempts should be made to weigh the patient at approximately the same time of day, if a sequence of weights is necessary.

D A standardized balance scale is used to measure weight. The patient should remove his or her shoes and heavy outer clothing. If a sequence of repeated weights is necessary, then the nurse should attempt to weigh the patient at approximately the same time of day and with the same types of clothing worn each time.

A patient is complaining of severe knee pain after twisting it during a basketball game and is requesting pain medication. Which action by the nurse is appropriate? a. Completing the physical examination first and then giving the pain medication b. Telling the patient that the pain medication must wait until after the x-ray images are completed c. Evaluating the full range of motion of the knee and then medicating for pain d. Administering pain medication and then proceeding with the assessment

D According to the American Pain Society (1992), "In cases in which the cause of acute pain is uncertain, establishing a diagnosis is a priority, but symptomatic treatment of pain should be given while the investigation is proceeding. With occasional exceptions, (e.g., the initial examination of the patient with an acute condition of the abdomen), it is rarely justified to defer analgesia until a diagnosis is made. In fact, a comfortable patient is better able to cooperate with diagnostic procedures."

When assessing a 75-year-old patient who has asthma, the nurse notes that he assumes a tripod position, leaning forward with arms braced on the chair. On the basis of this observation, the nurse should: a. Assume that the patient is eager and interested in participating in the interview. b. Evaluate the patient for abdominal pain, which may be exacerbated in the sitting position. c. Assume that the patient is having difficulty breathing and assist him to a supine position. d. Recognize that a tripod position is often used when a patient is having respiratory difficulties.

D Assuming a tripod position—leaning forward with arms braced on chair arms—occurs with chronic pulmonary disease. The other actions or assumptions are not correct.

The nurse is preparing to assess a hospitalized patient who is experiencing significant shortness of breath. How should the nurse proceed with the assessment? a. The patient should lie down to obtain an accurate cardiac, respiratory, and abdominal assessment. b. A thorough history and physical assessment information should be obtained from the patient's family member. c. A complete history and physical assessment should be immediately performed to obtain baseline information. d. Body areas appropriate to the problem should be examined and then the assessment completed after the problem has resolved.

D Both altering the position of the patient during the examination and collecting a mini database by examining the body areas appropriate to the problem may be necessary in this situation. An assessment may be completed later after the distress is resolved.

A 65-year-old man with emphysema and bronchitis has come to the clinic for a follow-up appointment. On assessment, the nurse might expect to see which finding? a. Anasarca b. Scleroderma c. Pedal erythema d. Clubbing of the nails

D Clubbing of the nails occurs with congenital cyanotic heart disease and neoplastic and pulmonary diseases. The other responses are assessment findings not associated with pulmonary diseases.

The nurse is preparing to measure fat and lean body mass and bone mineral density. Which tool is appropriate? a. Measuring tape b. Skinfold calipers c. Bioelectrical impedance analysis (BIA) d. Dual-energy x-ray absorptiometry (DEXA)

D DEXA measures both bone mineral density and fat and lean body mass. BIA measures fat and lean body mass but not bone mineral density. A measuring tape measures distance or length, and skinfold calipers are used to determine skinfold thickness.

A patient has been admitted to the hospital with vertebral fractures related to osteoporosis. She is in extreme pain. This type of pain would be classified as: a. Referred. b. Cutaneous. c. Visceral. d. Deep somatic.

D Deep somatic pain comes from sources such as the blood vessels, joints, tendons, muscles, and bone. Referred pain is felt at one site but originates from another location. Cutaneous pain is derived from the skin surface and subcutaneous tissues. Visceral pain originates from the larger, interior organs.

A 52-year-old patient describes the presence of occasional floaters or spots moving in front of his eyes. The nurse should: a. Examine the retina to determine the number of floaters. b. Presume the patient has glaucoma and refer him for further testing. c. Consider these to be abnormal findings, and refer him to an ophthalmologist. d. Know that floaters are usually insignificant and are caused by condensed vitreous fibers.

D Floaters are a common sensation with myopia or after middle age and are attributable to condensed vitreous fibers. Floaters or spots are not usually significant, but the acute onset of floaters may occur with retinal detachment.

A 32-year-old woman is at the clinic for "little white bumps in my mouth." During the assessment, the nurse notes that she has a 0.5 cm white, nontender papule under her tongue and one on the mucosa of her right cheek. What would the nurse tell the patient? a. "These spots indicate an infection such as strep throat." b. "These bumps could be indicative of a serious lesion, so I will refer you to a specialist." c. "This condition is called leukoplakia and can be caused by chronic irritation such as with smoking." d. "These bumps are Fordyce granules, which are sebaceous cysts and are not a serious condition."

D Fordyce granules are small, isolated white or yellow papules on the mucosa of the cheek, tongue, and lips. These little sebaceous cysts are painless and are not significant. Chalky, white raised patches would indicate leukoplakia. In strep throat, the examiner would see tonsils that are bright red, swollen, and may have exudates or white spots.

An assessment of a 23-year-old patient reveals the following: an auricle that is tender and reddish-blue in color with small vesicles. The nurse would need to know additional information that includes which of these? a. Any change in the ability to hear b. Any recent drainage from the ear c. Recent history of trauma to the ear d. Any prolonged exposure to extreme cold

D Frostbite causes reddish-blue discoloration and swelling of the auricle after exposure to extreme cold. Vesicles or bullae may develop, and the person feels pain and tenderness.

During auscultation of a patient's heart sounds, the nurse hears an unfamiliar sound. The nurse should: a. Document the findings in the patient's record. b. Wait 10 minutes, and auscultate the sound again. c. Ask the patient how he or she is feeling. d. Ask another nurse to double check the finding.

D If an abnormal finding is not familiar, then the nurse may ask another examiner to double check the finding. The other responses do not help identify the unfamiliar sound.

Which disorder is characterized by a single node that is chronically enlarged and nontender i a. Retropharyngeal abscess b. Streptococcal pharyngitis c. Mononucleosis d. Toxoplasmosis e. Herpes simplex

D - Toxoplasmosis

Fluorescing lesions are best distinguished using a(n) a. incandescent lamp. b. magnifying glass. c. transilluminator. d. Wood's lamp. e. halogen lamp.

D - Wood's lamp.

A patient is unable to read even the largest letters on the Snellen chart. The nurse should take which action next? a. Refer the patient to an ophthalmologist or optometrist for further evaluation. b. Assess whether the patient can count the nurse's fingers when they are placed in front of his or her eyes. c. Ask the patient to put on his or her reading glasses and attempt to read the Snellen chart again. d. Shorten the distance between the patient and the chart until the letters are seen, and record that distance.

D If the person is unable to see even the largest letters when standing 20 feet from the chart, then the nurse should shorten the distance to the chart until the letters are seen, and record that distance (e.g., "10/200"). If visual acuity is even lower, then the nurse should assess whether the person can count fingers when they are spread in front of the eyes or can distinguish light perception from a penlight. If vision is poorer than 20/30, then a referral to an ophthalmologist or optometrist is necessary, but the nurse must first assess the visual acuity.

The nurse is assessing the lungs of an older adult. Which of these changes are normal in the respiratory system of the older adult? a. Severe dyspnea is experienced on exertion, resulting from changes in the lungs. b. Respiratory muscle strength increases to compensate for a decreased vital capacity. c. Decrease in small airway closure occurs, leading to problems with atelectasis. d. Lungs are less elastic and distensible, which decreases their ability to collapse and recoil.

D In the aging adult, the lungs are less elastic and distensible, which decreases their ability to collapse and recoil. Vital capacity is decreased, and a loss of intra-alveolar septa occurs, causing less surface area for gas exchange. The lung bases become less ventilated, and the older person is at risk for dyspnea with exertion beyond his or her usual workload.

When inspecting the anterior chest of an adult, the nurse should include which assessment? a. Diaphragmatic excursion b. Symmetric chest expansion c. Presence of breath sounds d. Shape and configuration of the chest wall

D Inspection of the anterior chest includes shape and configuration of the chest wall; assessment of the patient's level of consciousness and the patient's skin color and condition; quality of respirations; presence or absence of retraction and bulging of the intercostal spaces; and use of accessory muscles. Symmetric chest expansion is assessed by palpation. Diaphragmatic excursion is assessed by percussion of the posterior chest. Breath sounds are assessed by auscultation.

The nurse is preparing to assess a patient's abdomen by palpation. How should the nurse proceed? a. Palpation of reportedly "tender" areas are avoided because palpation in these areas may cause pain. b. Palpating a tender area is quickly performed to avoid any discomfort that the patient may experience. c. The assessment begins with deep palpation, while encouraging the patient to relax and to take deep breaths. d. The assessment begins with light palpation to detect surface characteristics and to accustom the patient to being touched.

D Light palpation is initially performed to detect any surface characteristics and to accustom the person to being touched. Tender areas should be palpated last, not first.

A father brings in his 2-month-old infant to the clinic because the infant has had diarrhea for the last 24 hours. He says his baby has not been able to keep any formula down and that the diarrhea has been at least every 2 hours. The nurse suspects dehydration. The nurse should test skin mobility and turgor over the infant's: a. Sternum. b. Forehead. c. Forearms. d. Abdomen.

D Mobility and turgor are tested over the abdomen in an infant. Poor turgor, or tenting, indicates dehydration or malnutrition. The other sites are not appropriate for checking skin turgor in an infant.

During ocular examinations, the nurse keeps in mind that movement of the extraocular muscles is: a. Decreased in the older adult. b. Impaired in a patient with cataracts. c. Stimulated by cranial nerves (CNs) I and II. d. Stimulated by CNs III, IV, and VI.

D Movement of the extraocular muscles is stimulated by three CNs: III, IV, and VI.

The nurse is reviewing the principles of pain. Which type of pain is due to an abnormal processing of the pain impulse through the peripheral or central nervous system? a. Visceral b. Referred c. Cutaneous d. Neuropathic

D Neuropathic pain implies an abnormal processing of the pain message. The other types of pain are named according to their sources.

During an examination of the eye, the nurse would expect what normal finding when assessing the lacrimal apparatus? a. Presence of tears along the inner canthus b. Blocked nasolacrimal duct in a newborn infant c. Slight swelling over the upper lid and along the bony orbit if the individual has a cold d. Absence of drainage from the puncta when pressing against the inner orbital rim

D No swelling, redness, or drainage from the puncta should be observed when it is pressed. Regurgitation of fluid from the puncta, when pressed, indicates duct blockage. The lacrimal glands are not functional at birth.

The nurse is aware that the four areas in the body where lymph nodes are accessible are the: a. Head, breasts, groin, and abdomen. b. Arms, breasts, inguinal area, and legs. c. Head and neck, arms, breasts, and axillae. d. Head and neck, arms, inguinal area, and axillae.

D Nodes are located throughout the body, but they are accessible to examination only in four areas: head and neck, arms, inguinal region, and axillae.

During an assessment of the sclera of a black patient, the nurse would consider which of these an expected finding? a. Yellow fatty deposits over the cornea b. Pallor near the outer canthus of the lower lid c. Yellow color of the sclera that extends up to the iris d. Presence of small brown macules on the sclera

D Normally in dark-skinned people, small brown macules may be observed in the sclera.

The nurse is reviewing the development of the newborn infant. Regarding the sinuses, which statement is true in relation to a newborn infant? a. Sphenoid sinuses are full size at birth. b. Maxillary sinuses reach full size after puberty. c. Frontal sinuses are fairly well developed at birth. d. Maxillary and ethmoid sinuses are the only sinuses present at birth.

D Only the maxillary and ethmoid sinuses are present at birth. The sphenoid sinuses are minute at birth and develop after puberty. The frontal sinuses are absent at birth, are fairly well developed at age 7 to 8 years, and reach full size after puberty

Pigmentary demarcation lines are a. a precursor of skin cancer b. more common on people with fair skin c. commonly seen on the face d. a normal variation e. may decrease during pregnancy

D - a normal variation

An ophthalmic examination reveals papilledema. The nurse is aware that this finding indicates: a. Retinal detachment. b. Diabetic retinopathy. c. Acute-angle glaucoma. d. Increased intracranial pressure.

D Papilledema, or choked disk, is a serious sign of increased intracranial pressure, which is caused by a space-occupying mass such as a brain tumor or hematoma. This pressure causes venous stasis in the globe, showing redness, congestion, and elevation of the optic disc, blurred margins, hemorrhages, and absent venous pulsations. Papilledema is not associated with the conditions in the other responses.

For the first time, the nurse is seeing a patient who has no history of nutrition-related problems. The initial nutritional screening should include which activity? a. Calorie count of nutrients b. Anthropometric measures c. Complete physical examination d. Measurement of weight and weight history

D Parameters used for nutrition screening typically include weight and weight history, conditions associated with increased nutritional risk, diet information, and routine laboratory data. The other responses reflect a more in-depth assessment rather than a screening.

A newborn infant has Down syndrome. During the skin assessment, the nurse notices a transient mottling in the trunk and extremities in response to the cool temperature in the examination room. The infant's mother also notices the mottling and asks what it is. The nurse knows that this mottling is called: a. Café au lait. b. Carotenemia. c. Acrocyanosis. d. Cutis marmorata.

D Persistent or pronounced cutis marmorata occurs with infants born with Down syndrome or those born prematurely and is a transient mottling in the trunk and extremities in response to cool room temperatures. A café au lait spot is a large round or oval patch of light-brown pigmentation. Carotenemia produces a yellow-orange color in light-skinned persons. Acrocyanosis is a bluish color around the lips, hands and fingernails, and feet and toenails.

A patient has had arthritic pain in her hips for several years since a hip fracture. She is able to move around in her room and has not offered any complaints so far this morning. However, when asked, she states that her pain is "bad this morning" and rates it at an 8 on a 1-to-10 scale. What does the nurse suspect? The patient: a. Is addicted to her pain medications and cannot obtain pain relief. b. Does not want to trouble the nursing staff with her complaints. c. Is not in pain but rates it high to receive pain medication. d. Has experienced chronic pain for years and has adapted to it.

D Persons with chronic pain typically try to give little indication that they are in pain and, over time, adapt to the pain. As a result, they are at risk for underdetection.

The mother of a 2-year-old toddler is concerned about the upcoming placement of tympanostomy tubes in her son's ears. The nurse would include which of these statements in the teaching plan? a. The tubes are placed in the inner ear. b. The tubes are used in children with sensorineural loss. c. The tubes are permanently inserted during a surgical procedure. d. The purpose of the tubes is to decrease the pressure and allow for drainage.

D Polyethylene tubes are surgically inserted into the eardrum to relieve middle ear pressure and to promote drainage of chronic or recurrent middle ear infections. Tubes spontaneously extrude in 6 months to 1 year.

While obtaining a health history from the mother of a 1-year-old child, the nurse notices that the baby has had a bottle in his mouth the entire time. The mother states, "It makes a great pacifier." The best response by the nurse would be: a. "You're right. Bottles make very good pacifiers." b. "Using a bottle as a pacifier is better for the teeth than thumb-sucking." c. "It's okay to use a bottle as long as it contains milk and not juice." d. "Prolonged use of a bottle can increase the risk for tooth decay and ear infections."

D Prolonged bottle use during the day or when going to sleep places the infant at risk for tooth decay and middle ear infections.

A 4-year-old boy is brought to the emergency department by his mother. She says he points to his stomach and says, "It hurts so bad." Which pain assessment tool would be the best choice when assessing this child's pain? a. Descriptor Scale b. Numeric rating scale c. Brief Pain Inventory d. Faces Pain Scale—Revised (FPS-R)

D Rating scales can be introduced at the age of 4 or 5 years. The FPS-R is designed for use by children and asks the child to choose a face that shows "how much hurt (or pain) you have now." Young children should not be asked to rate pain by using numbers.

A man has come in to the clinic for a skin assessment because he is worried he might have skin cancer. During the skin assessment the nurse notices several areas of pigmentation that look greasy, dark, and "stuck on" his skin. Which is the best prediction? a. Senile lentigines, which do not become cancerous b. Actinic keratoses, which are precursors to basal cell carcinoma c. Acrochordons, which are precursors to squamous cell carcinoma d. Seborrheic keratoses, which do not become cancerous

D Seborrheic keratoses appear like dark, greasy, "stuck-on" lesions that primarily develop on the trunk. These lesions do not become cancerous. Senile lentigines are commonly called liver spots and are not precancerous. Actinic (senile or solar) keratoses are lesions that are red-tan scaly plaques that increase over the years to become raised and roughened. They may have a silvery-white scale adherent to the plaque. They occur on sun-exposed surfaces and are directly related to sun exposure. They are premalignant and may develop into squamous cell carcinoma. Acrochordons are skin tags and are not precancerous.

The nurse is discussing appropriate foods with the mother of a 3-year-old child. Which of these foods are recommended? a. Foods that the child will eat, no matter what they are b. Foods easy to hold such as hot dogs, nuts, and grapes c. Any foods, as long as the rest of the family is also eating them d. Finger foods and nutritious snacks that cannot cause choking

D Small portions, finger foods, simple meals, and nutritious snacks help improve the dietary intake of young children. Foods likely to be aspirated should be avoided (e.g., hot dogs, nuts, grapes, round candies, popcorn).

During a morning assessment, the nurse notices that the patient's sputum is frothy and pink. Which condition could this finding indicate? a. Croup b. Tuberculosis c. Viral infection d. Pulmonary edema

D Sputum, alone, is not diagnostic, but some conditions have characteristic sputum production. Pink, frothy sputum indicates pulmonary edema or it may be a side effect of sympathomimetic medications. Croup is associated with a barking cough, not sputum production. Tuberculosis may produce rust-colored sputum. Viral infections may produce white or clear mucoid sputum.

The nurse is performing an eye-screening clinic at a daycare center. When examining a 2-year-old child, the nurse suspects that the child has a "lazy eye" and should: a. Examine the external structures of the eye. b. Assess visual acuity with the Snellen eye chart. c. Assess the child's visual fields with the confrontation test. d. Test for strabismus by performing the corneal light reflex test.

D Testing for strabismus is done by performing the corneal light reflex test and the cover test. The Snellen eye chart and confrontation test are not used to test for strabismus.

The nurse is assessing a patient's eyes for the accommodation response and would expect to see which normal finding? a. Dilation of the pupils b. Consensual light reflex c. Conjugate movement of the eyes d. Convergence of the axes of the eyes

D The accommodation reaction includes pupillary constriction and convergence of the axes of the eyes. The other responses are not correct.

A 19-year-old college student is brought to the emergency department with a severe headache he describes as, "Like nothing I've ever had before." His temperature is 40° C, and he has a stiff neck. The nurse looks for other signs and symptoms of which problem? a. Head injury b. Cluster headache c. Migraine headache d. Meningeal inflammation

D The acute onset of neck stiffness and pain along with headache and fever occurs with meningeal inflammation. A severe headache in an adult or child who has never had it before is a red flag. Head injury and cluster or migraine headaches are not associated with a fever or stiff neck.

The nurse educator is preparing an education module for the nursing staff on the dermis layer of skin. Which of these statements would be included in the module? The dermis: a. Contains mostly fat cells. b. Consists mostly of keratin. c. Is replaced every 4 weeks. d. Contains sensory receptors.

D The dermis consists mostly of collagen, has resilient elastic tissue that allows the skin to stretch, and contains nerves, sensory receptors, blood vessels, and lymphatic vessels. It is not replaced every 4 weeks.

The nurse is assessing children in a pediatric clinic. Which statement is true regarding the measurement of blood pressure in children? a. Blood pressure guidelines for children are based on age. b. Phase II Korotkoff sounds are the best indicator of systolic blood pressure in children. c. Using a Doppler device is recommended for accurate blood pressure measurements until adolescence. d. The disappearance of phase V Korotkoff sounds can be used for the diastolic reading in children.

D The disappearance of phase V Korotkoff sounds can be used for the diastolic reading in children, as well as in adults

The nurse educator is preparing an education module for the nursing staff on the epidermal layer of skin. Which of these statements would be included in the module? The epidermis is: a. Highly vascular. b. Thick and tough. c. Thin and nonstratified. d. Replaced every 4 weeks.

D The epidermis is thin yet tough, replaced every 4 weeks, avascular, and stratified into several zones.

The nurse is reviewing the structures of the ear. Which of these statements concerning the eustachian tube is true? a. The eustachian tube is responsible for the production of cerumen. b. It remains open except when swallowing or yawning. c. The eustachian tube allows passage of air between the middle and outer ear. d. It helps equalize air pressure on both sides of the tympanic membrane.

D The eustachian tube allows an equalization of air pressure on each side of the tympanic membrane so that the membrane does not rupture during, for example, altitude changes in an airplane. The tube is normally closed, but it opens with swallowing or yawning.

Before auscultating the abdomen for the presence of bowel sounds on a patient, the nurse should: a. Warm the endpiece of the stethoscope by placing it in warm water. b. Leave the gown on the patient to ensure that he or she does not get chilled during the examination. c. Ensure that the bell side of the stethoscope is turned to the "on" position. d. Check the temperature of the room, and offer blankets to the patient if he or she feels cold.

D The examination room should be warm. If the patient shivers, then the involuntary muscle contractions can make it difficult to hear the underlying sounds. The end of the stethoscope should be warmed between the examiner's hands, not with water. The nurse should never listen through a gown. The diaphragm of the stethoscope should be used to auscultate for bowel sounds.

The tissue that connects the tongue to the floor of the mouth is the: a. Uvula. b. Palate. c. Papillae. d. Frenulum.

D The frenulum is a midline fold of tissue that connects the tongue to the floor of the mouth. The uvula is the free projection hanging down from the middle of the soft palate. The palate is the arching roof of the mouth. Papillae are the rough, bumpy elevations on the tongue's dorsal surface.

The mother of a 2-year-old is concerned because her son has had three ear infections in the past year. What would be an appropriate response by the nurse? a. "It is unusual for a small child to have frequent ear infections unless something else is wrong." b. "We need to check the immune system of your son to determine why he is having so many ear infections." c. "Ear infections are not uncommon in infants and toddlers because they tend to have more cerumen in the external ear." d. "Your son's eustachian tube is shorter and wider than yours because of his age, which allows for infections to develop more easily."

D The infant's eustachian tube is relatively shorter and wider than the adult's eustachian tube, and its position is more horizontal; consequently, pathogens from the nasopharynx can more easily migrate through to the middle ear. The other responses are not appropriate.

When examining an infant, the nurse should examine which area first? a. Ear b. Nose c. Throat d. Abdomen

D The least-distressing steps are performed first, saving the invasive steps of the examination of the eye, ear, nose, and throat until last.

In an individual with otitis externa, which of these signs would the nurse expect to find on assessment? a. Rhinorrhea b. Periorbital edema c. Pain over the maxillary sinuses d. Enlarged superficial cervical nodes

D The lymphatic drainage of the external ear flows to the parotid, mastoid, and superficial cervical nodes. The signs are severe swelling of the canal, inflammation, and tenderness. Rhinorrhea, periorbital edema, and pain over the maxillary sinuses do not occur with otitis externa.

Newborns are more vulnerable to hypothermia because of a. the presence of coarse terminal hair. b. desquamation of the stratum corneum. c. their covering of vernix caseosa. d. a poorly developed subcutaneous fat layer. e. excessive secretion of eccrine sweat glands.

D - a poorly developed subcutaneous fat layer

During a checkup, a 22-year-old woman tells the nurse that she uses an over-the-counter nasal spray because of her allergies. She also states that it does not work as well as it used to when she first started using it. The best response by the nurse would be: a. "You should never use over-the-counter nasal sprays because of the risk of addiction." b. "You should try switching to another brand of medication to prevent this problem." c. "Continuing to use this spray is important to keep your allergies under control." d. "Using these nasal medications irritates the lining of the nose and may cause rebound swelling."

D The misuse of over-the-counter nasal medications irritates the mucosa, causing rebound swelling, which is a common problem.

The nurse is assessing for clubbing of the fingernails and expects to find: a. Nail bases that are firm and slightly tender. b. Curved nails with a convex profile and ridges across the nails. c. Nail bases that feel spongy with an angle of the nail base of 150 degrees. d. Nail bases with an angle of 180 degrees or greater and nail bases that feel spongy.

D The normal nail is firm at its base and has an angle of 160 degrees. In clubbing, the angle straightens to 180 degrees or greater and the nail base feels spongy.

An examiner is using an ophthalmoscope to examine a patient's eyes. The patient has astigmatism and is nearsighted. The use of which of these techniques would indicate that the examination is being correctly performed? a. Using the large full circle of light when assessing pupils that are not dilated b. Rotating the lens selector dial to the black numbers to compensate for astigmatism c. Using the grid on the lens aperture dial to visualize the external structures of the eye d. Rotating the lens selector dial to bring the object into focus

D The ophthalmoscope is used to examine the internal eye structures. It can compensate for nearsightedness or farsightedness, but it will not correct for astigmatism. The grid is used to assess size and location of lesions on the fundus. The large full spot of light is used to assess dilated pupils. Rotating the lens selector dial brings the object into focus.

The nurse is preparing to use an otoscope for an examination. Which statement is true regarding the otoscope? The otoscope: a. Is often used to direct light onto the sinuses. b. Uses a short, broad speculum to help visualize the ear. c. Is used to examine the structures of the internal ear. d. Directs light into the ear canal and onto the tympanic membrane.

D The otoscope directs light into the ear canal and onto the tympanic membrane that divides the external and middle ear. A short, broad speculum is used to visualize the nares.

A patient has a normal pupillary light reflex. The nurse recognizes that this reflex indicates that: a. The eyes converge to focus on the light. b. Light is reflected at the same spot in both eyes. c. The eye focuses the image in the center of the pupil. d. Constriction of both pupils occurs in response to bright light.

D The pupillary light reflex is the normal constriction of the pupils when bright light shines on the retina. The other responses are not correct.

The nurse is preparing to perform a physical assessment. The correct action by the nurse is reflected by which statement? The nurse: a. Performs the examination from the left side of the bed. b. Examines tender or painful areas first to help relieve the patient's anxiety. c. Follows the same examination sequence, regardless of the patient's age or condition. d. Organizes the assessment to ensure that the patient does not change positions too often.

D The steps of the assessment should be organized to ensure that the patient does not change positions too often. The sequence of the steps of the assessment may differ, depending on the age of the person and the examiner's preference. Tender or painful areas should be assessed last.

The nurse is assessing a patient's pain. The nurse knows that the most reliable indicator of pain would be the: a. Patient's vital signs. b. Physical examination. c. Results of a computerized axial tomographic scan. d. Subjective report.

D The subjective report is the most reliable indicator of pain. Physical examination findings can lend support, but the clinician cannot exclusively base the diagnosis of pain on physical assessment findings.

When checking for proper blood pressure cuff size, which guideline is correct? a. The standard cuff size is appropriate for all sizes. b. The length of the rubber bladder should equal 80% of the arm circumference. c. The width of the rubber bladder should equal 80% of the arm circumference. d. The width of the rubber bladder should equal 40% of the arm circumference.

D The width of the rubber bladder should equal 40% of the circumference of the person's arm. The length of the bladder should equal 80% of this circumference.

When assessing the quality of a patient's pain, the nurse should ask which question? a. "When did the pain start?" b. "Is the pain a stabbing pain?" c. "Is it a sharp pain or dull pain?" d. "What does your pain feel like?"

D To assess the quality of a person's pain, the patient is asked to describe the pain in his or her own words.

A mother brings her child into the clinic for an examination of the scalp and hair. She states that the child has developed irregularly shaped patches with broken-off, stublike hair in some places; she is worried that this condition could be some form of premature baldness. The nurse tells her that it is: a. Folliculitis that can be treated with an antibiotic. b. Traumatic alopecia that can be treated with antifungal medications. c. Tinea capitis that is highly contagious and needs immediate attention. d. Trichotillomania; her child probably has a habit of absentmindedly twirling her hair.

D Trichotillomania, self-induced hair loss, is usually due to habit. It forms irregularly shaped patches with broken-off, stublike hairs of varying lengths. A person is never completely bald. It occurs as a child absentmindedly rubs or twirls the area while falling asleep, reading, or watching television. (See Table 12-12, Abnormal Conditions of Hair, for descriptions of the other terms.)

The nurse is taking the history of a patient who may have a perforated eardrum. What would be an important question in this situation? a. "Do you ever notice ringing or crackling in your ears?" b. "When was the last time you had your hearing checked?" c. "Have you ever been told that you have any type of hearing loss?" d. "Is there any relationship between the ear pain and the discharge you mentioned?"

D Typically with perforation, ear pain occurs first, stopping with a popping sensation, and then drainage occurs.

The nurse is performing an assessment. Which of these findings would cause the greatest concern? a. Painful vesicle inside the cheek for 2 days b. Presence of moist, nontender Stensen's ducts c. Stippled gingival margins that snugly adhere to the teeth d. Ulceration on the side of the tongue with rolled edges

D Ulceration on the side or base of the tongue or under the tongue raises the suspicion of cancer and must be investigated. The risk of early metastasis is present because of rich lymphatic drainage. The vesicle may be an aphthous ulcer, which is painful but not dangerous. The other responses are normal findings.

An older adult patient in a nursing home has been receiving tube feedings for several months. During an oral examination, the nurse notes that patient's gums are swollen, ulcerated, and bleeding in some areas. The nurse suspects that the patient has what condition? a. Rickets b. Vitamin A deficiency c. Linoleic-acid deficiency d. Vitamin C deficiency

D Vitamin C deficiency causes swollen, ulcerated, and bleeding gums, known as scorbutic gums. Rickets is a condition related to vitamin D and calcium deficiencies in infants and children. Linoleic-acid deficiency causes eczematous skin. Vitamin A deficiency causes Bitot spots and visual problems.

When examining an older adult, the nurse should use which technique? a. Avoid touching the patient too much. b. Attempt to perform the entire physical examination during one visit. c. Speak loudly and slowly because most aging adults have hearing deficits. d. Arrange the sequence of the examination to allow as few position changes as possible.

D When examining the older adult, arranging the sequence of the examination to allow as few position changes as possible is best. Physical touch is especially important with the older person because other senses may be diminished.

The nurse notices that a patient's submental lymph nodes are enlarged. In an effort to identify the cause of the node enlargement, the nurse would assess the patient's: a. Infraclavicular area. b. Supraclavicular area. c. Area distal to the enlarged node. d. Area proximal to the enlarged node.

D When nodes are abnormal, the nurse should check the area into which they drain for the source of the problem. The area proximal (upstream) to the location of the abnormal node should be explored.

The nurse is performing a general survey of a patient. Which finding is considered normal? a. When standing, the patient's base is narrow. b. The patient appears older than his stated age. c. Arm span (fingertip to fingertip) is greater than the height. d. Arm span (fingertip to fingertip) equals the patient's height.

D When performing the general survey, the patient's arm span (fingertip to fingertip) should equal the patient's height. An arm span that is greater than the person's height may indicate Marfan syndrome. The base should be wide when the patient is standing, and an older appearance than the stated age may indicate a history of a chronic illness or chronic alcoholism.

A patient visits the clinic because he has recently noticed that the left side of his mouth is paralyzed. He states that he cannot raise his eyebrow or whistle. The nurse suspects that he has: a. Cushing syndrome. b. Parkinson disease. c. Bell palsy. d. Experienced a cerebrovascular accident (CVA) or stroke.

D With an upper motor neuron lesion, as with a CVA, the patient will have paralysis of lower facial muscles, but the upper half of the face will not be affected owing to the intact nerve from the unaffected hemisphere. The person is still able to wrinkle the forehead and close the eyes. (See Table 13-4, Abnormal Facial Appearances with Chronic Illnesses, for descriptions of the other responses.)

The nurse notices that a school-aged child has bluish-white, red-based spots in her mouth that are elevated approximately 1 to 3 mm. What other signs would the nurse expect to find in this patient? a. Pink, papular rash on the face and neck b. Pruritic vesicles over her trunk and neck c. Hyperpigmentation on the chest, abdomen, and back of the arms d. Red-purple, maculopapular, blotchy rash behind the ears and on the face

D With measles (rubeola), the examiner assesses a red-purple, blotchy rash on the third or fourth day of illness that appears first behind the ears, spreads over the face, and then over the neck, trunk, arms, and legs. The rash appears coppery and does not blanch. The bluish-white, red-based spots in the mouth are known as Koplik spots.

A patient is asked to indicate on a form how many times he eats a specific food. This method describes which of these tools for obtaining dietary information? a. Food diary b. Calorie count c. 24-hour recall d. Food-frequency questionnaire

D With this tool, information is collected on how many times per day, week, or month the individual eats particular foods, which provides an estimate of usual intake.

A 72-year-old patient has a history of hypertension and chronic lung disease. An important question for the nurse to include in the health history would be: a. "Do you use a fluoride supplement?" b. "Have you had tonsillitis in the last year?" c. "At what age did you get your first tooth?" d. "Have you noticed any dryness in your mouth?"

D Xerostomia (dry mouth) is a side effect of many drugs taken by older people, including antidepressants, anticholinergics, antispasmodics, antihypertensives, antipsychotics, and bronchodilators.

You are conducting a preschool examination on a 5-year-old child. Which injury would most l was being abused? a. Recent bruising over both knees b. A healed laceration under the chin c. A bruise on the right shin with associated abrasion of tissue d. Bruises in various stages of resolution over body soft tissues e. A scab on the elbow

D - Bruises in various stages of resolution over body soft tissues Toddlers and older children who bruise themselves accidentally are bruised over bony promin chin, and shin. Bruises over soft tissues are more consistent with abuse.

A 29-year-old white woman appears jaundiced. A cause of liver disease has been excluded. ask? a. Whether she had unprotected sex b. If she has a history of diabetes mellitus c. Whether she has unusual bleeding problems d. If she eats a lot of yellow and orange vegetables e. If she has a family history of peripheral vascular disease

D - If she eats a lot of yellow and orange vegetables

Which nodes are also called Virchow nodes? a. Internal mammary b. Anterior axillary c. Deep cervical d. Supraclavicular e. Preauricular

D - Supraclavicular

What term would you use to document a respiratory rate greater than 20 breaths/min in an ad a. Dyspnea b. Orthopnea c. Platypnea d. Tachypnea e. Cheyne-Stokes

D - Tachypnea

Which nodes are most often associated with inflammation? a. Shotty b. Movable c. Fixed d. Tender e. Matted

D - Tender Tenderness is almost always indicative of inflammation.

When there is consolidation in the lung tissue, the breath sounds are louder and easier to he produces softer sounds. This is because a. consolidation will echo in the chest. b. consolidation is a poor conductor of sound. c. air-filled lung sounds are from smaller spaces. d. air-filled lung tissue is an insulator of sound. e. consolidation causes hyperinflation of the lungs.

D - air-filled lung tissue is an insulator of sound

Adventitious breath sounds previously referred to as rales has been replaced with the term a. wheezes. b. crunches. c. vesicular. d. crackles. e. rhonchi.

D - crackles

During chest assessment, you note the patient's voice quality while you are auscultating the intensified, there is a nasal quality to the voice, and e's sound like a's. This sound described a. sonorous. b. bronchophony. c. pectoriloquy. d. egophony. e. resonance.

D - egophony.

Painful vesicles are associated with a. psoriasis. b. pityriasis rosea. c. paronychia. d. herpes zoster. e. rosacea.

D - herpes zoster. Herpes zoster (shingles) produces painful itching or burning of the dermatome area. Psoriasi infection and rosacea may be tender to touch and do not produce vesicles.

Expected respiratory changes of normal aging include a. increased chest expansion. b. more frequent use of respiratory muscles. c. accentuated lumbar curve. d. more prominent bony structures. e. flattening of the dorsal thoracic curve.

D - more prominent bony structures.

Normal supraclavicular lymph nodes are a. clustered in a capsule. b. firm and discrete. c. less than 3 cm in diameter. d. not palpable. e. matted.

D - not palpable.

Brittle nails are typical findings in a. adolescents. b. infants. c. pregnant women. d. older adults. e. children.

D - older adults.

Fetal gas exchange is mediated by the a. pleura. b. heart. c. amniotic fluid. d. placenta. e. lungs.

D - placenta.

During pregnancy, the leukocyte count normally increases because of more a. reticulocytes. b. lymphocytes. c. atypical lymphocytes. d. polymorphonuclear leukocytes. e. monocytes.

D - polymorphonuclear leukocytes The type of leukocyte that increases during pregnancy is the polymorphonuclear leukocytes basophils), not the lymphocytes (including monocytes), reticulocytes, or atypical lymphocyte

Linea nigra is commonly found on the abdomens of a. newborns. b. infants and children. c. adolescents. d. pregnant women. e. older adults.

D - pregnant women. Pregnant women commonly develop pigmentation of the abdomen from the symphysis pubis

As you take vital signs on Mr. B, age 78 years, you note that his respirations are 40 breaths/ mucosa is pink. Concerning Mr. B's respirations, you would a. document his rate as normal. b. do nothing because his color is pink. c. note that his rate is below normal. d. report that he has an above-average rate. e. ignore one abnormal result.

D - report that he has an above-average rate

The examiner percusses for diaphragmatic excursion along the a. vertebral column. b. midvertebral line. c. midaxillary line. d. scapular line. e. sternum.

D - scapular line.

Pigmented, raised, warty lesions over the face and trunk should be assessed by an experien a. cutaneous tags from lentigines. b. furuncles from folliculitis. c. sebaceous hyperplasia from eczema. d. seborrheic keratoses from actinic keratoses. e. herpes zoster from varicella.

D - seborrheic keratoses from actinic keratoses. Actinic keratoses have malignant potential, and seborrheic keratoses do not. Because they c practitioner should make the determination.

The most important clue to the diagnosis of a child with an immunodeficiency disease is a. family history. b. illness in siblings. c. previous hospitalizations. d. serious recurring infections. e. cervical adenitis.

D - serious recurring infections

The most common causes of acute suppurative lymphadenitis are a. HIV and herpes zoster. b. Haemophilus influenzae and adenovirus. c. herpes simplex type 1 and type 2. d. streptococcal and staphylococcal organisms. e. mumps.

D - streptococcal and staphylococcal organisms

A signal for alarm during newborn chest assessment is a. crackles. b. rhonchi. c. gurgles from the gastrointestinal tract. d. stridor. e. a mobile xiphoid.

D - stridor.

The diaphragm of the stethoscope is better than the bell for auscultation of the lungs becaus a. amplifies all types of sounds. b. filters extraneous sounds. c. pinpoints focal sound areas. d. transmits high-pitched sounds. e. transmits low-pitched sounds.

D - transmits high-pitched sounds.

When examining lymph nodes near a joint in the arm or leg, which of the following maneuver a. Extension of the extremity b. Circumduction of the extremity c. Abduction of the extremity d. Rotation of the extremity e. Flexion of the extremity

E - Flexion of the extremity

A 5-year-old child with discrete vesicles on an erythemic base (dew drops on a rose petal ap are spreading to the trunk. The child has a low-grade fever and feels tired. What is the nurse a. Ask about a family history of rashes. b. Inquire about other patterns of physical abuse. c. Inspect the buccal mucosa for Koplik spots. d. Inform the parent that this will resolve within a couple of days. e. Teach infectious control measures.

E - Teach infectious control measures. The description of this child's complaint is a varicella rash, not physical abuse or rubeola. Ch disease and can be prevented by immunization. The period of communicability lasts from 1 o until all the vesicles have crusted over, which usually takes about 1 week.

Which of the following is a noncandidal fungal infection? a. Pityriasis rosea b. Psoriasis c. Milia d. Rosacea e. Tinea corporis

E - Tinea corporis Tinea corporis is the only listed fungal infection (noncandidal);

A Dennie-Morgan fold is probably caused by a. birth trauma. b. high fever. c. excess adipose tissue. d. kidney disease. e. chronic rubbing.

E - chronic rubbing. : Persons with chronic atopic or allergic conditions tend to rub their eyes so much that it caus the eye, called the Dennie-Morgan fold.

Tangential lighting is best used for inspecting skin a. color. b. turgor. c. exudates. d. symmetry. e. contour.

E - contour.

Women with terminal hair growth in a male distribution pattern should receive further evaluati a. circulation condition. b. gastrointestinal disorder. c. inflammatory state. d. nutritional deficit. e. endocrine disorder.

E - endocrine disorder.

Enlarged inguinal nodes are likely to be associated with a. cat scratch disease. b. pelvic inflammatory disease. c. uterine cancer. d. testicular cancer. e. genital herpes.

E - genital herpes.

Your trauma patient has no auscultated breath sounds in the right lung field. You can hear ad cause of this abnormality could be that the patient a. has a closed head injury. b. has minimal fluid in the pleural space. c. is moaning and in severe pain. d. is receiving high oxygen flow. e. has a pneumothorax.

E - has a pneumothorax.

To accommodate the enlarging uterus of pregnancy, the chest changes result in a. intercostal muscle atrophy. b. lowering of the resting diaphragm. c. decreased alveoli expansion. d. decreased diaphragmatic movement. e. increased costal angle.

E - increased costal angle.

An expected finding from chest palpation in the adult would be a. costal angle of 120 degrees. b. cracking over the sternal notch. c. greater right chest expansion. d. crepitus. e. inflexibility of the xiphoid.

E - inflexibility of the xiphoid.

Cells that line the lymph node sinuses perform the specific function of a. fat absorption. b. fetal immunization. c. hematopoiesis. d. platelet production. e. phagocytosis.

E - phagocytosis. Lymph nodes defend against the invasion of microorganisms by phagocytosis.

Which structure is the site of new nail growth? a. Cuticle b. Paronychium c. Eponychium d. Nail bed e. Matrix

E - sex hormones

Nodes that are palpable just inferior to the chin are the a. infraclavicular. b. Virchow. c. submandibular. d. parotid. e. submental.

E - submental. Nodes most inferior to the chin and distal from the mandible are the submental nodes.

Pale, shiny skin of the lower extremities may reflect a. excessive steroids. b. a history of vigorous exercise. c. peptic ulcer disease. d. vasculitis. e. systemic disease.

E - systemic disease.


Conjuntos de estudio relacionados

B6 Different Media in Social Studies Propoganda, sources, emotion self preservation

View Set

Current Events Comprehension Questions 2022

View Set

Principles 2: Unit 8: Property Management

View Set

Level 9 - "Texas Real Estate License Act" - Chapter 7 - "Subchapter F - Broker-Lawyer Committee"

View Set